Чемпионат: Окский марафон - 2014 (Пущино) URL: /znatoki/boris/reports/201402Puschino.html Дата: 01-Feb-2014 Тур: 1 тур. "Лес гинкго" (Ивантеевка - Мытищи - Москва - Воронеж - Смоленск) Редактор: Антон Волосатов (Ивантеевка) Инфо: Редактор благодарит Константина Сахарова (Ивантеевка), Руслана Хаиткулова (Стерлитамак - Нью-Йорк), Вадима Штанникова (Королёв), Александра Коробейникова (Саратов), а также команду "Румба" в составе: Денис Беликов, Александр Володин, Елена Герасимова, Марина Межова, Ольга Михайлова и Александр Московкин (все - Москва) за тестирование тура и ценные советы. Вопрос 1: <раздатка> | представитель одного из царств | то же, что "отнимай силой" | продолговатый ящик с крышкой | то же, что "неискусен", "невежлив" Назовите стихотворное произведение, в котором описанные четыре слова используются не в своих прямых значениях. Ответ: "Хорошее отношение к лошадям". Комментарий: Слова "гриб", "грабь", "гроб" и "груб" используются Маяковским как звукоподражательные стуку копыт о мостовую, а не в своих привычных значениях. Текст раздатки написан лесенкой, что служит подсказкой. Это был так называемый тотемный вопрос, приуроченный к началу года Синей Лошади, наступившему сегодня. Источник: 1. http://www.feb-web.ru/feb/mayakovsky/texts/mp0/mp2/mp2-025-.htm 2. http://dic.academic.ru/dic.nsf/ogegova/42618/ 3. http://dic.academic.ru/dic.nsf/ushakov/782178/ 4. http://dic.academic.ru/dic.nsf/ogegova/42810/ 5. http://dic.academic.ru/dic.nsf/ushakov/783370/ Автор: Антон Волосатов (Ивантеевка) Вопрос 2: Один интернет-пользователь рассказывает, как увидел у своего друга-программиста листок, на котором тот расписывал ручку. Какие два слова многократно на нем повторялись? Ответ: Hello world. Зачет: Привет, мир! Комментарий: Обычно первые слова для упражнений в языках программирования. Источник: http://bash.im/quote/415022/ Автор: Антон Волосатов (Ивантеевка) Вопрос 3: Действие фильма "Пробуждение жизни" предположительно происходит после смерти главного героя. В начале фильма его подвозят автостопом на необычной машине, которая, по одной из версий, является аллюзией на НЕЕ. Назовите ЕЕ двумя словами. Ответ: Ладья Харона. Зачет: Лодка Харона. Комментарий: Автомобиль, выполненный в виде катера, везет героя к начальной точке его путешествия. Источник: 1. Х/ф "Пробуждение жизни" (реж. Ричард Линклейтер, 2001 г.). 2. Версия автора вопроса. Автор: Антон Волосатов (Ивантеевка) Вопрос 4: Когда заканчивался нэп и начинались массовые репрессии, мрачно шутили, что из ресторана на крыше гостиницы "Европейская", в котором кутили богатые дельцы, можно увидеть ИХ. Вопреки пожеланию литературного персонажа, немецкий философ ИХ так и не посетил. Назовите ИХ. Ответ: Соловки. Зачет: Соловецкие острова. Комментарий: Намек на то, что разбогатевших членов общества часто начинали преследовать и отправлять в лагеря. Иван Бездомный из "Мастера и Маргариты" сгоряча желал отправить Иммануила Канта на Соловки. Источник: 1. И. Головкина (Римская-Корсакова). Лебединая песнь (Побежденные). - Ч. 2. - Гл. 10. 2. http://www.solovki.ca/writers_023/bulgakov.php Автор: Антон Волосатов (Ивантеевка) Вопрос 5: [Ведущему: проартикулировать буквы "В", "Г", "А" и "Я", например, при помощи личных имен.] Внимание, в вопросе есть замены. Размышляя о перспективных направлениях транспортной системы, Никита Моисеев говорил, что Русь развивалась от В до Г, а новая Россия может простираться от А до Я. Какие слова мы заменили словами "от А до Я"? Ответ: Из англичан в японцы. Зачет: По смыслу с упоминанием англичан и японцев. Комментарий: Путь из варяг в греки был очень важен для экономики Руси. У сегодняшней России есть возможность развивать северный морской путь от Британских до Японских островов. Источник: http://www.rusnor.org/pubs/articles/6344.htm Автор: Антон Волосатов (Ивантеевка) Вопрос 6: В кинокомедии "Airplane!" [Эйрплэйн] одного из пилотов коллеги часто невольно отвлекают во время переговоров. Назовите имя этого пилота. Ответ: Роджер. Комментарий: Аналогом русскоязычного "Вас понял" у американцев служит фраза "Roger that!" или "Roger!". Во время переговоров с диспетчером пилот с этим именем постоянно откликался, думая, что обращаются к нему. Источник: http://www.sfy.ru/?script=airplane_ts (поиск по выражению "cleared for take off") Автор: Антон Волосатов (Ивантеевка) Вопрос 7: Вскоре после кончины писателя Сэмюэля Беккета, друга и помощника Джеймса Джойса, был выпущен фильм, посвященный творчеству Беккета. Напишите первое слово из русского перевода его названия. Ответ: Поминки. Комментарий: Фильм называется "Поминки по Сэму" - явная аллюзия на роман Джойса "Поминки по Финнегану". Источник: 1. http://ru.wikipedia.org/wiki/Беккет,_Сэмюэл 2. http://rutracker.org/forum/viewtopic.php?t=2973050 Автор: Антон Волосатов (Ивантеевка) Вопрос 8: Перед выездом на марафон, чтобы предотвратить возможные конфликты в команде по поводу места за столом, автор вопроса составил ИХ. Из-за НИХ однажды пострадал герой поэмы. Назовите ИХ двумя словами. Ответ: Индивидуальные графики. Зачет: Личные/персональные графики. Комментарий: Чтобы всё было честно и справедливо, для каждого был составлен график игры. Веничка Ерофеев изучал подчиненных, составляя графики выпитого ими спиртного. Источник: 1. ЛОАВ. 2. http://www.moskva-petushki.ru/works/moskva-petushki/12 Автор: Антон Волосатов (Ивантеевка) Вопрос 9: <раздатка> <...> "Ах, ты, чудодей! Очистив весь край понемногу, Ты в ссылку отправил всех честных людей И - сам поднимаешь тревогу". Перед вами фрагмент диалога из стихотворения Дмитрия Минаева. Заглавие стихотворения совпадает с названием того, что появилось в Ленинграде в 1988 году. Напишите это название. Ответ: "Король и шут". Комментарий: Отправить в ссылку "всех честных людей", конечно, может только монарх, а прямо сказать ему об этом - только шут. Во втором предложении речь идет об известной панк-группе. Источник: 1. http://vivovoco.ibmh.msk.su/VV/CAMERTON/MINAEV.HTM 2. http://ru.wikipedia.org/wiki/Король_и_Шут Автор: Александр Шевченко (Москва) Вопрос 10: Внимание, словом "ИКС" в тексте вопроса заменено несколько слов. В битве при КиноскефАлах римляне не щадили никого из противников, так как не понимали греческого ИКСА. Напишите название коллектива, которое описывает наиболее известный нам ИКС. Ответ: "Руки вверх!". Комментарий: ИКС - это жест сдачи в плен. Греки при сдаче поднимали вверх копье, а римляне не воспринимали должным образом этот знак и убивали капитулянтов. Источник: Т. Моммзен. История Рима. - Кн. 3. - Гл. 7 (http://centant.spbu.ru/sno/lib/momm/3-7.htm). Автор: Антон Волосатов (Ивантеевка) Вопрос 11: Фреска в одном из греческих монастырей изображает ангела, льва, быка и орла и отсылает к видЕнию пророка Иезикииля. Какой предмет каждое из этих существ держит в руках? Ответ: Книгу. Зачет: Свиток, рукопись, Евангелие, Новый Завет, Библию. Комментарий: Считается, что таким образом пророк указал на четырех апостолов-евангелистов. Источник: http://www.angelologia.ru/iconographia/199_ikony_angelov.htm Автор: Антон Волосатов (Ивантеевка) Вопрос 12: В начале 1970-х годов один циркач, задумывая дерзкое предприятие, отправил другу открытку из США. На открытке он дорисовал некий элемент. Что этот элемент соединял? Ответ: Северную и Южную башни Всемирного Торгового Центра. Зачет: Две башни WTC, башни-близнецы и т.п. Комментарий: Французский циркач Филипп ПетИ задумал пройти по канату между вновь построенными небоскребами, о чем и дал знать другу, дорисовав соединительную линию между крышами. Источник: Д/ф "Человек на канате" (реж. Джеймс Марш, 2007 г.). Автор: Антон Волосатов (Ивантеевка) Вопрос 13: Во время подготовки перехода между башнями Всемирного Торгового Центра рассматривались такие варианты, как бейсбольная бита и радиоуправляемый самолет, но в итоге решили вооружиться ИМ. На гербе какого города есть несколько ИХ? Ответ: Великие Луки. Комментарий: Чтобы перекинуть леску для натягивания каната с одной крыши на другую, ее привязали к стреле, которой выстрелили из лука. На гербе Великих Лук - три лука. Источник: 1. Д/ф "Человек на канате" (реж. Джеймс Марш, 2007 г.). 2. http://ru.wikipedia.org/wiki/Герб_Великих_Лук Автор: Антон Волосатов (Ивантеевка) Вопрос 14: Музыкант Брайан Ферри однажды сообщил, что ему нравятся произведения 30-40-х годов XX века, и был обвинен в симпатиях к нацизму. Хотя он принес извинения, в оформлении его нового альбома обыгрывается известная картина, что делает название альбома двусмысленным. Напишите это название. Ответ: "Olympia". Зачет: "Олимпия". Комментарий: "Олимпия" - не только название фильма Лени Рифеншталь, но и картина Эдуара Мане, на которую и была аллюзия. Человеку, не видевшему обложку, название может показаться двусмысленным. Источник: 1. http://www.welt.de/wams_print/article744996/Ich-waere-gern-ein-Amateur.html 2. http://www.bryanferry.com/music/bryanferry/olympia/ Автор: Антон Волосатов (Ивантеевка) Вопрос 15: Автор рецензии на альбом дэт-металлической группы просит убрать детей подальше во время прослушивания этого альбома. В названии группы присутствует фамилия английского священника. Один остроумный журналист в статье о политике островного государства по привлечению населения также упоминает эту фамилию. Назовите ее. Ответ: Мальтус. Комментарий: Английский экономист и священнослужитель Томас Мальтус полагал, что все проблемы человечества возникают из-за избыточной тяги людей к обзаведению детьми, поэтому он проповедовал воздержание, чтобы хоть как-то уменьшить количество детей. Мальта не так давно разработала программу по оживлению экономики путем предоставления постоянного места жительства богатым гражданам других государств, о чем журналист не преминул скаламбурить, что Мальтуса она не боится. Источник: 1. http://heavymetal.about.com/od/cdreviews/tp/New-Heavy-Metal-Album-Reviews-November-26-2013.01.htm 2. http://www.medical-enc.ru/m/12/maltuzianstvo.shtml 3. http://www.alleuropa.ru/maljta-namerena-oprovergnutj-maljtusa Автор: Руслан Хаиткулов (Москва), Олег Холодов (Серпухов) Вопрос 16: (pic: 20140028.jpg) Напишите два слова, встречающиеся в обеих закрытых от вас подписях. Ответ: Made in. Зачет: Сделано в, изготовлено в, произведено в. Комментарий: Подписи гласят, что автомобиль - дешевое китайское изделие, а велосипед - образец немецкого качества, потому и держит удар лучше. Хотя, скорее всего, велосипед тоже китайского производства. :-) Источник: http://www.flickr.com/photos/charles_nouyrit/7561941878/ Автор: Ксения Кашкарова (Москва) Вопрос 17: В одном сказочном мультсериале есть персонаж по имени Макинтош. Назовите фамилию его бабушки. Ответ: [Гренни] Смит. Комментарий: Макинтош - не только компьютер или одежда, но и сорт яблок. Логично, что бабушку тоже назвали одним из сортов, название которого, помимо прочего, указывает на родственную связь. Источник: 1. http://mlp.wikia.com/wiki/Granny_Smith 2. http://www.yabloki.by/grenni-smit/ Автор: Ксения Кашкарова (Москва) Вопрос 18: Герой произведения Игоря Бажанова хочет СДЕЛАТЬ ЭТО со своей возлюбленной и оставить ей две спинки. Ответьте двумя словами: что сделать? Ответ: Разделить постель. Зачет: Разделить/поделить постель/кровать/ложе. Комментарий: "... Мне так хотелось разделить с тобой постель: // Тебе две спинки, а мне матрац". Источник: http://www.hzabei.ru/index.php?name=text&op=view&id=12 Автор: Антон Волосатов (Ивантеевка) Вопрос 19: Однажды в качестве заставки Google [гугл] выступала интерактивная форма с заголовком "Опишите, что вы здесь видите". Кому была посвящена эта заставка? Ответ: [Герману] Роршаху. Комментарий: На гугл-дудле было знаменитое пятно с предложением указать, кто что в нем видит. Источник: http://www.google.com/doodles/hermann-rorschachs-129th-birthday Автор: Антон Волосатов (Ивантеевка) Вопрос 20: <раздатка> Он встал из своего угла и, прежде чем шагнуть, осмотрел пол, как будто необходимо было хорошенько выбрать, куда поставить ногу, потом с такими же предосторожностями ступил еще раз, подтянул другую ногу и в двух метрах от Рональда с Бэпс встал как вкопанный. Назовите роман, из которого взята данная цитата. Ответ: "Игра в классики". Комментарий: Приведено начало первой главы одной из версий романа - весьма похоже на саму игру. Источник: http://www.aphorisms.su/knigi/romani/igra_v_klassiki/kniga_1/glava_14.html Автор: Антон Волосатов (Ивантеевка) Вопрос 21: [Ведущему: кавычки не озвучивать.] Одна российская группа не успела найти нового барабанщика, поэтому назвала свой гастрольный тур "ОНИ". В "вопросе" ИХ два. Назовите ИХ двумя словами. Ответ: Безударные гласные. Комментарий: Ритм-секция заметно обеднела, но вокальная сторона только выдвинулась вперед. В слове "вопросе" - два безударных гласных звука. Источник: http://www.zvuki.ru/R/P/5677/ Автор: Антон Волосатов (Ивантеевка) Вопрос 22: Персонаж братьев Вачовски носит имя еврейского происхождения и утверждает, что призван "охранять самое важное". В одном из эпизодов он получает характеристику, которая у знатоков может ассоциироваться с игрой. Воспроизведите эту характеристику одним словом. Ответ: Бескрылый. Комментарий: Речь идет о программе в облике мужчины по имени Seraph (в русском переводе - Серафим), охраняющей Пифию, - прямая аналогия с ангелом и Богом. Один из персонажей называет его бескрылым ангелом. Популярная среди знатоков игра - бескрылки. Источник: 1. http://www.x-matrix.net/Scripts/Reloaded.txt (поиск по выражению "matters most") 2. http://www.winterfeld.de/Lars/stuff/transcript_matrix_revolutions.html (поиск по выражению "angel without wings" и слову "wingless") Автор: Антон Волосатов (Ивантеевка) Вопрос 23: В пьесе Алексея Толстого "Царь Борис" жених Ксении Годуновой узнаёт, что его будущий тесть стал царем, убив царевича Дмитрия. Из какой страны прибыл этот жених? Ответ: Дания. Комментарий: Жених - датский принц (кстати, реальное историческое лицо). Узнав, что отец его возлюбленной - узурпатор и цареубийца, он испытывает чувства, подобные чувствам Гамлета. К "Гамлету" в этой трагедии вообще масса отсылок. Источник: http://www.e-reading.mobi/bookreader.php/56633/Tolstoii_-_Car_Boris.html Автор: Александр Шевченко (Москва) Вопрос 24: Персонаж одного романа говорит, что в первую очередь реклама призывает покупать не вещи, а ЭТО. Персонаж какого отечественного произведения желал, чтобы ЭТО было бесплатным? Ответ: "Пикник на обочине". Комментарий: Персонаж Пелевина говорит, что рекламируется не товар, а простое человеческое счастье. Артур Барбридж из романа Стругацких желал "счастье для всех, даром...". Спасибо всем за внимание! Источник: 1. http://pelevin.nov.ru/romans/pe-genp/10.html 2. http://lib.ru/STRUGACKIE/picnic.txt Автор: Антон Волосатов (Ивантеевка) Тур: 2 тур. "Дети" (Серпуховский район) Редактор: Максим Евланов (Харьков) Вопрос 1: Внимание, в вопросе есть замена. Статья в журнале "Вокруг света", посвященная космодромам, называлась "Космодромы - "Внимание, минутная готовность"". "Внимание, минутная готовность" - первые слова из популярной песни группы "Браво". Какие три слова мы заменили словами "Внимание, минутная готовность"? Ответ: "Ключ на старт". Комментарий: А перед командой "Ключ на старт" в циклограмме пуска идут команды "Внимание" и "Минутная готовность". Мы начинаем наш тур. Источник: 1. http://www.vokrugsveta.ru/vs/article/2902/ 2. http://www.akkord-guitar.ru/3603-bravo-klyuch-na-start.html 3. http://ru.wikipedia.org/wiki/Циклограмма_пуска Вопрос 2: Внимание, в вопросе есть замены. По одной из версий СИНИЙ и КРАСНЫЙ - это братья. А по другой версии, это один персонаж, но в разных жизненных ситуациях. Тем не менее, в очередном новогоднем ролике известной компании СИНИЙ изображен победителем, а КРАСНЫЙ - проигравшим. Назовите эту компанию. Ответ: "Pepsi". Зачет: "Пепси". Комментарий: По сюжету ролика Дед Мороз в синей шубе, символизирующий "Пепси-колу", забивает гол в ворота Деда Мороза в красной шубе, символизирующего "Кока-колу". Дальше на экране появляется синяя банка пепси и слоган: "Живи здесь и сейчас". Синий - когда трезвый и холодно, а красный - когда пьяный и жарко. Источник: 1. Русская сказка "Два Мороза". 2. http://otvet.mail.ru/question/49584642 3. http://www.youtube.com/watch?v=QAdTBrkuyGo Вопрос 3: На одном из пиратских сайтов предлагались рейтинг произведений по скачиваемости и оценки этих же произведений пользователями. Для большинства произведений значения этих показателей совпадали. Однако одно произведение сильно выбивалось из этого ряда: оно было в первом десятке по оценкам пользователей и лишь в пятом десятке по скачиваемости. И это несмотря на обилие предложенных хостингов, звуковых форматов и даже возможность скачать составляющие произведение части по отдельности! Назовите автора этого произведения. Ответ: [Джон] Кейдж. Комментарий: Никто не хочет качать тишину. :-) Источник: 1. http://ru.wikipedia.org/wiki/4%E2%80%B233%E2%80%B3 2. ЛОАВ. Вопрос 4: Внимание, в вопросе есть замена. В восемнадцатом веке ГЛАДИОЛУС, попадавший в Россию сухопутным путем, назывался кяхтинским, а тот, что попадал морским путем, - кантонским. ГЛАДИОЛУСОМ на Волге называли неторопливый, длительный процесс. Какое слово мы заменили словом "ГЛАДИОЛУС", если даже дедушка Ленин посвятил ему целых два декрета? Ответ: Чай. Источник: 1. http://www.inpinto.com/articles/kofe_chay_kakao/kofe_v_rossii_simvol_roskoshi/ 2. http://www.teafruit.ru/index.php?route=information/information&information_id=6 Вопрос 5: На одной карикатуре кот, сидя на скамейке рядом с киногероем, держит в лапах сразу девять коробок. Назовите этого киногероя. Ответ: Форрест Гамп. Комментарий: Форрест Гамп сидит на скамейке на площади в городе Саванна, штат Джорджия. Людям, которые время от времени садятся к нему на скамейку, он рассказывает историю своей жизни. Обыгрывается фраза Форреста: "Жизнь - как коробка шоколадных конфет. Никогда не знаешь, какая начинка тебе попадется". У кота, как известно, девять жизней. :-) Источник: (pic: 20140029.jpg) Вопрос 6: В системе классификации народных сказок Аарне-Томпсона этот сюжет имеет номер 310. Известное произведение на этот сюжет впервые было переведено на русский язык Петром Полевым под названием "Колокольчик". Этот же сюжет лег в основу фильма 2010 года, оригинальное название которого - "Запутанная". Если исходить из научных данных, героине этого фильма было около 175 лет. Назовите имя этой героини. Ответ: Рапунцель. Комментарий: У представителей европеоидной расы, например, волосы растут в среднем примерно на 1 см в месяц, а длина волос Рапунцель в мультфильме "Рапунцель: Запутанная история" - около 21 метра. Источник: 1. http://ru.wikipedia.org/wiki/Рапунцель 2. http://ru.wikipedia.org/wiki/Рапунцель:_Запутанная_история 3. http://www.hair-ok.ru/2012/04/skorost-rosta-volos/ Вопрос 7: Внимание, в вопросе есть замена. О ЛЕВШЕ сегодня известно достаточно много. Так, рост ЛЕВШИ определяется состоянием одной его детали и может изменяться от 1 м 68 см до чуть более 1 м 80 см. Вес ЛЕВШИ - 238 килограммов. "Родом" он из Мексики, а монолог Гамлета ему категорически противопоказан. Какое слово мы заменили на "ЛЕВША"? Ответ: Бендер. Комментарий: Робот Бендер - левша. Код уничтожения Бендера - 2B, что созвучно "To Be" - "быть" (англ.) (серия "Where No Fan Has Gone Before"). Его рост зависит от того, выпущена антенна или нет. Источник: http://ru.wikipedia.org/wiki/Бендер_(Футурама) Вопрос 8: Внимание, в вопросе есть замена. В 2003 году луна астероида "(45) Eugenia", названного в честь императрицы Евгении, урожденной графини Монтихо, получила название Клаудио Маркизио. Как заметил один почитатель Клаудио Маркизио, "он обычно оставлял вопросы собеседников без ответов". Кого мы заменили на Клаудио Маркизио? Ответ: Маленький Принц. Комментарий: Название связано с погибшим в молодости принцем Наполеоном-Эженом (Наполеон IV), сыном императрицы Евгении, в честь которой назван сам астероид. Прозвище "Маленький принц" Клаудио Маркизо дал комментатор Клаудио Зулиани. Вторым прозвищем является "Туринский Де Росси". Источник: 1. http://ru.wikipedia.org/wiki/Маркизио,_Клаудио 2. http://ru.wikipedia.org/wiki/Маленький_принц 3. http://www.livelib.ru/book/1000254306 Вопрос 9: Этот памятник называют самой яркой антитезой Медному всаднику. Ходила легенда, по которой памятник предполагалось установить в Уральских горах, на границе Европы и Азии. Сразу после открытия памятник вызвал самые противоречивые отзывы: от восторженных до резко критических. По городу немедленно пошли гулять стихи, которые живы и сегодня: Стоит на площади комод, На комоде - бегемот, На бегемоте - обормот, На обормоте - шапка. Мы не просим назвать этот памятник. Воспроизведите фразу, которой охарактеризовал свое творение его создатель. Ответ: "Я изобразил одно животное на другом". Зачет: "Я не занимаюсь политикой. Я изобразил одно животное на другом". Комментарий: Памятник Александру III работы Паоло Трубецкого. Александр III - основатель Сибирского железнодорожного пути от Петербурга до Владивостока. Источник: http://www.peterburg.ru/sight/aleksandr-iii-obormot-na-begemote Вопрос 10: В 1992 году в Москве в рамках празднования Дня славянской письменности и культуры был установлен памятник Кириллу и Мефодию. Надпись на постаменте выполнена по-старославянски и гласит: "Святым равноапостольным первоучителям славянским Мефодию и Кириллу. Благодарная Россия". Любопытно, но позднее специалисты обнаружили на постаменте целых пять... Мы не спрашиваем пять чего. В ответе напишите то, где обнаружили сразу две. Ответ: Россия. Комментарий: Лингвисты, внимательно изучив надпись, обнаружили пять орфографических ошибок: в слове "апостол" и в имени Мефодий вместо омеги пишется "о"; в имени Кирилл вместо "i" должна стоять буква "и"; а в слове Россия сразу две ошибки - вместо "о" должна быть омега, вместо "и" должно писаться "i". Вот такой вышел курьез. Источник: http://www.allo495.ru/files-view-482.html Вопрос 11: В конце августа он пострадал от хулиганов, которые украли у него банку варенья и нанесли телесные повреждения. Поэтому с августа ОН на больничном, а со своими поклонниками ОН общается через социальные сети и обещал вернуться "домой" 2 февраля 2014 годя. Хотя, если судить по надписи, ОН - это не он, а ОНА. Назовите ЕГО тремя словами. Ответ: Ежик в тумане. Комментарий: Ежик сидит на пеньке с котомкой, лапки одеты в рукавички. Называется композиция почему-то "Лошадка". 2 февраля празднуют День Ежика (прототип Дня сурка в Древнем Риме). Ну и в конце мы поднапустили тумана. Источник: 1. http://kiev.segodnya.ua/kpeople/Kievskiy-Ezhik-v-tumane-do-fevralya-budet-na-bolnichnom-i-vernetsya-s-novoy-mordochkoy-461784.html 2. http://777-konstantin.blogspot.ru/2011/12/blog-post_06.html Вопрос 12: Внимание, в вопросе есть замены. Проект москвича Виталия Сабурова "Россия - ЧЕБУРАШКА. Страна ГЁМБЁЦ" победил на конкурсе арт-концепций "Национальная идея России". По мнению Сабурова, "в этом вечном движении ... есть залог нашей непобедимости". Мы не спрашиваем, какое слово мы заменили на "ЧЕБУРАШКА". Скажите, что мы заменили на "ГЁМБЁЦ". Ответ: Ванька-встанька. Незачет: Неваляшка. Комментарий: Гёмбёц (венг. gömböc) - пример трехмерного выпуклого тела с одной устойчивой и одной неустойчивой точкой равновесия. Чебурашка (чебурахнуться) - потомок неваляшки. Проект назывался "Россия - Неваляшка. Страна Ванька-встанька". Источник: 1. http://www.maxpark.com/community/129/content/1887363 2. http://ru.wikipedia.org/wiki/Гёмбёц 3. http://www.rusprom.biz/russkaya-derevyannaya-igrushka/30-nevalyashka-derevyannaya-igrushka Вопрос 13: В Средние века эта порода носила следующие названия: "Собака кузнецов и гадалок"; "Собака, побеждающая тьму". Назовите эту породу. Ответ: Далматинцы. Источник: http://rus-katana-dogs.narod.ru/poroda/pr176.html Вопрос 14: Блиц. Героиня романа Бориса Акунина "Левиафан" Рената Клебер утверждала, что мужчины больше похожи на семейство псовых, и подразделяла их на три основных типа: ПЕРВЫЕ, ВТОРЫЕ и ТРЕТЬИ. 1. ПЕРВЫЕ, как и их животные собратья, предпочитают легкую добычу, и в женщинах их привлекает доступность. Назовите ПЕРВЫХ одним словом. 2. ВТОРЫЕ обожают женщин слабых и беззащитных. Таких мужчин хлебом не корми - только дай тебя спасти и защитить. Назовите ВТОРЫХ одним словом. 3. ТРЕТЬИ - самый незамысловатый тип и начисто лишен воображения. Действует на них только нечто грубо-чувственное, вроде ненароком показанной лодыжки. Назовите ТРЕТЬИХ одним словом. Ответ: 1. Шакалы. 2. Овчарки. 3. Кобели. Источник: Б. Акунин. Левиафан. Вопрос 15: Внимание, цитата: "Я чувствовал себя тонущей собакой, - вспоминал он позднее, - в которую дети ради забавы кидают камни". Назовите автора этой цитаты. Ответ: Ганс Христиан Андерсен. Комментарий: Из этого страшного быта родилась не обида, а сказка - "Гадкий утенок". Источник: http://www.epwr.ru/quotauthor/519/ Вопрос 16: Первоначально при выборе места под первый в мире космодром склонялись к дальневосточному варианту. Но сразу несколько причин перевесили чашу весов в пользу Казахстана: климат, наличие путей сообщения, невысокая сейсмическая активность. Назовите еще одну причину устойчивым выражением, придуманным в прошлом веке Эриком Артуром Блэром. Ответ: "Холодная война". Комментарий: При запуске космических ракет они летели бы в сторону США, а это неизбежно приводило бы к постоянным обострениям отношений, ибо отличить запуск космической ракеты от боевой было просто невозможно. Источник: 1. http://www.vokrugsveta.ru/vs/article/2902/ 2. http://ru.wikipedia.org/wiki/Джордж_Оруэлл Вопрос 17: На большинстве современных карт Европы территория этой страна обозначается в большинстве случаях светло-синим цветом. В этой стране много женщин-миллиардеров и еще больше женщин-миллионеров. Но вам предстоит назвать не самую богатую, а самую знаменитую представительницу этой страны. Ответ: Русалочка. Комментарий: Это Дания. Обозначать Гренландию не ледяным цветом как-то неестественно. Источник: 1. http://www.paneuro.ru/pics/map/euro_map.jpg 2. http://www.lhv.ru/page.php?pageId=378 3. ЛОАВ. Вопрос 18: Утверждают, что его "помешательство" началось с того, как он приобрел спасательный круг. Он умер в 1860 году, оставив после себя двести тысяч долларов долгу, но друзья уже не считали его сумасшедшим. Назовите этого человека. Ответ: Чарльз Гудийр. Комментарий: Круг был резиновый, и Гудийр посвятил всю свою жизнь "лечению" резины. Источник: http://www.studzona.com/referats/view/39159 Вопрос 19: Педантичный, сухой, строгий, логичный и при этом очень добрый, ОН - пример для советских детей. По одной из версий, у НЕГО итальянские корни. Про НЕГО со всей уверенностью можно сказать - ОН сделал себя сам. Назовите ЕГО. Ответ: Самоделкин. Источник: http://ru.wikipedia.org/wiki/Самоделкин Вопрос 20: Власти Греции в условиях кризиса используют всё более изощренные способы охоты на уклоняющихся от уплаты налогов граждан - теперь представители налоговой полиции отслеживают из космоса незарегистрированные ИКСЫ. ИКС появился в Белом Доме при Рузвельте, а один из создателей Бестолкового словаря переводит это слово как "великий бас или Шаляпин". Какое слово мы заменили на ИКС? Ответ: Бассейн. Комментарий: Бассейн - великий бас - Шаляпин. Источник: http://www.aif.ru/society/news/53512 Вопрос 21: В 30-е годы XX века в Копенгагене на Ратушной площади было построено здание угловой башни, на которой установили две золотые фигуры. Они тогда предсказывали погоду: если ожидался дождь - на первый план "выезжала" девушка с зонтиком, если солнце - "выезжала" девушка с НИМ. Назовите ЕГО словом французского происхождения. Ответ: Велосипед. Комментарий: Дания - велосипедная страна. Копенгаген - велосипедная столица. Источник: 1. http://www.redigo.ru/geo/Europe/Denmark/Copenhagen_/routes/102 2. http://ru.wikipedia.org/wiki/Велосипед 3. http://www.denmark-all.ru/index.php/2011-10-23-11-46-54/668-2012-12-30-16-09-47 Вопрос 22: По преданию Купала, которого еще в раннем детстве от сестры Костромы унесла птица Сирин, поднял венок сестры, проплывая мимо на лодке, и по обычаю они должны были пожениться. Кострома не узнала родного брата. И только после свадьбы жених и невеста узнали, что они брат и сестра. Тогда они решили покончить с собой и утопились в Ра-реке. Кострома превратилась в русалку (мавку), а в кого или во что по этой легенде превратился Купала? Ответ: Цветок Купала-да-Мавка. Зачет: Иван-да-Марья. Источник: http://www.indrikgrad.ru/oter/library/2007/08/29/library_8.html Вопрос 23: (pic: 20140030.jpg) Перед вами масштабированное изображение экспонатов из музея Андерсена. Существует несколько версий, для чего великий сказочник брал с собой в спальню веревку. По первой версии, чтобы во время пожара иметь возможность вылезти в окно. Впрочем, эта версия не объясняет, почему веревку Андерсен клал всегда себе в постель. А чего, согласно другой версии, опасался писатель? Ответ: Уменьшиться. Комментарий: Андерсен опасался уменьшиться и брал с собой веревку, чтобы иметь возможность спуститься по ней на пол. Источник: http://www.tamada26.ru/raznoe/87-andersen.html Вопрос 24: (pic: 20140031.jpg) Прослушайте отзыв на книгу Джошуа Ферис: "Книга написана от лица "мы". Мы - это сотрудники рекламного агентства. На американском дворе свои лихие 90-е, то есть время кризиса. Кто следующий? Буду за вами. Вас здесь не стояло. За мной не занимать. Ну, очень всё знакомо. Воспоминания свежи". Напишите состоящее из четырех слов название этого произведения в русском переводе. Ответ: "И не осталось никого". Комментарий: У нас в этом туре тоже не осталось вопросов - перерыв! Источник: http://www.sakhalinart.com/blog/post/i-ne-ostalos-nikogo-dzhoshua-ferris Тур: 3 тур. "Памяти Пауля" (Москва) Редактор: Серафим Шибанов (Москва) Вопрос 1: <раздатка> "Riziko!" На раздаточном материале вы видите название турецкого аналога ЭТОГО. Назовите русский аналог ЭТОГО двумя словами. Ответ: "Своя игра". Комментарий: В Турции игра "Jeopardy!" (Рискуй!) выходит под названием "Riziko" [ризико]. В России аналогом "Jeopardy!" является "Своя игра". Мы начинаем свою игру. Источник: http://ru.wikipedia.org/wiki/Jeopardy! Автор: Серафим Шибанов (Москва) Вопрос 2: Пять концов ЕЕ, расположенной на флаге юго-восточной страны, иногда трактуют так: рабочие, крестьяне, солдаты, интеллигенция и молодежь. Другая ОНА была отличительным знаком. Кого? Ответ: Евреев. Комментарий: Речь идет о желтой пятиконечной звезде на флаге Вьетнама. Шестиконечная же желтая звезда являлась принудительным знаком отличия евреев в нацистской Германии. Источник: 1. http://ru.wikipedia.org/wiki/Флаг_Вьетнама 2. http://ru.wikipedia.org/wiki/Жёлтая_звезда Автор: Серафим Шибанов (Москва) Вопрос 3: [Ведущему: запастись маленькими шоколадками.] <раздатка> Черная шелковистая голова Хартума стояла посреди лужи крови. Хотите шоколадку? Ответьте на этот вопрос правильно - и получите ее вместе с плюсом. Назовите роман, предложение из которого вы получили. Ответ: "Крестный отец". Комментарий: Хартум - лошадь. Что до плюса, то от такого предложения, как нам кажется, невозможно отказаться. Источник: http://lib.ru/PJUSO/crestnyj.txt Автор: Серафим Шибанов (Москва) Вопрос 4: В Советском энциклопедическом словаре 1982 года издания вслед за статьей о Социалистической Единой Партии Германии шла статья о НЕЙ. Назовите ЕЕ пятью словами. Ответ: Социалистическая Единая Партия Западного Берлина. Комментарий: В те годы Западный Берлин был отдельным политическим субъектом. В свое время от Социалистической Единой Партии Германии откололась Социалистическая Единая Партия Западного Берлина, при этом обе использовали в названии слово "единая". Источник: Советский энциклопедический словарь. - М., 1982. Автор: Серафим Шибанов (Москва) Вопрос 5: В произведении начала семнадцатого века среди действующих героев есть соломенные еноты. С "Соломенными енотами" в свое время сотрудничал Максим Юрак, более известный как Макс Йорик. Какие слова мы заменили "Соломенными енотами"? Ответ: Король и Шут. Комментарий: Название одной панк-группы мы заменили другим. Король и Шут - герои "Короля Лира". Забавно, что с группой "Король и Шут" работал человек по прозвищу Йорик. Сам Йорик - это другой шут из Шекспира. Источник: 1. http://lib.ru/SHAKESPEARE/shks_text4.txt 2. http://ru.wikipedia.org/wiki/Король_Лир 3. http://ru.wikipedia.org/wiki/Король_и_Шут 4. http://ru.wikipedia.org/wiki/Макс_Йорик Автор: Серафим Шибанов (Москва) Вопрос 6: Критикуя ЕГО, генеральный менеджер хоккейного "Автомобилиста" Леонид Вайсфельд заметил, что на "Жигулях" никто не ездит, все предпочитают "Мерседесы". Назовите ЕГО тремя словами. Ответ: Лимит на легионеров. Зачет: По смыслу с соблюдением формы вопроса. Комментарий: Вайсфельд сказал: "Действительно, ездим все на иномарках, смотрим иностранные телевизоры... Ну так вы включите "Рекорд" и смотрите или ездите на "Жигулях" - так нет, что-то все на "Мерседесах" ездят...". Источник: http://news.sportbox.ru/Vidy_sporta/Hokkej/KHL/spbnews_NI385522_Leonid-Vaysfelid-Davayte-kloniruem-Malkina Автор: Серафим Шибанов (Москва) Вопрос 7: Луи Фердинанд Селин в романе "Путешествие на край ночи" говорит, что люди придают деньгам слишком большое значение, практически возводя их в религию. Кассу в банке писатель сравнивает с НЕЙ. Назовите ЕЕ. Ответ: Исповедальня. Комментарий: "Не думайте, что долларопоклонники, входя в банк, могут им пользоваться, как вздумается. Ничего подобного. Они говорят с Долларом вполголоса, нашептывая ему что-то через небольшую решетку, ну прямо-таки исповедуются". Источник: Л.Ф. Селин. Путешествие на край ночи. http://lib.ru/INPROZ/SELIN/kraj_nochi.txt Автор: Евгений Коватенков (Москва) Вопрос 8: <раздатка> И вдруг мне стало непереносимо мерзко, перо выпало из моих рук, разбрызгивая чернила. Что случилось? Опять <...> Нет, это была не она, комната хранила свой обычный приторно-дружелюбный вид. Мы не спрашиваем, что здесь стоит на месте отточия. Назовите произведение, фрагмент которого вам раздали. Ответ: "Тошнота". Источник: http://lib.rus.ec/b/145395/read#t2 Автор: Серафим Шибанов (Москва) Вопрос 9: (pic: 20140032.jpg) В начале фильма 2004 года герой везет на кухню гостиницы омаров, однако из-за жары доставить в сохранности получилось только половину груза. Назовите этот фильм. Ответ: "Отель "Руанда"". Комментарий: Главный герой - управляющий отеля в Кигали, столице Руанды. В свое время на флаге Руанды на желтой вертикальной полосе была изображена заглавная латинская R. На это мы вам намекали раздаточным материалом. Источник: 1. Х/ф "Отель "Руанда". http://www.kinobox.org/cat5/6019-otel-ruanda.html 2. http://www.imdb.com/title/tt0395169/trivia?item=tr0955960 Автор: Серафим Шибанов (Москва) Вопрос 10: [Ведущему: выделить голосом слова "эта страна".] В латышском языке эта страна именуется Кривейя. Назовите эту страну. Ответ: Россия. Комментарий: Возможно, название связано с племенами кривичей, которые давным-давно жили на территории России. Источник: http://ltg.wikipedia.org/wiki/Kr%C4%ABveja Автор: Серафим Шибанов (Москва) Вопрос 11: На картине Бертоломеуса Спрангера ОНА держит на плече увесистую дубину, в то время как сидящему рядом герою досталось веретено. Назовите ЕЕ имя. Ответ: Омфала. Комментарий: Герой - это не просто герой картины, а именно что Герой. Картина называется "Геракл и Омфала". Будучи в плену у Омфалы, Геракл претерпел много унижений, в том числе его заставляли прясть. Источник: http://www.liveinternet.ru/users/4000579/post146104586/ Автор: Серафим Шибанов (Москва) Вопрос 12: Мать Майкла Ханта умерла от рака легких через год после рождения сына. Хочется верить, что данной информации вам хватит, чтобы назвать прозвище Майкла Ханта, под которым он более известен. Ответ: Курильщик. Зачет: Cigarette Smoking Man, C.S.Man, Cancer-Man. Комментарий: Отрицательный персонаж сериала "Секретные материалы", чье имя большую часть времени неизвестно, но которого легко идентифицировать по сигарете. Фраза "Я хочу верить" - один из тэглайнов сериала. Источник: http://ru.wikipedia.org/wiki/Курильщик_(Секретные_материалы) Автор: Серафим Шибанов (Москва) Вопрос 13: По легенде, прадед Марии, умирая, произнес слова: "Ветер уносит запах АЛЬФЫ...", что и обусловило последующий выбор Марии. Что мы заменили АЛЬФОЙ? Ответ: Мирра. Комментарий: Речь идет о поэтессе Марии Лохвицкой, более известной под именем Мирра. Источник: http://ru.wikipedia.org/wiki/Лохвицкая,_Мирра_Александровна Автор: Серафим Шибанов (Москва) Вопрос 14: Автор вопроса заметил, что судьба Икара заключается в том, чтобы СДЕЛАТЬ ЭТО. Ответьте шестью словами из источника 1988 года, что такое "СДЕЛАТЬ ЭТО". Ответ: Упасть опаленным звездой по имени Солнце. Комментарий: Как известно, Икар слишком близко подлетел к Солнцу, воск расплавился, и Икар упал. Источник 1988 года - песня "Звезда по имени Солнце". Написана она была в 1988 году во время съемок фильма "Игла". Источник: 1. http://ru.wikipedia.org/wiki/Звезда_по_имени_Солнце_(песня) 2. http://ru.wikipedia.org/wiki/Игла_(фильм) Автор: Серафим Шибанов (Москва) Вопрос 15: ОНО есть у персонажей серии компьютерных игр "Mortal Kombat" [Мортал Комбат] Горо, Кинтаро и Шивы. В песне из кинофильма 1962 года ОНО стало определением. Для кого? Ответ: Для друга. Комментарий: ОНО - это третье плечо. У Горо, Кинтаро и Шивы по четыре руки и два плечевых пояса. В песне к кинофильму "Путь к причалу" есть слова: "Друг мой - третье мое плечо". Источник: 1. http://ru.mortalkombat.wikia.com/wiki/Шоканы 2. http://lib.ru/KSP/petrow.txt 3. http://ru.wikipedia.org/wiki/Путь_к_причалу Автор: Серафим Шибанов (Москва) Вопрос 16: Две первые очереди детища Николая Александровича открылись после его гибели - в 1894 и 1896. А убит он был душевнобольным. Напишите фамилию Николая Александровича. Ответ: Алексеев. Комментарий: Московская психиатрическая больница N 1 носит имя Алексеева (ранее Кащенко). Источник: http://ru.wikipedia.org/wiki/Алексеев,_Николай_Александрович_(предприниматель) Автор: Серафим Шибанов (Москва) Вопрос 17: Дуплет. 1. На постере к этому фильму 1976 года мальчик отбрасывает тень в виде волка. Назовите этот фильм. 2. Заглавным героем фильма 2003 года является клоун. Назовите его имя. Ответ: 1. "Омен". 2. Немо. Комментарий: Мальчика в фильме "Омен" охранял волк, а главным героем фильма "В поисках Немо" является рыба-клоун. Если прочитать слово "Омен" наоборот, то получится "Немо". Источник: 1. http://ru.wikipedia.org/wiki/Омен_(фильм,_1976) 2. http://ru.wikipedia.org/wiki/В_поисках_Немо Автор: Серафим Шибанов (Москва) Вопрос 18: В фильмах "Дилижанс" и "Винчестер-73" ОНА является мистическим указанием скорой гибели игрока. По народным поверьям, ОНА помогает избавиться от опухолей и бородавок. Назовите ЕЕ двумя существительными. Ответ: Рука мертвеца. Комментарий: "Рука мертвеца" - это обозначение покерной комбинации из тузов и восьмерок черной масти. Именно эти карты держал в руках "гроза Дикого Запада" Билл Хикок, когда был застрелен в 1876 году. Чтобы избавиться от бородавки, надо провести по ней рукой мертвеца. Источник: 1. http://en.wikipedia.org/wiki/Dead_man's_hand_in_popular_culture 2. http://www.bibliotekar.ru/430/24.htm Автор: Серафим Шибанов (Москва) Вопрос 19: Объясняя свою аллегорию, философ утверждал, что ОНА - это чувственный мир, в котором живут люди. Одна из НИХ, известная своими рисунками, находится в 30 км от Сантандера и называется... Как именно? Ответ: Альтамира. Комментарий: ОНА - это пещера. Так Платон объяснял миф о пещере. Пещера Альтамира известна наскальной живописью и находится в Испании. Источник: 1. http://ru.wikipedia.org/wiki/Миф_о_пещере 2. http://ru.wikipedia.org/wiki/Альтамира_(пещера) Автор: Серафим Шибанов (Москва) Вопрос 20: Иногда для этого явления используется французский термин "прескевю", то есть "почти увиденное". В его русском описании фигурирует часть тела. В качестве примера данного феномена Википедия приводит произведение, созданное в середине 80-х годов XIX века. Назовите это произведение. Ответ: "Лошадиная фамилия". Комментарий: Ситуация, описываемая в русском языке словами "вертится на кончике языка", приведена в рассказе А.П. Чехова, когда герой помнит, что фамилия связана с лошадьми, но саму ее вспомнить не может. Источник: 1. http://www.geshtaltpsy.ru/fenomeny_psihiki.php 2. http://ru.wikipedia.org/wiki/На_кончике_языка_(состояние) 3. http://ru.wikipedia.org/wiki/Лошадиная_фамилия Автор: Серафим Шибанов (Москва) Вопрос 21: Во время чисток и массовой депортации населения в Кампучии только ИМ удалось сохранить свои рабочие места. ОНИ - прозвище игроков клуба, в 2009 году переехавшего из Ростова в Краснодар. Назовите ИХ. Ответ: Железнодорожники. Комментарий: Переселение населения производилось как раз при участии железнодорожников. Клуб "Локомотив-Кубань" ранее представлял Ростов, а еще ранее - Минеральные Воды. Источник: 1. http://www.e-reading.link/chapter.php/1013349/140/Chernaya_kniga_kommunizma._Prestupleniya%2C_terror%2C_repressii.html 2. http://ru.wikipedia.org/wiki/Локомотив-Кубань Автор: Серафим Шибанов (Москва) Вопрос 22: Егор Летов в одной из песен вывел в ЕЕ образе Яну Дягилеву. Назовите ЕЕ имя. Ответ: Офелия. Комментарий: Яна Дягилева тоже утонула. У "Гражданской Обороны" есть песня "Офелия". Источник: http://ru.wikipedia.org/wiki/Офелия Автор: Серафим Шибанов (Москва), по идее Александра Рождествина (Самара) Вопрос 23: В 1966 году в Чаде одновременно и независимо друг от друга восстали оседлые жители востока страны и кочевники севера. Эти события получили название "ТАКОЙ мятеж". Назовите династию, которая с тринадцатого века стала использовать ТАКОЕ существо в качестве своего символа. Ответ: Палеологи. Комментарий: События получили название "Двуглавый мятеж". В Россию двуглавый орел перекочевал благодаря Софье Палеолог, жене Ивана III. Источник: 1. http://www.conflictologist.org/main/cwchad.htm 2. http://en.wikipedia.org/wiki/Byzantine_flags_and_insignia Автор: Серафим Шибанов (Москва) Вопрос 24: <раздатка> Месяц в городе ночью не виден, Как над сквером - отдельный плафон. Ах, зачем так безмолвен Никитин, Словно морем огней напоён. Автоматы у входа в аллею Тоже будто бы странники тут, Как слепцы - ни о чем не жалеют, И даяния медного ждут. Нам-то кажется полночь пустою, Грубоватым - неоновый свет. Он хоть движется, зренье - в застое: Вот чего опасайся, поэт! Ну а чтоб глубоко, не парадно Оценить, как мы нынче живем, Встать на место Никитина надо - И не в сквере, а в веке былом... В статье Википедии, где приведено это стихотворение, отмечено, что в последней строке вместо одной буквы должна стоять другая. Напишите название упомянутой статьи, состоящее из одного слова. Ответ: Акростих. Комментарий: Вместо И должно быть Ы, но с Ы начать строчку очень трудно. Источник: http://ru.wikipedia.org/wiki/Акростих Автор: Серафим Шибанов (Москва) Тур: 4 тур. "IDDQD - UNRIP" (Москва) Вопрос 1: Как-то раз в юмористической передаче "Yesterday Live" монолог ведущего о проблемах современного театра завершился восклицанием: "О, верный зритель! Жаль, что ты [ПРОПУСК]!". Заполните пропуск одним символом. Ответ: 1. Источник: Передача "Yesterday Live". Автор: Евгений Кравченко (Москва) Вопрос 2: В мультипликационном фильме "Питер Пэн" один из героев фехтует, используя АЛЬФУ. С какого слова начинается известный афоризм, заканчивающийся АЛЬФОЙ? Ответ: Театр. Комментарий: Герои мультфильма разыгрывают сцену поединка Питера Пэна с Капитаном Крюком, при этом Джон изображает крюк при помощи вешалки. Источник: 1. http://ru.wikipedia.org/wiki/Питер_Пэн_(мультфильм) 2. http://video.yandex.ru/users/yelena-simko/view/491/ (03:06) 3. http://ru.wikipedia.org/wiki/Станиславский,_Константин_Сергеевич 4. http://dic.academic.ru/dic.nsf/dic_wingwords/2678/ Автор: Виктор Бойков (Раменское) Вопрос 3: По меткому замечанию знакомого автора вопроса, с течением времени бараны за баранкой сменили [ПЕРВЫЙ ПРОПУСК] на [ВТОРОЙ ПРОПУСК]. Восстановите оба пропуска в абсолютно точных словоформах и в правильном порядке. Ответ: КозлОв, кОзлах. Источник: ЛОАВ. Автор: Антон Тилипман (Москва) Вопрос 4: (pic: 20140033.jpg) Увидев эмблему Московского городского штаба молодежно-студенческих отрядов, автор вопроса предположил, что члены штаба сыты и не испытывают проблем в половой жизни, чувствуют себя защищенными и избавились от страха и неудач. АЛЬФА этих членов - третья или четвертая, а на эмблеме не хватает трех букв. Мы не просим назвать АЛЬФУ. Напишите три буквы, которых, по мнению автора, не хватает на эмблеме. Ответ: а, л, у. Комментарий: Изображенная на эмблеме пирамида напомнила автору вопроса пирамиду Маслоу, на середину которой указывает стрелка. Словом "АЛЬФА" в вопросе заменено слово "ступень". Если из фамилии Маслоу изъять буквы "а", "л" и "у", то получится присутствующая на эмблеме аббревиатура МСО. Источник: 1. http://www.dsizmaylovo.ru/stud_img/news/mso_vao/image2.jpeg 2. http://ru.wikipedia.org/wiki/Пирамида_потребностей_по_Маслоу Автор: Виктор Бойков (Раменское) Вопрос 5: В дни дерби обстановка в Севилье не похожа на праздник. Журналист Дмитрий Навоша отмечает, что даже ОНА в окрестностях арены "БЕтиса" становится отчасти зеленой. Назовите ее словом с дефисом. Ответ: Кока-кола. Комментарий: Цвета футбольного клуба "Севилья" - красно-белые, а "Бетиса" - бело-зеленые. Рекламщики побоялись размещать рядом со стадионом "Бетиса" биллборд в цветах команды-противника и заменили красный цвет на зеленый. Слово "праздник" в вопросе намекает на известный рекламный слоган Coca-Cola "Праздник к нам приходит". Источник: http://www.terrikon.com/posts/3925 Автор: Виктор Бойков (Раменское) Вопрос 6: (pic: 20140034.jpg) На розданном вам гербе - архангел, поражающий черномазого дьявола. Остроумный автор вопроса полагает, что это кара за допущенное в 1821 году кощунство. О каком конкретно кощунстве идет речь? Ответ: Написание "Гавриилиады". Зачет: По слову "Гавриилиада". Комментарий: На гербе Архангельска изображен Архангел Гавриил, а дьявол на том же гербе поразительно напоминает А.С. Пушкина, прадедушка которого, как известно, был родом с Черного континента. Герб может быть истолкован как символическое наказание Пушкина за пародийную поэму "Гаврилиада", главным героем которой как раз и является Архангел Гавриил. Источник: 1. http://ru.wikipedia.org/wiki/Архангельск 2. http://ru.wikipedia.org/wiki/Гавриилиада Автор: Лидия Антоненко (Кратово) Вопрос 7: В начале карьеры ОН подрабатывал в ночном клубе для того, чтобы оплатить свое обучение на стоматолога. В кавер-версии одной из ЕГО песен, записанной российской музыкальной группой, звучат слова: "Чтоб я так жил!". Назовите ЕГО. Ответ: Dr. Alban. Зачет: Доктор Албан, Doctor Alban, Албан Нвапа, Alban Uzoma Nwapa. Комментарий: Известный шведско-нигерийский певец и музыкант Албан Нвапа в начале карьеры взял себе сценический псевдоним Dr. Alban, где приставка "Dr." (сокращение от слова "doctor") является данью его медицинскому образованию. Фраза "Чтоб я так жил!" является отсылкой к названию самой известной композиции Dr.Alban'а "It's my life". Источник: 1. http://www.youtube.com/watch?v=vLp2_ouK6lo 2. http://ru.wikipedia.org/wiki/Доктор_Албан Автор: Виктор Бойков (Раменское) Вопрос 8: ИКС обнаружен у пяти отрядов млекопитающих - насекомоядных, рукокрылых, грызунов, хищных и большинства приматов. При этом у человека ИКС отсутствует, в связи с чем бытует мнение, что Ева была создана не из ребра Адама. Мы не спрашиваем, что такое ИКС. Ответьте, какое животное является абсолютным рекордсменом по длине ИКСА. Ответ: Морж. Комментарий: ИКС - это бакулюм, кость в соединительной ткани полового члена. Источник: 1. http://ru.wikipedia.org/wiki/Бакулюм 2. http://ru.wikipedia.org/wiki/Морж 3. У. Мастерс, В. Джонсон, Р. Колодни. Основы сексологии. Автор: Антон Тилипман (Москва) Вопрос 9: Целевая аудитория основанной в 1999 году кондитерской фабрики - дети и подростки. Свою продукцию она позиционирует как "ежедневные маленькие радости". Напишите название этой фабрики. Ответ: "Меньшевик". Комментарий: Скорее всего, основатели предприятия назвали его по аналогии с известной российской кондитерской фабрикой "Большевик" (кстати, она была основана в 1855 году под названием "Адольф Сиу и Компания"). Источник: http://www.menshevik.ru/ Автор: Виктор Бойков (Раменское) Вопрос 10: Во избежание путаниц при радиопомехах авиадиспетчеры пользуются международным фонетическим алфавитом, при использовании которого иногда приходится отходить от стандартов. Так, сотрудники аэропорта "Атланта - Хартсфилд-Джексон", где находится хаб крупной авиакомпании, вместо ПЕРВОЙ используют ВТОРОГО. Символ большой ПЕРВОЙ входит в символ, связанный со ВТОРЫМ. Назовите ПЕРВУЮ и ВТОРОГО. Ответ: Delta, David. Комментарий: Чтобы не путать с позывными авиакомпании "Delta Airlines". Аэропорт Атланты - крупнейший хаб этой авиакомпании, на ее долю приходится до 60% рейсов, обслуживаемых этим аэропортом. Заглавная греческая дельта обозначается треугольником. Источник: http://en.wikipedia.org/wiki/NATO_phonetic_alphabet Автор: Евгений Кравченко (Москва), в редакции Гая Кузнецова (Москва) Вопрос 11: (pic: 20140035.jpg) Статья под закрытым нами заголовком посвящена девушке с обложки, а сам заголовок призывает забыть обо всём на свете и восхититься красотой этой девушки, которая может сделать вас рассеянным или невнимательным. Воспроизведите этот призыв. Ответ: Раскройте варежку. Зачет: По смыслу с упоминанием слова "варежка". Комментарий: Согласно "Большому словарю русских поговорок", одно из значений выражения "раскрыть варежку" - "становиться невнимательным, рассеянным". А изображенная на обложке телеведущая Алла Михеева в буквальном смысле "раскрывает варежки" в фотосессии, попавшей в данный выпуск журнала "MAXIM". Источник: 1. http://www.pdf-magazine.net/men/79-maxim-32013.html 2. http://dic.academic.ru/dic.nsf/proverbs/13753/ Автор: Виктор Бойков (Раменское) Вопрос 12: Заголовок статья об изоляции режима Муаммара Каддафи назывался так же, как известное произведение современного классика. Напишите это название. Ответ: "Полковнику никто не пишет". Источник: 1. http://www.islamrf.ru/news/world/w-opinions/17530/ 2. http://ru.wikipedia.org/wiki/Полковнику_никто_не_пишет Автор: Евгений Кравченко (Москва) Вопрос 13: Герой Алексея Панина, рекламируя комедийный сериал "Однажды в милиции", предлагает расшифровку: "ОНИ". ОНО упоминается в уникальном стихотворении. Кто автор этого стихотворения? Ответ: [Афанасий] Фет. Комментарий: Герой произносит фразу: "Знаете, почему нас называют "милиция"? Потому что у нас милые лица". В стихотворении без глаголов А.А. Фета "Шепот, робкое дыханье..." есть строчка "ряд волшебных изменений милого лица". Источник: 1. http://www.youtube.com/watch?v=4e61Fm4nrc4 2. http://az.lib.ru/f/fet_a_a/text_0042.shtml Автор: Виктор Бойков (Раменское) Вопрос 14: В песне современной панк-группы, вышедшей на альбоме "2084" [две тысячи восемьдесят четвертый], герои собираются курить. Ответьте максимально точно, что именно. Ответ: [Сигареты] "Winston". Комментарий: Название альбома "2084" - аллюзия на произведения Джорджа Оруэлла "1984", главным героем которого является Уинстон Смит. Источник: 1. http://ru.wikipedia.org/wiki/1984_(роман) 2. http://megalyrics.ru/lyric/ptvp/vinston.htm Автор: Виктор Бойков (Раменское) Вопрос 15: Герой песни "БАльник" мечтает порвать платье танцующей с ним девушки особым способом. В описании этого способа рифмуются два слова, не различимые на слух. Напишите эти слова. Ответ: Ногой, нагой. Зачет: В любом порядке. Комментарий: Группа "Бобры" в своей песне "Бальник" поет: "... И платье на тебе порвать, наступив ногой, Хочу увидеть нагой (хочу увидеть нагой), Тебя увидеть нагой (тебя увидеть нагой), Хочу тебя нагой...". Источник: 1. http://bobry.shurf.ru/music/album-1257 2. http://www.dodiez.ru/music/Бобры/Бальник/ Автор: Виктор Бойков (Раменское) Вопрос 16: 8 августа в эскимосской деревушке Коцебу можно полюбоваться парой ИКСОВ, хотя еще за месяц до того целый день можно прождать впустую. Ответьте аббревиатурой, где встречают по полтора десятка каждый день. Ответ: На МКС. Комментарий: Ежегодно 8 августа жители города Коцебу, штат Аляска, встречают два заката в течение одного дня - один вскоре после полуночи, другой - незадолго до полуночи в конце дня. А в июне-июле в Коцебу полярный день. Источник: 1. http://www.pinnacle-travel.org/alaska/Kotzebue.htm 2. http://www.factroom.ru/facts/5868 Автор: Евгений Кравченко (Москва) Вопрос 17: <раздатка> Не с Корнеем Чуковским [ПРОПУСК] ли Я решил испытать нынче дактили? Если б мы здесь бутылку раскокали, Я писал и писал бы в Чукоккале. Прослушайте часть стихотворения известного лингвиста Романа Якобсона, опубликованного в рукописном альманахе "Чукоккала" в 1919 году: Не с Корнеем Чуковским [ПРОПУСК] ли Я решил испытать нынче дактили? Если б мы здесь бутылку раскокали, Я писал и писал бы в Чукоккале. В наши дни ПРОПУСК нередко пишут слитно. Восстановите пропуск. Ответ: В, контакте. Комментарий: "Чукоккала" - своего рода аналог современного сообщества в социальной сети "Вконтакте" с администратором и возможностью "подписчиков" писать на "стене". Источник: http://www.bungalos.ru/b/gendlin_perebiraya_starye_bloknoty/78 Автор: Антон Тилипман (Москва) Вопрос 18: Одна из сложных проблем лингвистики - определение границы между ИКСОМ и ИГРЕКОМ. "Политическое" определение этой разницы заключено в афоризме, приписываемом Максу Вайнрайху: "ИКС - это ИГРЕК, у которого есть армия и флот". Назовите ИГРЕК. Ответ: Диалект. Комментарий: А ИКС - соответственно, язык. В данном выражении указывается на то, что политический фактор является одним из важнейших факторов восприятия того или иного говора как отдельного языка или всего лишь диалекта. Источник: 1. http://ru.wikipedia.org/wiki/Проблема_%C2%ABязык_или_диалект%C2%BB 2. http://ru.wikipedia.org/wiki/Язык_%E2%80%94_это_диалект,_у_которого_есть_армия_и_флот Автор: Евгений Кравченко (Москва) Вопрос 19: Заголовок статьи в журнале "Эксперт" о необходимости создания стимулов для долгосрочного кредитования упоминает ЕГО. Согласно шутливому афоризму, ОН - женская сексуальная фантазия. Назовите ЕГО двумя словами. Ответ: Длинный рубль. Источник: 1. http://expert.ru/expert/2010/39/v_pogone_za_rublem/ 2. http://www.anekdot.ru/id/373262/ Автор: Евгений Кравченко (Москва) Вопрос 20: В 2010 году народный артист России представил публике концертную программу под названием: "[ПЕРВЫЙ ПРОПУСК] много, [ВТОРОЙ ПРОПУСК] один!". Заполните любой из пропусков. Ответ: Певцов. Комментарий: Речь шла об актере театра и кино, певце Дмитрии Певцове. Источник: http://ru.wikipedia.org/wiki/Певцов,_Дмитрий_Анатольевич Автор: Виктор Бойков (Раменское) Вопрос 21: По словам британского писателя, название для цикла своих будущих романов он придумал еще в молодости, лежа однажды пьяным на инсбрукском поле в ходе путешествия по Европе. Напишите это название. Ответ: "Автостопом по галактике". Зачет: "The Hitchhiker's Guide to the Galaxy", "Путеводитель автостопщика по галактике", "Путеводитель для путешествующих по галактике автостопом" и т.п. Комментарий: Путешествовал автостопом, лежал на спине и смотрел на звезды. Источник: http://ru.wikipedia.org/wiki/Автостопом_по_галактике_(серия_романов) Автор: Евгений Кравченко (Москва) Вопрос 22: Рассказывают, что в качестве одного из аргументов ОН однажды захватил с собой африканские фиги. Назовите ЕГО двумя, тремя или четырьмя словами. Ответ: Катон Старший. Зачет: Марк Порций Катон; Марк Порций Катон Старший. Источник: 1. http://lib.ru/POEEAST/PLUTARH/plutarkh4_9.txt 2. http://www.azbyka.ru/otechnik/?Tertullian/k_yazichnikam=2_16 3. http://www.razlib.ru/istorija/kapitoliiskaja_volchica_rim_do_cezarei/p33.php 4. http://ru.wikipedia.org/wiki/Марк_Порций_Катон_Старший Автор: Виктор Бойков (Раменское) Вопрос 23: Соберитесь, тряпки! В изолированном тропическом районе на склонах горы Фоджа в Новой Гвинее обнаружено множество неизвестных ранее видов фауны. Как утверждает Википедия, животные не боятся человека, в частности, длинноклювая проехидна позволила ученым СДЕЛАТЬ ЭТО. Какие четыре слова мы заменили на "СДЕЛАТЬ ЭТО"? Ответ: Взять себя в руки. Источник: http://ru.wikipedia.org/wiki/Новая_Гвинея Автор: Евгений Кравченко (Москва) Вопрос 24: Жалуясь на невозможность дозвониться в службу техподдержки, один клиент упомянул "даже не ПЕРВУЮ, а ВТОРОЙ". Владимир Винокур, который иногда заполнял ПЕРВУЮ, инициативу по введению в армии ВТОРОГО назвал "детским садом". Назовите и ПЕРВУЮ, и ВТОРОЙ. Ответ: Музыкальная пауза, тихий час. Источник: 1. http://forum.netbynet.ru/index.php?showtopic=121322&st=1060 2. http://www.mk.ru/politics/article/2009/12/18/402174-tihiy-chas-probil.html Автор: Евгений Кравченко (Москва) Тур: 5 тур. "Любовь в середине зимы" (Москва - Санкт-Петербург) Редактор: Владислав Король Инфо: Редактор благодарит за помощь в работе над пакетом Наталию Новыш, Андрея Одегова, Сергея Терентьева, Владимира Цвингли, Серафима Шибанова, Андрея Штефана и Елизавету Штефан. Вопрос 1: (pic: 20140036.jpg) Назовите одним словом то, что должно быть на месте знака вопроса. Ответ: Вазелин. Комментарий: (pic: 20140037.jpg) Слева Лакан, справа канкан; слева Фасбиндер, справа цилиндр; слева Селин, справа вазелин. Всё согласно песне Псоя Короленко "Припев два раза". Источник: 1. http://www.psy-fund.kz/jak_lakan.page 2. http://www.kinopoisk.ru/name/163778/ 3. http://briefly.ru/selin/ 4. http://webkind.ru/text/0832842_99880062p12496581_text_pesni_pripev-dva-raza.html Автор: Серафим Шибанов Вопрос 2: Внимание, в вопросе есть замена. В 1993 году "МОНГОЛИЯ" проиграла "Индокитаю". Что заменено словом "МОНГОЛИЯ"? Ответ: "Урга - территория любви". Зачет: "Урга". Комментарий: Премию "Оскар" за лучший фильм на иностранном языке в 1993 году получил французско-вьетнамский фильм "Индокитай", а советско-французский фильм "Урга - территория любви" был лишь номинантом. Действие фильма происходит в китайском автономном районе Внутренняя Монголия. Источник: http://ru.wikipedia.org/wiki/Премия_%C2%ABОскар%C2%BB_за_лучший_фильм_на_иностранном_языке Автор: Владислав Король Вопрос 3: По словам Дмитрия Галковского, век назад Россия стремительно летела в преисподнюю. Что, по его мнению, было характерно для тогдашнего политического спектра, если весьма умеренная партия октябристов квалифицировалась как "крайне правая"? Ответ: Красное смещение. Комментарий: "Русская галактика летела в преисподнюю почти со скоростью света. Это видно по красному смещению в ее политическом спектре. Партии в России делились на ультракрасные, махрово красные, багрово-красные, просто красные, умеренно красные и розовые. Последние (октябристы и националисты) квалифицировались... как "реакционные" и "крайне правые". Наконец, действительно правые партии вообще не рассматривались, считались стоящими за рамками приличного общества". Источник: Дмитрий Галковский. Бесконечный тупик (примечание к N 8). http://www.samisdat.com/3/311-014.htm Автор: Владислав Король Вопрос 4: График дежурств по студии, составленный одним из героев программы "Спокойной ночи, малыши!", часто нарушался. Как назвала этот график ведущая программы? Ответ: Филькина грамота. Комментарий: График составил пес Филя. Источник: Программа "Спокойной ночи, малыши!", март 2013 г. Автор: Владислав Король Вопрос 5: Во время процесса над участницами группы "Pussy Riot" Елена Ямпольская заметила, что в России PR [пиАр] легко превращается... Во что? Ответ: ЗК. Комментарий: В русской раскладке клавиатуры аббревиатура PR превращается в ЗК. Источник: http://www.portal-kultura.ru/articles/kolonka-glavnogo-redaktora/neprostaya-istoriya Автор: Владислав Король, в редакции Вадима Молдавского Вопрос 6: Назовите фамилию владельца петербургского агентства недвижимости "М16". Ответ: Малафеев. Комментарий: Бизнесмен Вячеслав Малафеев также известен как вратарь "Зенита" и сборной России, играющий под распространенным у вратарей номером 16. А оружие тут ни при чем. Источник: http://www.m16.bz/about Автор: Владислав Король Вопрос 7: При проектировании советской атомной бомбы вычислители на трофейных счетных машинах конвейерным методом просчитывали модели взрыва. Сообщив полученный результат по цепочке, вычислитель должен был поступить с ним ТАК. С кем следовало ТАК поступить по приговору суда? Ответ: Герострат. Комментарий: По инструкции, после передачи по цепочке промежуточного результата вычислений его следовало забыть. За сожжение храма суд приговорил Герострата к смерти и, главное, к забвению. Источник: 1. "Компьютерра", 2009, N 43. 2. http://dic.academic.ru/dic.nsf/dic_wingwords/2508/ Автор: Владислав Король Вопрос 8: В пьесе Ибсена "Гедда Габлер" заглавная героиня своими словами намеренно разрушает наладившуюся жизнь одного из героев и его доверие к подруге. Десмонд Маккарти уподобил Гедду Габлер "Яду, отравляющему разум". Какие две буквы заменены в предыдущем предложении? Ответ: Го. Комментарий: Критик сопоставил отношения Гедды и Лёвборга с отношениями Яго и Отелло. Источник: А. Образцова. Стелла Патрик Кэмпбелл. - М.: Искусство, 1973. - C. 107. Автор: Наталия Новыш Вопрос 9: Набирая SMS, одна девушка пропустила вторую букву в прилагательном, характеризующем ее новый цвет волос. Система Т9 предложила ей другое прилагательное. Какое именно? Ответ: "Умный". Комментарий: Девушка перекрасилась из блондинки в брюнетку. Система интеллектуального ввода текста настолько интеллектуальна, что умеет даже шутить (темный - тмный - умный). Источник: http://re-gata.livejournal.com/220474.html Автор: Владислав Король, в редакции Андрея Одегова Вопрос 10: По ходу действия фильма в кадре появляются большая железная ванна с номером 2, шапочка для бассейна с номером 43, бегущие по берегу моря спортсмены с номерами 70 и 71, плывущая по реке лодка с номером 100... Назовите этот фильм. Ответ: "Утопая в числах". Зачет: "Отсчет утопленников", "Drowling by Numbers". Комментарий: Темы воды и чисел красной нитью проходят сквозь фильм Питера Гринуэя, по ходу действия картины в кадре последовательно появляются числа от 1 до 100. Кстати, четверо несчастных погибли как раз в ванне, бассейне, море и реке. Во дворце спорта "Ока" днем работает бассейн, желающие могут посетить его - это повысит бодрость духа и улучшит игровую атмосферу. Только перед этим желательно отдохнуть, чтобы не произошло непоправимое. Вечная слава воде! Источник: http://www.petergreenaway.org.uk/numberloc.htm Автор: Владислав Король Вопрос 11: Василий Авченко в шутку назвал кАмбалу дауншифтером, предпочитающим активности философскую неподвижность, и сравнил ее с героями известного произведения. Назовите это произведение. Ответ: "На дне". Комментарий: Герои пьесы Горького тоже опустились. Источник: Василий Авченко. Глобус Владивостока. Автор: Владислав Король Вопрос 12: По словам Льва Лурье, в начале восьмидесятых известное выражение приобрело глубокий политэкономический смысл и ВТОРЫЕ были на самом деле важнее ПЕРВЫХ. Действительно, без блата приобрести дефицитные товары было сложно. Назовите имя того, кого сейчас можно увидеть на ПЕРВЫХ. Ответ: Аполлон. Комментарий: "Не имей сто рублей, а имей сто друзей". На современной купюре в 100 рублей изображены Большой театр и квадрига тетриппа Аполлона. Источник: "Огонек", 2013, N 36. Автор: Владислав Король Вопрос 13: Дмитрий Галковский назвал ИМИ советских философов Эвальда Ильенкова, Жореса Медведева, Ричарда Косолапова и Энгельса Чудинова. В одном из слов предыдущего предложения мы пропустили пять букв. Напишите это слово в исходном виде. Ответ: Химерами. Комментарий: По аналогии с мифологической Химерой в биологии так называют организмы, состоящие из генетически разнородных тканей, в лингвистике - слова, части которых происходят из разных языков. К "гротескному поколению" советских философов со странными сочетаниями имен и фамилий Галковский относится критически. Источник: Дмитрий Галковский. Бесконечный тупик (примечание к N 321). http://www.samisdat.com/3/311-328.htm Автор: Владислав Король Вопрос 14: В русском переводе фильма "Зеленая миля" сидящий на электрическом стуле герой упомянул ЕЕ после того, как на его макушке закрепили последний круглый электрод. Назовите ЕЕ шестибуквенным словом. Ответ: Крышка. Комментарий: (pic: 20140038.jpg) В оригинале звучат слова "I'm getting the cap", а в русской версии - "Теперь мне крышка". Дело было во время репетиции казни - тюремный уборщик, игравший роль приговоренного, шутливо комментировал происходящее. Источник: 1. Фильм "Зеленая миля", 38-я минута. 2. http://www.script-o-rama.com/movie_scripts/g/green-mile-script-transcript-darabont.html Автор: Владислав Король Вопрос 15: Поезд, соединявший Бишкек с югом Киргизии, за один рейс ДЕЛАЛ ЭТО девять раз и вскоре был отменен. Назовите одним словом тех, кто попытался СДЕЛАТЬ ЭТО в произведении 1939 года. Ответ: Самураи. Зачет: Японцы. Комментарий: Столица Киргизии переименована в Бишкек в 1991 году, а юг страны отделен от севера высокими горами. После распада СССР поезд шел по территориям четырех государств с весьма причудливыми границами и разным визовым режимом по отношению друг к другу, и долго так продолжаться не могло. В советской песне самураи решили перейти границу у реки, но у них ничего не вышло. Источник: 1. Антон Кротов. Дом для всех (Ош-2009). 2. http://www.sovmusic.ru/text.php?fname=3tankist Автор: Владислав Король Вопрос 16: [ПЕРВАЯ СТРОКА ПРОПУЩЕНА], Не кричу, не требую суда. Потому что так и не иначе Жизнь сложилась раз и навсегда. Восстановите пропущенную первую строку в четверостишии Сергея Гандлевского. Ответ: "Не жалею, не зову, не плачу". Комментарий: Грустное стихотворение Гандлевского во многом перекликается с есенинскими строками. Например, у него первая строка - "Расцветали яблони и груши", а у Есенина читаем: "Всё пройдет, как с белых яблонь дым". Источник: 1. http://www.vavilon.ru/texts/gandlevsky1-3.html 2. http://www.feb-web.ru/feb/esenin/texts/es1/es1-163-.htm Автор: Владислав Король Вопрос 17: (pic: 20140039.jpg) Картина Николая Копейкина, фрагмент которой вы видите, называется "ИКС". Другая картина Николая Копейкина тоже называется "ИКС". Кто на ней изображен? Ответ: Николай Копейкин. Комментарий: Картина называется "АВТОпортрет". Обычный автопортрет у него тоже есть. Источник: http://www.gqdom.ru/shop.php_g=680.html Автор: Владислав Король Вопрос 18: <раздатка> Там ходит поэт салакою, там каждый, кому не лень, В стихах ___________-балакает по тысяче строчек в день. И будет награда царская тому, кто всё это за год С непереводимо-____________ на русский переведет. Заполните оба пропуска в розданном вам фрагменте шуточного стихотворения советского переводчика Сергея Хмельницкого. Ответ: Кабардино, балкарского. Комментарий: В советское время многие поэты зарабатывали стихотворными переводами с языков народов СССР по подстрочнику. Источник: Нина Воронель. Содом тех лет. http://www.flibusta.net/b/161392/read Автор: Владислав Король Вопрос 19: <раздатка> A: 3, 3, 5, 5, 7, 7 B: 2, 2, 4, 4, 9, 9 C: 1, 1, 6, 6, 8, 8 В одной статье описываются три игральные кости, отмаркированные в соответствии с раздаточным материалом. Назовите игру, неоднократно упоминающуюся в этой статье. Ответ: "Камень, ножницы, бумага". Комментарий: В парах A-B, B-C и C-A при большом числе испытаний первый кубик выигрывает у второго больше партий, чем проигрывает (вероятность выигрыша первого кубика равна 5/9). "Нетранзитивность превосходства" - принцип логики реального мира, наблюдаемый в теории вероятностей, социальных науках, биологии. В англоязычной литературе его часто называют "rock-paper-scissors". Источник: 1. http://old.computerra.ru/magazine/360108/ 2. http://www.edp.org/dice.htm Автор: Владислав Король, в редакции Сергея Терентьева Вопрос 20: <раздатка> Бесстрашной родины сынам, Их верным мужеству сердцам Я шлю свой пламенный привет И песню, горные орлы! Перед вами фрагмент стихотворения дагестанского поэта Сулеймана Стальского, в котором заменено одно слово. Напишите замененное слово. Ответ: Салам. Комментарий: Вместо "привет". Рифмовка AAAB, а не AABB. Однако в конце стихотворения поэт шлет конникам, героям пробега вокруг Кавказского хребта, свой пламенный привет. Лакец Эффенди Капиев, вероятно, переводил с лезгинского также по подстрочнику. Источник: С. Стальский. Шаири. - Пятигорск: Типолитография имени Анджиевского, 1936. - С. 95. Автор: Владислав Король Вопрос 21: По словам Хуана Карлоса Копеса, у НЕГО "четыре ноги, две головы и одно сердце". Каким ОНО было в названии фильма 1972 года? Ответ: Последним. Комментарий: Речь о фильме Бертолуччи - "Последнее танго в Париже". Источник: 1. http://www.total-positive.com/2011/12/tango.html 2. http://www.imdb.com/title/tt0070849/ Автор: Алексей Пономарев Вопрос 22: Песня Василия Шумова "В.О.Р. N 1" рассказывает о "новом виде олимпийского троеборья в Сочи". "В.О.Р." в названии песни - это аббревиатура. Расшифруйте ее тремя существительными. Ответ: Взятка. Откат. Распил. Комментарий: Автор песни критикует российские власти и злоупотребления при строительстве олимпийских объектов. Источник: http://www.vasilyshumov.com/?cat=29 Автор: Владислав Король Вопрос 23: Внимание, в вопросе есть замена. По словам Марины Леско, заглавная героиня фильма "ХИЩНИЦА" поражает бесчеловечностью даже бандитов. Чтобы вырваться на свободу, она использует других людей, некоторые из которых погибают. Какое слово заменено словом "ХИЩНИЦА"? Ответ: Чужая. Комментарий: "Погоняло у нее откуда такое? - Из фильма. Смотрел, про космос? Там дракон был, с кислотой вместо крови. Яйца в людей откладывал". "Поражает бесчеловечностью" - подсказка. Чужой и Хищник - представители инопланетных рас из голливудских фильмов. Источник: 1. http://ru.wikipedia.org/wiki/Чужая_(фильм,_2010) 2. http://www.newlookmedia.ru/?p=8165 3. http://www.gondolier.ru/154/154chuzai_1.html Автор: Владислав Король Вопрос 24: Внимание, в вопросе есть замена. В шестидесятые годы Лев Ландау проходил курс лечения в Праге. Получив одно радостное известие, супруга академика заметила, что с ее плеч свалилась НЕПОДЪЕМНАЯ НОША. Какие два слова заменены словами "НЕПОДЪЕМНАЯ НОША"? Ответ: Невыносимая тяжесть. Комментарий: В заглавии романа Милана Кундеры, описывающего Прагу шестидесятых, фигурирует "невыносимая легкость". Но мы не любим "игру в дырки"! Всем спасибо. Источник: Кора Ландау. Академик Ландау. Как мы жили. Автор: Владислав Король Тур: 6 тур. "Андреевские заразы" (Москва) Вопрос 1: В песне Вероники Долиной ОНА обращается к своему ненайденному мужу и нерождённому сыну. Какой топоним упоминается в этой песне рядом с "светлым распятьем над черными дверьми"? Ответ: Домреми. Комментарий: Это Жанна д'Арк. Домреми - ее родная деревня. "Домреми" рифмуется с "дверьми". Источник: http://www.kulichki.com/rusclub/Songs/Dolina/sempesen.html Автор: Андрей Волыхов Вопрос 2: В песне "Черная речка" ведро шампанского рифмуется с названием литературного произведения. Назовите его. Ответ: "Женитьба Фигаро". Комментарий: Аллюзия на "Откупори шампанского бутылку иль перечти "Женитьбу Фигаро". Источник: "Серебряная свадьба", песня "Черная речка". Автор: Николай Некрылов Вопрос 3: Blavod - ТАКАЯ ОНА, придуманная маркетологом Марком Дорманом. При смешении с клюквенным соком дает жидкость пурпурного цвета. Что мы заменили на "ТАКАЯ ОНА"? Ответ: Черная водка. Комментарий: "Blavod" - акроним от "black vodka". Про цвет при смешении тоже не просто так упомянуто. Источник: http://ru.wikipedia.org/wiki/Blavod Автор: Игорь Тюнькин Вопрос 4: В одном из своих стихотворений Иосиф Бродский, рассуждая о том, что природа оставила нам от динозавров, сожалеет о том, что по НЕМУ некуда позвонить. Назовите ЕГО. Ответ: Позвоночник. Источник: И. Бродский. До сих пор. Автор: Николай Некрылов Вопрос 5: Николай Ямской, выражая скептическое выражение к переменам, произошедшим после Октябрьской революции, заменяет букву Т на Ч в строчке из известной песни. Воспроизведите получившуюся строку. Ответ: "До основанья - а зачем?". Источник: Н. Ямской. Московские бульвары: начало прогулки. Автор: Андрей Волыхов Вопрос 6: В одном мультфильме индус после секса обещает партнерше рассказать всем о том, что она осталась ЕЙ. Ответьте точно, какое слово мы заменили на "ЕЙ". Ответ: Неприкасаемой. Источник: "Симпсоны", s07e21. Автор: Николай Некрылов Вопрос 7: Согласно передаче "С точки зрения науки", ОНА становится видимой за счет резкого сжатия, приводящего к конденсации влаги. Рассказывая о технике безопасности, участники "Разрушителей мифов" предостерегают, что ОНА способна серьезно повредить легкие. Назовите ЕЕ двумя словами, начинающимися на одну и ту же букву. Ответ: Взрывная волна. Комментарий: Во фронте взрывной волны происходит резкое сжатие воздуха, что приводит в конденсации влаги. Взрывная волна способна сильно повредить легкие, распространяясь по телу человека. Источник: 1. "С точки зрения науки". Дело о планете Земля. Сила взрыва. 2. "Разрушители мифов". Автор: Игорь Тюнькин Вопрос 8: Певица Ольга Краузе воспроизводит реакцию настройщика, которого после войны пригласили к приобретенному ее семьей пианино: "Стать обладателем возрожденного "Беккер" - это сказочная удача!". А какое качество инструмента прагматичный настройщик хвалил больше всего? Ответ: "И, что самое ценное, его же не сопрут - надорвутся!". Зачет: По смыслу (слишком тяжелое, чтобы украли). Источник: http://www.proza.ru/2006/10/16-362/ Автор: Екатерина Лобкова Вопрос 9: Ги Бретон цитирует высказывание о том, что Генрих IV был бы идеальным героем, если бы его постигла участь его соотечественника. Назовите этого соотечественника. Ответ: Пьер Абеляр. Комментарий: Генрих IV был известен большой любвеобильностью. Источник: Г. Бретон. В кругу королев и фавориток. Автор: Андрей Волыхов Вопрос 10: Когда герои мультсериала "Американский папаша" усыновляют ребенка по имени Немо, с ним начинаются некоторые проблемы. Вскоре они понимают, что Немо - это ИКС. Назовите ИКС двумя словами, начинающимися на соседние буквы алфавита. Ответ: Омен наоборот. Источник: Мультсериал "Американский папаша". Автор: Николай Некрылов Вопрос 11: В одном из произведений Терри Пратчетта фигурируют странствующие из клана в клан Макгоннаглы, названные в честь одного из худших поэтов-баталистов Шотландии. Какая фамилия использовалась в русском переводе вместо Макгоннагла? Ответ: Бездомный. Комментарий: Образец плохого поэта - Иван Бездомный. Источник: Т. Пратчетт. Вольный народец. Автор: Игорь Тюнькин Вопрос 12: Форум, посвященный проблемам геронтологии, назывался "50 ИКС: все ИКСЫ зрелого возраста". Что мы заменили на ИКС? Ответ: Плюс. Источник: http://www.50plus.ru/ Автор: Андрей Волыхов Вопрос 13: В начале 1990-х было распространено шуточное предложение поставить первым Юрия, вторым Владимира, третьим Григория. Назовите фамилию любого из них. Ответ: Болдырев. Зачет: Лукин, Явлинский. Комментарий: Как известно, первые три буквы в названии партии "Яблоко" - аббревиатура фамилий основателей - Явлинского, Болдырева и Лукина. Источник: http://www.ej.ru/?a=note&id=8406 Автор: Андрей Волыхов Вопрос 14: Экономист Константин Сонин рассказал в своем Живом Журнале, что свои первые успешные статьи начал писать отчасти под влиянием ареста Михаила Ходорковского, и что теперь ему есть что отметить. Именем какого литературного персонажа был назван этот пост? Ответ: Виконт де Бражелон. Комментарий: Десять лет спустя. Источник: http://ksonin.livejournal.com/524981.html Автор: Кирилл Борусяк Вопрос 15: В продолжении одного розыгрыша человеку, исполняющему роль ЕГО, сзади на плечо кладет руку, вымазанную сажей, другой человек, играющий роль ЕГО, и спрашивает: "А зачем тебе мой хлебушек?". Назовите ЕГО. Ответ: Черный альпинист. Зачет: Черный турист и любые другие словосочетания из двух слов, содержащие прилагательное "черный" и какое-либо хобби, связанное с походной деятельностью. Источник: http://ru.wikipedia.org/wiki/Чёрный_альпинист Автор: Николай Некрылов Вопрос 16: Согласно заметке на сайте lenta.ru, сколько ни пытались другие блогеры сделать собственную рубрику "Фото дня", [ДВА СЛОВА ПРОПУЩЕНО] в Рунете так и не появилось. Два пропущенных слова отличаются друг от друга лишь капитализацией. Воспроизведите пропущенные слова. Ответ: Другого Другого. Комментарий: ЖЖ юзер drugoi долгое время был признанным лидером по фотографиям в Рунете. Источник: http://lenta.ru/photo/2011/02/02/livejournal/#19 Автор: Игорь Тюнькин Вопрос 17: В песне Тома Лерера отмечается, что она отличает даму от дамбы, а вот его друг при этом не меняется. А как его зовут? Ответ: Сэм. Зачет: Сэмуэль, Sam. Комментарий: Том Лерер поет на английском. Как и в русском языке, дамбу от дамы отличает одна буква, только это буква "e". А вот его друг Sam остается same - тем же самым. Источник: http://www.youtube.com/watch?v=7TKDcHEcE8Q Автор: Игорь Тюнькин Вопрос 18: Заполните пропуски в гарике Игоря Губермана: Друзья мои! Навек вам нежно [ПРОПУСК 1], я щедростью душевной вашей [ПРОПУСК 2]; надеюсь, я не буду вами [ПРОПУСК 1], и этот долг не будет вами [ПРОПУСК 2]. Ответ: Предан, взыскан. Комментарий: Друзья мои! Навек вам нежно предан, я щедростью душевной вашей взыскан; надеюсь, я не буду вами предан, и этот долг не будет вами взыскан. Источник: http://www.litru.ru/?book=11236&page=2 Автор: Екатерина Лобкова Вопрос 19: Михаил Кочетков говорил: "Самая эротичная мужская профессия - позировать ЕМУ". ОН родился в 1846 году. Назовите ЕГО. Ответ: [Карл] Фаберже. Источник: 1. http://fedalnik.livejournal.com/3249.html 2. http://ru.wikipedia.org/wiki/Фаберже,_Карл Автор: Андрей Волыхов Вопрос 20: В одном мультсериале влюбленные собаки фотографируются, засунув головы в плакат с одной картиной. Назовите автора этой картины. Ответ: [Кассиус Марселус] Кулидж. Комментарий: Американский художник, известный серией картин, представляющих собак в повседневной жизни людей. Источник: 1. http://en.wikipedia.org/wiki/Cassius_Marcellus_Coolidge 2. Мультсериал "Симпсоны". Автор: Николай Некрылов Вопрос 21: Дуплет. 1. В одной из серий "Симпсонов" англоязычное название устройства для эвтаназии, исполняющего при этом музыку, двумя буквами отличается от названия реального плеера. Воспроизведите название этого устройства. 2. Одна из композиций, исполняемой на этом устройстве, соответствует максимальному уровню. Какой американской метал-группе она принадлежит? Ответ: 1. Diepod. 2. "Megadeath". Комментарий: Название "diepod" образовано комбинацией слов "die" - умереть и "iPod" - названия популярного плеера. Название группы "Megadeath" можно перевести как "мегасмерть". Источник: "Симпсоны", s17e16. Автор: Игорь Тюнькин Вопрос 22: Назовите первую станцию Московского метрополитена, на которой после реконструкции были добавлены световые полосы на посадочных платформах. Ответ: "Электрозаводская". Комментарий: Название станции тоже связано с электричеством. На всякий случай: на всех упомянутых в источнике более ранних станциях полосы были установлены сразу, а не при реконструкции. Источник: http://ru.wikipedia.org/wiki/Электрозаводская_(станция_метро) Автор: Андрей Волыхов Вопрос 23: Оперкулум - это выпуклая с одной стороны крышечка, закрывающая устье раковины морских и пресноводных брюхоногих. Джеймс Хорнелл, будучи директором департамента рыбного хозяйства Мадраса, опубликовал несколько статей об индийских моллюсках, в том числе он описал как местное население использует оперкулум некоторых из них. Ответьте словом, которое пишется через дефис: в качестве чего при этом он использовал оперкулум особо крупного вида моллюсков? Ответ: Пресс-папье. Источник: http://www.manandmollusc.net/operculum_paul.html Автор: Николай Некрылов Вопрос 24: Прослушайте цитату из стихотворения Евгения Сазонова "Раздумины о чистоте литературного языка": И скажу я вам в виде напутствия: Так пишите строку за строкой, Чтоб добиться [два слова пропущено] Недостатков, указанных мной! Заполните пропуск устойчивым выражением. Ответ: "... наличья отсутствия...". Зачет: "... наличия отсутствия...". Источник: Л.К. Граудина, Е.Н. Ширяев. Культура русской речи. - М.: НОРМА, 2009. - С. 226. Автор: Екатерина Лобкова Тур: 7 тур. "Тапир во время чумы" (сборная) Вопрос 1: Персонаж Диккенса в припадке ревности счел ИХ излишними. ОНИ фигурируют в произведениях американского и русского писателей. Назовите ИХ двумя словами. Ответ: Зеленые очки. Комментарий: У ревности зеленые глаза. Писатели - Баум и Волков. Источник: http://www.knigki.ru/demo/book/6511/131.html Автор: Андрей Островский Вопрос 2: В конце 1960-х Чарльз Мэнсон работал над концепцией грядущей расовой войны. В вышедшем тогда же "ИКСЕ" Мэнсон находил множество подтверждений своих взглядов, хотя значения некоторых строк в его трактовке радикально менялись. Назовите "ИКС". Ответ: "Белый альбом". Зачет: "The White Album", "The Beatles". Комментарий: Особенно важным для Мэнсона был трек "Helter Skelter", который якобы означал грядущий переворот. Источник: http://en.wikipedia.org/wiki/Helter_Skelter_(Manson_scenario) Автор: Юрий Разумов Вопрос 3: Дэвид Фостер Уоллес сравнил с НЕЙ непроницаемое лицо персонажа. Некоторые считают, что ОНА находится между зрителями и критиками. Назовите ЕЕ двумя словами. Ответ: Пятая стена. Комментарий: По аналогии с четвертой стеной - между действием и зрителем - некоторые критики вводят пятую. Источник: 1. David Foster Wallace, Infinite Jest. 2. http://en.wikipedia.org/wiki/Fourth_wall#Fifth_wall Автор: Юрий Разумов Вопрос 4: Владимир Буковский говорит, что изначально этим словом называли движения заключенных внутри тесной, как колода карт, камеры. Напишите это слово. Ответ: Тусовка. Комментарий: Зэки как бы "перетасовываются" внутри камеры. Источник: http://www.snob.ru/magazine/entry/52352 Автор: Николай Рябых Вопрос 5: В стихотворении МОриса Огдена палач, обращаясь к герою, говорит: "Первым был приезжий, потом еврей. Я не действовал против воли твоей". Стихотворение Огдена, заканчивающееся упоминанием пустой площади, иногда сравнивают с более известным текстом, в котором рассказчик неоднократно вспоминает, как ДЕЛАЛ ЭТО. Что мы заменили словами "ДЕЛАЛ ЭТО"? Ответ: Молчал. Комментарий: В стихотворении Огдена рассказчик не пытается помешать палачу, который поочередно казнит всех жителей города. Такое же бездействие описывается и у Мартина Нимёллера: "Когда они пришли за коммунистами, я молчал...". Источник: http://www.colta.ru/articles/society/16 Автор: Андрей Островский Вопрос 6: Одним осенним днем, когда первый морозец уже прихватил ИХ, героиня Сола Беллоу объявила мужу, что им пора расстаться. Назовите ИХ одним словом. Ответ: Помидоры. Комментарий: Мы намекнули на известную прибаутку "Прошла любовь, завяли помидоры". Источник: http://lib.ru/INPROZ/BELLOU/gercog.txt Автор: Андрей Островский Вопрос 7: Знакомая автора вопроса как-то раз описала свои ощущения от доклада участника из Индии на IT-конференции. Ответьте двумя словами, начинающимися на одну и ту же букву, чему был посвящен этот доклад. Ответ: Распознавание речи. Комментарий: Хотя доклад был о компьютерном распознавании речи, речь докладчика было трудно распознать и человеку. Источник: https://twitter.com/Sadwind/status/385348075260502016 Автор: Иван Семушин Вопрос 8: Герой Пинчона говорит, что старая Флоренция утрачивает яркое свечение и всё больше приобретает свинцово-серый оттенок. При этом персонаж сравнивает Флоренцию с НИМ, используя, так сказать, довольно свежую метафору - ведь диалог происходит в конце XIX века. Назовите ЕГО. Ответ: Радий. Комментарий: При радиоактивном распаде радий (что означает "лучистый") превращается в итоге в свинец. Источник: Т. Пинчон "V.". Автор: Иван Семушин Вопрос 9: На одном сайте этим словом называют специалистов по борьбе с утечками на нефтепроводах. Назовите известный роман, в котором это слово служит ироническим наименованием рода занятий основных персонажей. Ответ: "Моби Дик, или Белый кит". Комментарий: Это слово - "фонтанщик". Источник: 1. http://www.pasf-mcpb.ru/dokumenty/literatura 2. http://modernlib.ru/books/melvill_german/mobi_dik_ili_beliy_kit/read/ Автор: Евгений Пашковский Вопрос 10: В одном романе говорится, что в ЭТОЙ стране нет ни гор, ни ям, ни темных пятен, одни лишь чистые цвета и прямые линии. Назовите эту страну. Ответ: Голландия. Комментарий: Неслучайно, видимо, в этой стране родился Пит Мондриан, на картинах которого в основном тоже чистые цвета и прямые линии. Источник: С. Нотебоом. День поминовения. Автор: Иван Семушин Вопрос 11: Восстание против короля Франциска II в 1860 году начали монахи палермского монастыря Ганчиа, что должно было послужить напоминанием о НЕЙ. Назовите ЕЕ двумя словами. Ответ: Сицилийская вечерня. Комментарий: Знаменитое восстание сицилийцев в 1282 году, сигналом к которому послужил колокольный звон. Источник: http://www.giuseppe-garibaldi.ru/bez-rubriki/dzhuzeppe-garibaldi-a-lure-stranica-70.html Автор: Андрей Островский Вопрос 12: CLARITY [клэрити] - метод обработки неживой нервной ткани, который позволяет СДЕЛАТЬ ЭТО, добившись прозрачности, и специальным гелем прокрасить отдельные клетки. Герой Андрея Платонова ДЕЛАЛ ЭТО самому себе при помощи книг. Что мы заменили словами "ДЕЛАТЬ ЭТО"? Ответ: Промывать мозги. Комментарий: В первом случае - в прямом смысле, а во втором - в переносном. Источник: 1. http://www.slon.ru/biz/1038188/ 2. http://books.rusf.ru/unzip/add-on/xussr_mr/platoa15.htm?2/4 Автор: Андрей Островский Вопрос 13: Фриденсрайх Хундертвассер стремился к единению с природой. В частности, он хотел быть захороненным в саду без одежды и гроба. Не случайно в названии одной из его картин идут подряд два схожих латинских слова. Напишите их. Ответ: Homo, humus. Комментарий: После смерти Хундертвассер хотел стать единым целым с землей. Источник: 1. http://family.booknik.ru/articles/sovremennye-geroi/tantsuyushchie-okna-khundertvassera/ 2. http://www.hundertwasser.ru/painting/pages/1986-cat849A-HWG95.html Автор: Иван Семушин Вопрос 14: Энди Гринуолд пишет, что определение "эмо" стало постыдным в музыкальном сообществе. Гринуолд упоминает ЕЕ на ремне гитары. Правда, в случае с эмо ОНА - это английская "E" [и]. Назовите ЕЕ двумя словами. Ответ: Алая буква. Комментарий: По аналогии с алой "A" из романа Готорна, которую героиня должна была носить на одежде. Источник: Andy Greenwald, Nothing Feels Good: Punk Rock, Teenagers, and Emo. Автор: Юрий Разумов Вопрос 15: Эрнст Фишер ставит под сомнение факт гонений на этого человека, ведь он "переместил" людей ближе к Богу. Назовите этого человека. Ответ: [Николай] Коперник. Комментарий: Убрав человека из центра Вселенной, автор идеи о гелиоцентрической солнечной системе как бы приблизил человека к небесам. Источник: Э.П. Фишер. Растут ли волосы у покойника? Мифы современной науки. Автор: Николай Рябых Вопрос 16: В произведении Чарльза Стросса "Очень [ПРОПУСК]" герои противодействуют КГБ в Антарктиде. Какие два слова мы пропустили? Ответ: Холодная война. Источник: Ч. Стросс. Очень холодная война. Автор: Иван Семушин Вопрос 17: В этом вопросе словом "ИКС" заменено четыре слова. Итальянский автор Антонио Табукки так пишет о своем персонаже: "... Перейра впервые посетил меня однажды вечером в сентябре 1992-го. В то время он еще не звался Перейрой, ... был чем-то расплывчатым, ускользающим и туманным, но уже тогда у него было твердое намерение стать героем романа. Это был пока что один ИКС". Сколько ИКСОВ упоминается в названии произведения 1921 года? Ответ: Шесть. Комментарий: Это произведение - пьеса Луиджи Пиранделло "Шесть персонажей в поисках автора". Источник: 1. А. Табукки. Утверждает Перейра. 2. http://ru.wikipedia.org/wiki/Пиранделло,_Луиджи Автор: Иван Семушин Вопрос 18: Герой Артуро Переса-Реверте поддерживал хорошие отношения и с правительством Андалусии, и с оппозицией. Кроме того, он оставался нейтрален по отношению к "ПЕРВОЙ" и "ВТОРОМУ". Назовите "ПЕРВУЮ" и "ВТОРОЙ". Ответ: "Севилья" и "Бетис". Комментарий: Севилья - столица Андалусии. В городе есть два футбольных клуба, ведущих между собой непримиримую вражду. Источник: А. Перес-Реверте. Кожа для барабана. - М.: Эксмо, 2005. - С. 119-120. Автор: Андрей Островский Вопрос 19: В этом вопросе "ИКС" заменяет одно слово. В начале одного из своих произведений Стивен Кинг называет ИКС "домом, который построил Будвайзер". Назовите это произведение. Ответ: "Худеющий". Комментарий: ИКС - живот. В начале книги "Худеющий" у персонажа большой пивной живот. Источник: http://www.loveread.ec/read_book.php?id=1624&p=5 Автор: Евгений Пашковский Вопрос 20: Эскимосы верят, что богиня морей Седна [ПРОПУСК]. Мы не спрашиваем, кто из мультипликационных персонажей [ПРОПУСК]. Просто ответьте, какие четыре слова мы пропустили. Ответ: Проживает на дне океана. Комментарий: Кто проживает на дне океана? Sponge Bob Square Pants! Источник: http://www.arimoya.ru/Astrology/sedna.html Автор: Андрей Островский Вопрос 21: В отличие от известного афоризма русского литератора, изречение Эмиля Чорана заканчивается словами: "... не ройся в памяти". А как оно начинается? Ответ: Хочешь быть счастливым. Зачет: Если хочешь быть счастливым; если ты хочешь быть счастливым. Комментарий: А Козьма Прутков говорил: "Если хочешь быть счастливым, будь им". Источник: http://magazines.russ.ru/inostran/1998/11/chorn.html Автор: Андрей Островский Вопрос 22: В заглавии англоязычной заметки об известном писателе упоминается стремительное движение к смерти. Назовите этого писателя. Ответ: Салман Рушди. Комментарий: "Rushdie and the Rush to Die". Салман Рушди после написания "Сатанинских стихов" уже много лет живет, рискуя умереть от рук фанатиков. Источник: http://www.notbored.org/rushdie.html Автор: Евгений Пашковский Вопрос 23: Лоренс Даррелл пишет, что замкнувшиеся в себе интеллектуалы воспринимают секс как нечто, данное свыше, поэтому для них секс - все равно что ОН. Назовите ЕГО двумя словами. Ответ: Золотой дождь. Комментарий: На Данаю пролился Зевс в виде золотого дождя, когда она находилась в замкнутом помещении. Источник: Л. Даррелл. Александрийский квартет. - Кн. 4 "Клеа". Автор: Иван Семушин Вопрос 24: Салман Рушди наделил персонажа, отпускающего ехидные реплики, такими чертами: иссушённое лицо, искривленные губы. Ответьте одним словом, с кем Рушди сравнивает этого персонажа. Ответ: Тролль. Комментарий: Такое описание появилось в книге Рушди задолго до появления "троллфейса". Источник: С. Рушди. Сатанинские стихи. Автор: Иван Семушин Тур: 8 тур. "Сборная с. Тупино" aka "БорисКо" (Ступино - Коломна) Инфо: Эпиграф. Пост пользователя trololo1111 в сообществе chgk.livejournal.com: "А существуют играющие в ЧГК алкоголики или наркоманы?". Ответ пользователя stephan_nn: "Алкоголики играют, наркоманы пишут вопросы". Вопрос 1: Внимание, в вопросе есть одна замена. В 2005 году НТВ предложило нам "ВОЗЛОЖЕНИЕ ЛЯЖКИ с Олегом Меньшиковым". Можно сказать, что некоторым из присутствующих некоторое время назад было предложено "ВОЗЛОЖЕНИЕ ЛЯЖКИ". Кто-то даже воспользовался этим предложением. Какие два слова мы заменили на "ВОЗЛОЖЕНИЕ ЛЯЖКИ"? Ответ: Первая ночь. Комментарий: Новогодняя передача на НТВ называлась "Первая ночь с Олегом Меньшиковым". 24 января 2014 года с адреса Олега Холодова была сделана рассылка по адресам, в которой, в частности, задавался вопрос "Какой тур ваша команда хотела бы проводить?" и прилагалось расписание, в котором было много свободных мест на первую ночь. "Право первой ночи" также носит название "право возложения ляжки". Автор: Юлия Кощеева, Виктор Плотников Вопрос 2: Закончите одним словом пирожок: аркадий часто бьет оксану в живот руками и ногой оксана любит и прощает и ждет когда его... Ответ: родит. Источник: http://vk.com/perawki?w=wall-28122932_32780 Автор: Олег Евстафьев Вопрос 3: Лидер одной из панк-групп выставил на интернет-аукцион лот стоимостью 510 долларов. Относительно невысокую цену музыкант объяснил тем, что состояние товара не тянет больше, чем на б/у. Заметка об этом любопытном факте появилась на портале vesti.ru. Автор заметки, видимо, не до конца понял, о чем идет речь, и предположил, что музыканту потребуется хирургическая операция. На самом же деле тот просто хотел продать ненужную вещь. Что было выставлено на аукцион? Ответ: ОргАн. Комментарий: Ехидные обсуждения на английском языке в Интернете о том, какой именно Орган хочет продать американец, сбили многих журналистов с толку. Как позже выяснилось, исполнитель продает не часть тела, а музыкальный инструмент. Источник: 1. http://www.ebay.com/itm/Fat-Mike-of-NOFX-Wurlitzer-Organ-/281190161092 2. https://www.facebook.com/permalink.php?story_fbid=10151638805041577&id=180985116576 3. http://www.vesti.ru/doc.html?id=1143465 Автор: Александр Романкин Вопрос 4: (pic: 20140040.jpg) Перед вами рекламный плакат клиники. На лечении чего она специализируется? Ответ: Заикание. Комментарий: Ass-ass-astronaut. Источник: http://www.coloribus.com/adsarchive/prints/stuttering-therapy-classes-ass-astronaut-18638205/ Автор: Юлия Кощеева, Виктор Плотников Вопрос 5: Внимание, в вопросе есть две замены. В отличие от математики, где допускается перемена мест слагаемых, перестановка слов местами может привести к интересным результатам. Например: "ИКС переАЛЬФИТЬ" - это почти то же самое, что "с АЛЬФОЙ переИКСИТЬ". Что мы заменили на ИКС и АЛЬФА? Ответ: Борщ, соль. Комментарий: "Борщ пересолить" - это то же самое, что "с солью переборщить". Источник: http://atkritka.com/188887/ Автор: Юлия Кощеева, Виктор Плотников Вопрос 6: Внимание, в вопросе есть одна замена. И еще немного об игре слов. Сторож прислушался и ушел на пенсию. Какое слово в предыдущей фразе мы заменили на "прислушался"? Ответ: Насторожился. Источник: http://atkritka.com/299952/ Автор: Юлия Кощеева, Виктор Плотников Вопрос 7: Ведущие седьмой площадки эрудит-квартета на калужском фестивале "Гостиный двор" как-то раз начали игру с фразы "Мы за НЕЕ русского языка". Назовите ЕЕ. Ответ: Ёфикация. Комментарий: Традиционно площадки эрудит-квартета на "Гостином дворе" обозначаются буквами алфавита. Седьмая площадка там традиционно носит название "Ё". Источник: ЛОАВ. Автор: Юлия Кощеева Вопрос 8: Однажды американка Джуди Берри выиграла конкурс, объявленный по радио. Рассчитывала она в качестве приза автомобиль получить. Маленькое и зеленое нечто вместо этого получила она. Однако Джуди смогла-таки отсудить у организаторов конкурса автомобиль. А какой марки? Ответ: "Тойота". Комментарий: Поначалу ей вручили игрушечного мастера Йоду. "Toyota" и "toy Yoda" - на слух неотличимы, и компания пыталась так пошутить, тем более конкурс был объявлен 1 апреля. Примечание: мы знаем, что дело происходило в Панаме, а не в США. Источник: http://usatoday30.usatoday.com/news/nation/2002/05/09/toy-yoda.htm Автор: Олег Евстафьев Вопрос 9: [Ведущему: не озвучивать, что "Вера" написано с большой буквы.] Если признать, что на картине Веласкеса "Завтрак" все-таки присутствует ЭТО, то окажется, что Вера не имеет к созданию ЭТОГО никакого отношения. Назовите ЭТО. Ответ: Граненый стакан. Комментарий: Вроде как прототип граненого стакана (только не с вертикальными, а с косыми гранями) есть на картине 1617-1618 годов, а значит, Вера Мухина - вовсе не его маменька. Источник: http://www.ginline.ru/intersno-pochitaty/istoriya-granenogo-stakana Автор: Юлия Кощеева Вопрос 10: Опубликованная в сентябре 2004 года интернет-статья посвящена событиям в Челябинске. По заголовку можно подумать, что речь в ней идет о природных аномалиях. На самом деле в статье говорится о нанесении ущерба казне и привлечении к ответственности виновных. Какое число упоминается в заголовке статьи? Ответ: 40. Комментарий: Статья в электронном периодическом издании "Интернет-проект "MEDIAZAVOD.RU" под названием "40 градусов "в тени"" повествует о незаконном обороте алкогольной продукции в Челябинске. Источник: http://www.mediazavod.ru/articles/16432 Автор: Александр Романкин Вопрос 11: Заглавный герой рассказа Аверченко утверждал, что в его фамилии должно быть две "с" [эс], так как она происходит из французского и означает "прекрасная обедня". Напишите фамилию героя. Ответ: Бельмесов. Зачет: Бельмессов. Источник: http://www.flibusta.net/b/159274/read Автор: Василий Сумин Вопрос 12: <раздатка> Ям-тирьям-тирьям, в коляске Две усатых Свистопляски Босиком, бегом-бегом Ловят ветер сапогом! Перед вами отрывок из "Хохотальной путаницы" Юнны Мориц. Автор вопроса предположил, что в этой частушечке-хохотушечке сделан некий пропуск, из-за чего ее текст стал нелогичным. Что же было пропущено? Ответ: [Буква] С. Комментарий: Свистопляски усатые, и значит, взрослые - и тем не менее в коляске. Но если букву С в слове "усатых" удвоить - то это бы многое объяснило. Источник: В вопросе. Автор: Олег Евстафьев Вопрос 13: (pic: 20140041.jpg) Розданная вам картинка послужила основой для демотиватора. В его тексте, в отличие от известной фразы, фигурирует другое лицо. Воспроизведите текст демотиватора. Ответ: "Хьюстон, у вас проблемы". Зачет: "Хьюстон, у вас проблема". Источник: (pic: 20140042.jpg) Автор: Олег Евстафьев Вопрос 14: Блиц. На одном изображении три героя рассуждают о своей будущей жизни. 1. Первый герой говорит: "Когда меня достанут из коробки, вдруг я не найду своего хозяина?". Назовите этого героя. 2. Второй герой сокрушается: "Когда меня достанут из ящика, вдруг я не найду друзей?". Назовите этого героя. 3. Третий герой грустно говорит: "Ребята, мне бы ваши проблемы...". Назовите этого героя. Ответ: 1. Барашек [из "Маленького принца"]. 2. Чебурашка. 3. Кот [Шрёдингера]. Источник: http://ryba-barrakuda.livejournal.com/270068.html Автор: Юлия Кощеева, Виктор Плотников Вопрос 15: В сериале "Морская полиция. Спецотдел" расследуется убийство военного, мать которого была выходцем из крохотного африканского племени. В ходе расследования в качестве сравнения используется название совсем другого племени. Какого? Ответ: Навахо. Комментарий: Речь идет о неком языке "джалу" - языке крохотного племени, почти мертвом. Он использовался для шифрования африканскими террористами, а этот американский военный, как носитель языка, помогал разобраться с шифром, за что и погиб преждевременно. Википедия, правда, в отличие от киноисточника, называет это язык "ауджила", но признаёт, что только небольшое население оазиса Джалу говорит на сохранившемся берберском языке. Источник: "NCIS. Спецотдел", s11e07. Автор: Юлия Кощеева Вопрос 16: Внимание, в вопросе есть одна замена. Согласно словарю В.В. Онуфриева, вождь - это АЛЬФА. "АЛЬФА" - название мультфильма режиссера Светланы Халькиной 2002 года. Героине этого мультфильма оказалось проще найти друга, чем АЛЬФУ. Мы не просим вас найти АЛЬФУ. Назовите АЛЬФУ четырьмя словами. Ответ: Рифма к слову "дождь". Источник: http://www.multy.tv/4547-rifma-k-slovu-dozhd.html Автор: Елизавета Феофилова Вопрос 17: Прослушайте слегка измененную цитату из романа "Апофегей" Юрия Полякова: "... потом они вместе шли ... и разговаривали - обо всем: о полном маразме профессорско-преподавательского состава, о явных переменах в интимной жизни студентов, ... о стрельбе по-македонски, об ... оговорке, которой порадовал общественность на .... пленуме державный ОВЕН". В одном из слов этой цитаты мы пропустили шесть букв. Напишите их. Ответ: БРОСЕЦ. Комментарий: Державный брОВЕНосец. Источник: http://www.flibusta.net/b/126810/read Автор: Юлия Кощеева Вопрос 18: Племянница автора вопроса очень огорчилась, узнав, что это слово имеет латинское происхождение и в первую очередь связано с музыкой. Она считала, что это делают перед началом процесса создания новых людей. Назовите это слово. Ответ: Прелюдия. Комментарий: Ну, логично, перед созданием новых людей нужна преЛЮДИЯ. Источник: ЛОАВ. Автор: Юлия Кощеева Вопрос 19: Внимание, пирожок: весенней ночью в туалете [Вторая строка пропущена] евгений проявил смекалку [Четвертая строка пропущена] Мы не спрашиваем, какой химикат упомянут в четвертой строчке этого пирожка. Назовите топоним, в название которого входят последние два слова из второй строчки. Ответ: Улица красных фонарей. Зачет: Квартал красных фонарей. Источник: http://www.perashki.ru/piro/21497/ Автор: Олег Евстафьев Вопрос 20: Внимание, в вопросе есть одна замена. Перевод в сериалах часто становится поводом для улыбки. Так, например, вместо "firewall" можно услышать "стена огня". Но иногда получается неплохо: при сообщении о конфликте пожилого мужчины с полицией выдвигается предположение, что "старика застукали со СНЕЖНОЙ бабулечкой". Какое слово мы заменили на "СНЕЖНОЙ"? Ответ: Ночной. Комментарий: "... Думаю, старика застукали с ночной бабулечкой...". Источник: "Морская полиция. Спецотдел", s11e07, 5'. Автор: Юлия Кощеева Вопрос 21: Во время Второй мировой войны немецкие танкисты, воевавшие на севере Африки, считали хорошей приметой переехать кучку верблюжьего навоза. Заметив это, союзники изготовили противотанковые мины, замаскированные под эти кучки. Потеряв несколько танков, немцы стали избегать нетронутый навоз. Тогда союзники изменили внешний вид мин - теперь на них появилось... Что? Ответ: Следы [гусениц] [танка]. Источник: http://muzey-factov.ru/tag/mines#5755 Автор: Юлия Кощеева, Виктор Плотников Вопрос 22: Внимание, в вопросе есть одна замена. Однажды во время праздника на одного из гостей опрокинули его тарелку. Согласно шутке автора вопроса, до конца застолья этот гость смотрел НЕПРИЯЗНЕННО. Какие два слова мы заменили словом "НЕПРИЯЗНЕННО"? Ответ: Из-под лобио. Комментарий: В тарелке было именно лобио. Источник: ЛОАВ. Автор: Олег Евстафьев Вопрос 23: Оказывается, логотип этой торговой марки символизирует переход от аналоговой техники, обозначенной волной, к цифровой, обозначенной двоичным кодом. Комментируя этот факт, пользователь ЖЖ иронично отметил, что точка в "аналоговой" части логотипа отражает еще одно понятие - корпускулярно-волновой дуализм. Назовите эту торговую марку. Ответ: VAIO. Комментарий: Буквы V и A изображены в виде волны, буквы I и O изображены в виде стилизованных единицы и нуля. Точка в букве А, по мнению ЖЖ-пользователя обозначает корпускулярно-волновой дуализм. Источник: http://mi3ch.livejournal.com/2458848.html Автор: Юлия Кощеева, Виктор Плотников Вопрос 24: Когда госпожа БульденкОва вышла замуж за господина Каминского, то ЭТО стало вызывать сомнения - казалось, что должно быть бОльшим. Когда госпожа Андреева после развода стала госпожой Ковалевой, ЭТО тоже стало вызывать сомнения, ибо казалось, что должно быть больше. У авторов данного вопроса ЭТО чуть больше 16 тысяч и чуть больше 25 тысяч. Назовите ЭТО. Ответ: Идентификатор в рейтинге ЧГК. Зачет: По смыслу. Комментарий: При создании рейтинга идентификаторы присваивались по алфавиту. Создание и расторжение браков привело к смене фамилий и, как следствие, несоответствию исходного алфавитного принципа и величины номера в рейтинге. Источник: http://ratingnew.chgk.info Автор: Юлия Кощеева (#16244), Виктор Плотников (#25120) Тур: 9 тур. "Юбка Мёбиуса" (Москва) Вопрос 1: Внимание, в вопросе есть замена. Прослушайте стихотворение Игоря Губермана: "Вовлекаясь во множество дел, Не мечись, как по джунглям ботаник, Не горюй, что не всюду успел, Может, ты СДЕЛАЛ ЭТО". В одном анекдоте старожил, делясь секретами долголетия, говорит, что когда-то СДЕЛАЛ ЭТО. Какие три слова мы заменили на "СДЕЛАЛ ЭТО"? Ответ: Опоздал на "Титаник". Источник: 1. http://magazines.russ.ru/ier/2009/31/gu2.html 2. http://www.anekdot.ru/id/-1070400012/ Автор: Наталья Церетели (Москва) Вопрос 2: [Ведущему: кавычки не озвучивать.] В передаче на "Нашем радио" ведущий отметил, что "Ленинград" стоит на ИКСЕ. Зная, что у слова "ИКС" есть несколько значений, вспомните еще одно из них и назовите школьный предмет, с которым ИКС связан. Ответ: Физкультура. Комментарий: ИКС - это мат. В передаче на радио упоминалась группа "Ленинград". Источник: Передача "Шизгара-шоу". Автор: Наталья Церетели (Москва) Вопрос 3: Эта всем нам известная последовательность в нынешнем виде существует фактически с 1918 года, а официально - лишь с 1942 года. Если приглядеться, в ней можно увидеть лесного зверька и вулканическую горную породу. О какой породе идет речь? Ответ: Туф. Комментарий: Речь в вопросе идет о русском алфавите. Источник: http://ru.wikipedia.org/wiki/Русский_алфавит Автор: Наталья Церетели (Москва) Вопрос 4: Прослушайте политический анекдот: Российскую республику назвали РСФСР, чтобы было удобно и Ленину ДЕЛАТЬ ЭТО ТАК, и Троцкому ДЕЛАТЬ ЭТО СЯК. Что мы заменили словами "ДЕЛАТЬ ЭТО ТАК" или "ДЕЛАТЬ ЭТО СЯК"? Ответ: Писать/читать слева направо, писать/читать справа налево. Зачет: По смыслу. Источник: РСФСР - палиндром, Ленину удобно писать по-русски, а Троцкому (Лейба Давидович Бронштейн) привычнее, видимо, наоборот. :-) Автор: Евгений Тиньков (Москва) Вопрос 5: Картина Модесто Бронкоса "Искупление ИКСА", посвященная популярной теории "отбеливания" бразильского населения в конце XIX века, происходившего в результате активного расового смешения, изображает три поколения одной семьи на пороге бедного жилья: бабушка-негритянка, дочь-мулатка, ее белый муж и, наконец, их практически белый ребенок. В названии картины мы заменили три буквы. Восстановите исходное название. Ответ: "Искупление Хама". Комментарий: Название картины - "Искупление Хама". Название отсылает к библейскому эпизоду о проклятии Хама и всех его потомков. Согласно легенде, потомки Хама были чернокожими из Африки, что долгое время оправдывало работорговлю и само рабство в Бразилии. По сюжету картины, рождение "отбеленного" внука теперь позволит избавить семью от страдания и плохих воспоминаний о рабстве. Таким образом, потомки Хама (негры) были искуплены. Автор: Евгений Тиньков (Москва) Вопрос 6: Научное название ИКСА - умбиликус. ИКСЫ различаются по размерам, форме, глубине. Слово "самба" в переводе с африканского диалекта буквально переводится как "ИКС", что символизировало интимность танца. ИКС - центр известной иллюстрации. Назовите ее автора. Ответ: Леонардо да Винчи. Комментарий: ИСК - это пупок. Известная иллюстрация - рисунок Витрувианского человека. Источник: 1. http://ru.wikipedia.org/wiki/Пуп 2. http://www.vestadance.ru/blog/post_1229191189.html 3. http://ru.wikipedia.org/wiki/Витрувианский_человек Автор: Наталья Церетели (Москва) Вопрос 7: [Ведущему: кавычки не озвучивать.] Этот человек написал свое завещание примерно за сорок лет до смерти. Считается, что прославившая его деятельность началась со "Стычки с финляндскими контрабандистами" и "Искушения". Назовите этого человека. Ответ: Третьяков. Комментарий: "Стычка с финляндскими контрабандистами" и "Искушение" считаются первыми картинами в коллекции Третьякова. Источник: http://ru.wikipedia.org/wiki/Третьяков,_Павел_Михайлович Автор: Наталья Церетели (Москва) Вопрос 8: Микропсия - это неврологическое состояние, при котором человек зрительно воспринимает предметы пропорционально уменьшенными. Временная микропсия может возникнуть у детей между 5 и 10 годами. Такое расстройство психики также называют синдромом ЕЕ. Назовите ЕЕ. Ответ: Алиса [в Стране чудес]. Источник: http://ru.wikipedia.org/wiki/Микропсия Автор: Наталья Церетели (Москва) Вопрос 9: В вопросе есть замены. Далеко не все страны, символика которых включает в себя АРАВИЙСКИЙ ПОЛУМЕСЯЦ, расположены на АРАВИЙСКОМ полуострове. Даже в Эстонии, отнюдь не АРАВИЙСКОЙ стране, одно время обсуждался проект флага с АРАВИЙСКИМ ПОЛУМЕСЯЦЕМ в качестве базового элемента. Какие слова заменены здесь на "АРАВИЙСКИЙ ПОЛУМЕСЯЦ"? Ответ: Скандинавский крест. Источник: http://ru.wikipedia.org/wiki/Флаг_Эстонии Автор: Игорь Мельниченко (Москва) Вопрос 10: Как известно, Шерлок Холмс всегда скептически оценивал деятельность представителей Скотланд-Ярда. Например, в вышедшем в 2009 году фильме "Шерлок Холмс" в эпизоде, когда доктор Ватсон и Шерлок Холмс подходят к месту преступления, есть такой диалог: - Полисмены проделали блестящую работу, уничтожив любые улики. - Да, они никогда не упустят возможность СДЕЛАТЬ ЭТО. Не СДЕЛАЙТЕ ЭТО, написав неправильный ответ. Что мы заменили словами "СДЕЛАТЬ ЭТО"? Ответ: Упустить возможность. Автор: Георгий Коколия (Москва) Вопрос 11: Слишком бдительная испанская цензура в свое время запретила стихотворение Хорхе Гильена "Маха и дон ИКС", несмотря на то, что речь в нем шла не о ИГРЕКЕ, а о поэте Кеведо. Какие слова мы заменили на ИКС и ИГРЕК? Ответ: Франсиско Франко. Источник: "Иностранная литература", 2005, N 7. Автор: Игорь Мельниченко (Москва) Вопрос 12: Согласно скандинавской легенде, богиня Гевьон получила от шведского конунга Гюльви в дар столько земли, сколько смогут утащить четыре быка за одни сутки. Сообразительная Гевьон превратила в быков своих сыновей, рожденных от великана, и они за одни сутки СДЕЛАЛИ ЭТО и утащили кусок земли в другое место. ЭТО СДЕЛАЛ также Дарий I, но уже на границе двух материков. Мы не спрашиваем вас, что заменено словами "СДЕЛАТЬ ЭТО". Назовите появившийся в результате действий Гевьон географический объект, название которого предположительно происходит от датского названия тюленей. Ответ: [Остров] Зеландия. Комментарий: В легенде о появлении острова Зеландия говорится, что он возник после того, как Гевьон с помощью сыновей вырыла канал, ставший впоследствии проливом Эресунд. А Дарий I завершил строительство Суэцкого канала. Источник: 1. Младшая Эдда. 2. http://ru.wikipedia.org/wiki/Зеландия_(остров) 3. http://ru.wikipedia.org/wiki/Суэцкий_канал Автор: Игорь Мельниченко (Москва) Вопрос 13: Прослушайте отрывок из одной из песен "Старшей Эдды", в котором повествуется о смерти бога Бальдра от побега омелы: "Видала, как Бальдр, бог окровавленный, Одина сын, смерть свою принял: стройный над полем стоял, возвышаясь, тонкий, прекрасный омелы побег". Какой вывод можно сделать о флоре Исландии на основании этого текста? Ответ: Омела не растет в Исландии. Зачет: По смыслу. Незачет: Омела не растет в Скандинавии. Комментарий: Омела растет только на деревьях, и ее описание в виде деревца, растущего в поле, неверно. Судя по всему, авторы строк о смерти Бальдра попросту не знали, как она выглядит, ведь "Старшая Эдда" - это сборник исландских песен, а омела встречается на территории Норвегии, но не Исландии. Источник: Старшая Эдда. - М.: АСТ, 2008. - С. 236. Автор: Игорь Мельниченко (Москва) Вопрос 14: Автор вопроса, услышав название картины "Портрет Алика Гинзбурга [ДВА СЛОВА ПРОПУЩЕНО]", на которой автор изобразил Алика Гинзбурга в не самый благополучный для здоровья того период жизни, предположил, что это не картина, а скульптура. Какие слова мы пропустили в названии картины? Ответ: В гипсе. Комментарий: На картине у Гинзбурга загипсована рука. Автор: Игорь Мельниченко (Москва) Вопрос 15: Прослушайте анекдот. "Церковь и клуб атеистов расположены по соседству. Так сказать, [пропуск]". Закончите эту небылицу расхожим выражением из четырех слов, одновременно являющимся названием фильма, снятого в 1964 году. Ответ: Хотите - верьте, хотите - нет. Источник: 1. http://www.amik.ru/Jokes/gid1387/pg4.html 2. http://www.kinopoisk.ru/film/44792/ Автор: Алексей Бутырин (Москва) Вопрос 16: В различных переводах названия этого произведения фигурируют слова "хутор", "уголок" и "хозяйство". Назовите его автора. Ответ: Джордж Оруэлл. Комментарий: Произведение - "Скотный двор", в оригинале - "Animal Farm". Источник: http://ru.wikipedia.org/wiki/Скотный_двор Автор: Наталья Церетели (Москва) Вопрос 17: В конце пятидесятых режиссер Акира Куросава решил снять что-нибудь совсем легкое. В получившемся фильме главными героями стали двое трусоватых крестьян, бравый генерал-мастер фехтования и юная своевольная принцесса. Назовите режиссера, прославившегося фильмами, снятыми, по его собственным словам, под впечатлением от вышеописанной картины Куросавы. Ответ: Джордж Лукас. Источник: http://ru.wikipedia.org/wiki/Три_негодяя_в_скрытой_крепости Автор: Дарья Степаньян (Москва) Вопрос 18: <раздатка> [пропуск] seem to be worth their weight in gold. They are golden looking, golden tasting, and golden priced. Вам был роздан отрывок из одной специализированной книги. Если перевести его на русский язык, то на месте пропуска будет стоять слово, происходящее от названия небольшого животного. Напишите это слово. Ответ: Лисички. Комментарий: Это отрывок из раздела кулинарной книги, посвященного лисичкам. Автор отмечает их внешний вид, превосходный вкус, а также высокую цену. Источник: http://www.mssf.org/cookbook/chanterelle.html Автор: Дарья Степаньян (Москва) Вопрос 19: (pic: 20140043.jpg) В статье "Неожиданные товары от известных брендов" автор представил, что будет, если знаменитые компании начнут производить то, чего от них никто не ждет. Какое название мы скрыли на розданной картинке? Ответ: Adidas. Комментарий: Три параллельные полосы - один из фирменных знаков Adidas. Источник: http://www.adme.ru/brend/neozhidannye-tovary-ot-izvestnyh-brendov-590805/ Автор: Дарья Степаньян (Москва) Вопрос 20: В начале своей карьеры Михаил Козаков играл Гамлета в театре им. Маяковского. Он говорил тогда, что представлял себе своего доброго убитого отца как ПЕРВОГО, а подлеца Клавдия - как ВТОРОГО. Назовите первого и второго. Ответ: Ленин, Сталин. Источник: Телеканал "Культура", программа "Силуэты" от 18.12.2013 г. Автор: Елена Николаева (Москва) Вопрос 21: (pic: 20140044.jpg) <раздатка> "Можете себе представить, одна знакомая моя, 42-летняя дама, узнала себя в двадцатилетней героине моей, и меня вся Москва обвиняет в пасквиле. Главная улика - внешнее сходство: дама пишет красками, муж у нее доктор, и живет она с художником". Какое слово мы пропустили в розданной цитате? Ответ: "Попрыгуньи". Комментарий: Чехов написал эти строки Лидии Авиловой о неприятной истории, приведшей к многолетней ссоре с Левитаном. Источник: Переписка А.П. Чехова. - Т. 2. - М.: Художественная литература, 1984. Автор: Елена Николаева (Москва) Вопрос 22: Внимание, в вопросе есть замена. Вряд ли Эдит Джованна Гассион знала о том, что из-за особенностей строения глаз воробьи видят мир "ТАК". Какие три слова мы заменили словом "ТАК"? Ответ: В розовом цвете. Зачет: В розовом свете. Комментарий: Эдит Джованна Гассион - настоящие имя и фамилия Эдит Пиаф, одна из самых известных песен которой называется "Жизнь в розовом цвете". Псевдоним "Пиаф" переводится как "воробушек". Источник: 1. http://www.eva.ru/animals/read-15076.htm 2. http://www.morebirds.ru/iz-za-osobennostej-stroeniya-glaz-vorobi-vidyat-mir-v-rozovom-svete.html 3. http://ru.wikipedia.org/wiki/Пиаф,_Эдит Автор: Георгий Коколия (Москва) Вопрос 23: Одна недавняя публикация журнала "Esquire" была посвящена жизни эквадорцев, переполненной страданиями, на фоне обширного разворачивания нефтяной индустрии в стране. Заголовок этой публикации представляет собой пословицу из четырех слов, одно из которых изменено на сложное слово с корнем "нефть". Назовите этот заголовок. Ответ: "Долгие нефтепроводы - лишние слезы". Источник: http://esquire.ru/photo/pipeline Автор: Наталья Филатова (Москва) Вопрос 24: (pic: 20140045.jpg) На рекламном плакате одного учреждения изображены набравшие вес некоторые выдающиеся личности. Чем занимается это учреждение? Ответ: Валютно-обменные операции. Зачет: Обмен валюты, банковские операции, банковская деятельность, инвестиционная деятельность и подобные виды деятельности, прямо связанные с увеличением прибыли клиентов от финансовых операций. Комментарий: Рекламный плакат намекает клиентам, что обменять валюту и совершить иные денежные операции здесь можно на наиболее выгодных условиях, значительно улучшив свое финансовое состояние. Источник: Рекламный плакат, увиденный автором вопроса в Таиланде. Автор: Евгений Тиньков (Москва) Тур: 10 тур. "Пацаны ваще котята" Вопрос 1: Когда участники команды "Пацаны ваще котята" узнали, что должны предоставить на Окский марафон свой пакет вопросов, прозвучала фраза "ИКС дефис ИГРЕК". А вот при составлении и редактировании нашего пакета нередко звучала фраза "ИГРЕК тире ИКС!". В одностишие Владимира Вишневского фразу "ИКС дефис ИГРЕК" получает в ответ на свой риторический вопрос литературный герой. Мы просим вас назвать только ИГРЕК. Ответ: Вопрос. Комментарий: Команда "Пацаны ваще котята" надеется, что наш пакет придется вам по вкусу. Источник: http://www.vishnevskii.ru/tx/27/ Автор: Варвара Секова Вопрос 2: В известной байке пассажир поезда Киев - Одесса спрашивает у проводника, какая бригада работает в поезде: киевская или одесская? В свою очередь проводник ДЕЛАЕТ ЭТО. Какие четыре слова мы заменили на "СДЕЛАТЬ ЭТО", если некоторые из вас недавно СДЕЛАЛИ ЭТО? Ответ: Ответить вопросом на вопрос. Комментарий: Проводник в свою очередь спрашивает: "А шо, у вас уже что-нибудь украли?", то есть отвечает вопросом на вопрос. Ответом на предыдущий вопрос было слово "вопрос". Одесская тематика вопроса дает намек. Источник: http://www.anekdot.ru/id/-9994887/ Автор: Варвара Секова Вопрос 3: В вопросе есть замены. Автора вопроса спросили: чем между собой различаются субтропическая, умеренная и субарктическая? Автор вопроса не без иронии ответил: "В субарктической самая высокая температура при самой низкой влажности; в субтропической - сравнительно низкая температура и самая высокая влажность, А умеренная - между субтропической и субарктической." Какие слова мы заменили на "субарктическая", "субтропическая" и "умеренная"? Ответ: Турецкая, финская и русская. Зачет: Хаммам, сауна, баня. Комментарий: Слова "субарктическая", "субтропическая" и "умеренная" косвенно намекают на географическое положение стран. Слово "ирония" в тексте вопроса намекает на банную тематику. Источник: 1. http://ru.wikipedia.org/wiki/Хаммам 2. http://ru.wikipedia.org/wiki/Сауна 3. http://ru.wikipedia.org/wiki/Баня#.D0.A0.D1.83.D1.81.D1.81.D0.BA.D0.B0.D1.8F_.D0.B1.D0.B0.D0.BD.D1.8F Автор: Варвара Секова Вопрос 4: Каждый, кто внимательно изучил школьный курс зоологии, знает наверняка, что уж кто-кто, а ТАКОЙ ОН - точно не существует, так как не сможет продеть верхние конечности в рукава из-за отсутствия ключиц. А по словам Абсурдопедии, ЭДАКИЙ ОН не испытывает трения о воздух, имеет абсолютно черное тело, дышит идеальным газом. Назовите ЭДАКОГО ЕГО как можно более точно. Ответ: Сферический конь в вакууме. Комментарий: Команда "Пацаны ваще котята" поздравляет знатаков с наступившим годом лошади. Источник: 1. http://www.horse.ru/oloshadi/structure.php?&cur=4632 2. http://absurdopedia.wikia.com/wiki/Сферический_конь_в_вакууме Автор: Варвара Секова Вопрос 5: <раздатка> Niko Belic, 35 years old. В одном из эпизодов игры GTA 4 главный герой отвечает на вопросы другого персонажа (в частности, называет свое имя и возраст). На просьбу "Опишите себя двумя словами" главгерой говорит по-русски: "ИКС; ИГРЕК". Спрашивающий, видимо, перепутав ИГРЕК с существительным, интересуется: "Так Вы [ПРОПУСК]?". Мы не спрашиваем, что мы заменили словом "ПРОПУСК". Ответьте, какое географическое название упоминается в следующем предложении. Ответ: СССР. Комментарий: ИКС - серб, ИГРЕК - молод. Диалог: - Опишите себя двумя словами. - Серб; молод. <усмехается> - Эээ... Так вы КОММУНИСТ? - Нет. Но полжизни прожил в СССР. Источник: Компьютерная игра "Grand Theft Auto 4". Автор: Сергей Вознесенский Вопрос 6: Герои одной онлайновой игры исследуют башню. Проходя по башне, они находят сцену и становятся невольными участниками одного из трех спектаклей. В первом спектакле приходится удивиться огромным зубам бабушки, во втором - убедиться, что соломенное пугало горит, а стальной боец ржавеет. Несчастных актеров третьего спектакля героям нужно уничтожить практически одновременно, иначе один будет воскрешать другого. Мы не спрашиваем вас имена тех, кого играют актеры в третьем спектакле. Назовите город, в котором разворачивается сюжет этого спектакля. Ответ: Верона. Комментарий: Речь идет о "World of Warcraft" и местности под названием "КаражАн". Проходя задание, герои натыкаются на сцену, где монстры играют спектакли "Красная шапочка", "Волшебник страны Оз" и "Ромео и Джульетта". Уроженцы Вероны, Ромео и Джульетта, должны умереть одновременно. Источник: http://www.igromania.ru/articles/56104/Analitika_Karazhan_samyi_populyarnyi_instans_World_of_Warcraft.htm Автор: Александр Плотников, Алексей Тютин Вопрос 7: В одной стратегической игре есть огромное древо технологий. После изучения технологии игрок может услышать подобранную к этой технологии цитату. "Бог из машины" - для машиностроения, "Сложные проценты - это самая могущественная сила во Вселенной" - для экономики, "Мы сделаем электричество таким дешевым, что жечь свечи будут только богачи" - соответственно, для электричества. А есть и такая цитата: "Если сияние тысячи солнц вспыхнуло бы в небе, это было бы подобно блеску Всемогущего... Я - Смерть, Разрушитель Миров". Мы не просим вас назвать ни технологию, ни человека, который произнес эту фразу. Назовите двумя словами кодовое название программы, которой руководил сказавший эту фразу человек. Ответ: Манхэттенский проект. Зачет: Проект Манхэттен, Manhattan Project. Комментарий: Древо технологий и подобранные к нему цитаты относятся к игре "Civilization IV". Фраза о сиянии тысячи солнц и разрушителе миров подобрана к технологии "расщепление урана" и принадлежит Роберту Оппенгеймеру, руководителю Манхэттенского проекта - на испытаниях атомной бомбы Тринити в июле 1945 года он произносит эту фразу, цитируя "Махабхарату". Источник: 1. http://ru.wikipedia.org/wiki/Sid_Meier's_Civilization_IV 2. http://ru.wikipedia.org/wiki/Манхэттенский_проект 3. http://ru.wikipedia.org/wiki/Оппенгеймер,_Роберт Автор: Александр Плотников Вопрос 8: В одной из миссий современной стратегии "Warfare" можно вызвать определенную поддержку, представители которой оперативно заберут убитых и переправят их с поля боя на базу. Какую мелодию напевает один из представителей этой поддержки? Ответ: "Полет валькирий". Комментарий: Валькирии в древнескандинавской мифологии занимались переправкой павших воинов в рай. А представляют поддержку транспортно-боевые вертолеты Ми-24. Новому времени - новые валькирии. :-) Источник: Стратегия "Warfare", 2008. Автор: Сергей Вознесенский Вопрос 9: Когда трижды Героя Социалистического труда Якова Борисовича Зельдовича избрали академиком, в Арзамасе-16 на банкете по случаю этого события Зельдовичу подарили черную академическую шапочку и плавки. На шапочке была надпись "Академия наук СССР". А какие два слова были написаны на плавках? Ответ: Действительный член. Источник: http://www.chemistry-chemists.com/N2_2012/H2/ChemistryAndChemists_2_2012-H2.html Автор: Варвара Секова Вопрос 10: В песне группы "Кожаный олень" американский президент зовет свою любовницу поиграть в НЕГО. А какой ОН в названии фильма 1999 года? Ответ: Зеленый. Источник: 1. "Кожаный олень", песня "Моника". 2. Фильм "Зеленый слоник". Автор: Андрей Алдашев Вопрос 11: Профессор-искусствовед А. Липков углубился в изучение некоторых подробностей культуры быта средневековой Европы и был, надо сказать, шокирован некоторыми из них. По итогам изучения вопроса он издал первый в России тематический труд "ИКС к ИГРЕКУ". Воспроизведите название, если авторы вопроса считают, что ИКС является наиболее характерным местом ИГРЕКА. Ответ: "Толчок к размышлению". Источник: http://modernlib.ru/books/lipkov_aleksandr/tolchok_k_razmishleniyu/read_1/ Автор: Варвара Секова Вопрос 12: В одной из серий мультсериала "Южный парк" санитарный инспектор проверяет воду аквапарка на предмет наличия в ней мочи. В своем отчете инспектор заявляет: "Мы измерили уровень пи аш воды аквапарка. Так вот...". Закончите предложение. Ответ: "... есть только пи, аш отсутствует". Зачет: По смыслу. Источник: Мультсериал "Южный парк", s14e13. Автор: Варвара Секова Вопрос 13: На одном из развлекательных сайтов было опубликовано изображение мобильного телефона, в каждый из разъемов которого вставлен штекер с проводом. Напишите имя женщины, фото которой установлено на дисплее телефона. Ответ: Саша Грей. Источник: (pic: 20140046.jpg) Автор: Варвара Секова, Сергей Вознесенский Вопрос 14: Фильм "Ловля лосося в Йемене" вышел в российском прокате под названием, представляющим собой популярное некоторое время назад выражение, хотя ИКС относится к другому семейству. Какое слово мы заменили на ИКС? Ответ: Язь. Комментарий: "Рыба моей мечты". Источник: http://www.kinopoisk.ru/film/484559/ Автор: Андрей Алдашев Вопрос 15: Один из популярных в Рунете художников изображает известного героя русской литературы XIX века, интерпретируя слово, заимствованное из английского языка, разбитое на две части. Вопреки вольной интерпретации художника, в состав этого слова входят целых два животных. Одно из них относится к отряду парнокопытных, а другое к отряду хищных. Напишите имя литературного героя. Ответ: Герасим. Комментарий: Изображение Герасима приводится в интерпретации слова "бульдог" (буль дог). Этимологически слово "Бульдог" происходит от английского "Bull dog" - бычья собака. Источник: 1. http://kandelyabr.livejournal.com/31585.html 2. http://ru.wikipedia.org/wiki/Быки 3. http://ru.wikipedia.org/wiki/Собака Автор: Варвара Секова Вопрос 16: (pic: 20140047.jpg) Доцент Казанского университета Евгений Султанов в своей статье о встрече президента с ведущими преподавателями правовых дисциплин делает акцент на том, что Владимир Путин в ходе встречи высказал ряд радикальных идей по развитию государственного устройства и совершенствованию Конституции. Заглавие этой статьи звучит как "Владимир Путин ДЕЛАЕТ ЭТО". Глядя на раздаточный материал, тоже можно сказать, что Владимир Путин ДЕЛАЕТ ЭТО. Какие два слова мы заменили на "ДЕЛАЕТ ЭТО"? Ответ: Опережает время. Источник: http://www.kpfu.ru/main_page?p_cid=62174&p_sub=220 Автор: Варвара Секова, Сергей Вознесенский Вопрос 17: Летом 1938 года сопки Заозерная и Безымянная были отбиты советскими войсками у японцев. Однако после 16 января 2013 года можно было в шутку предположить появление авторитетного мнения, что слово "Героев" и названии пермской улицы стоит заменить другим. Каким? Ответ: Деда. Комментарий: Вор в законе Аслан Усоян, известный под кличкой Дед Хасан, погиб в январе 2013 года. Улица в Перми была названа в честь героев конфликта у озера Хасан. Источник: 1. http://ru.wikipedia.org/wiki/Хасанские_бои_(1938) 2. http://ru.wikipedia.org/wiki/Усоян,_Аслан_Рашидович Автор: Андрей Алдашев Вопрос 18: <раздатка> Прототип известного персонажа любил расправляться с врагами с помощью ИКСА. А с помощью какого именно ИКСА принято расправляться с сородичами этого персонажа? Если бы розданный вам вопрос попал в пакет, то это был бы [ПРОПУСК 1], потому что в Базе уже есть несколько вопросов [ПРОПУСК 2]. Заполните любой из неотличимых на слух пропусков. Ответ: Прокол. Зачет: Про кол. Источник: Поиск по Базе Вопросов ЧГК по словам "КОЛ", "ВАМПИР", "ДРАКУЛА", "ВЛАД ЦЕПЕШ". Автор: Андрей Алдашев Вопрос 19: Знакомый автора вопроса, вернувшийся в университет после армии, поинтересовался, что ему нужно сделать в первую очередь. На что получил ответ: "В первую очередь [ПРОПУСК], а потом зайдите к нам". Восстановите пропуск. Ответ: "... восстановите пропуск...". Источник: Личный опыт автора вопроса. Автор: Сергей Вознесенский Вопрос 20: <раздатка> Ты скажешь: "Кому это нужно? Ведь можно найти любую Без шансов на мелодрамы...". А я еще молодцом, и не видно Снаружи не острую и не тупую От этой сердечной раны...   Я сделаю всё, как надо, Сломай меня, если хочешь, Без страха и без сомнения... Ты будешь дышать свободно, Я рада, спасибо за этот подарок, Он лучший на День Рождения! "Меня простите, Бога ради, Я думал, здесь собрались [п а у з а] девы, Я вам спою [ПРОПУСК] Из этой тоненькой тетради". Можно сказать, что на розданном вам материале тоже приведены [ПРОПУСК]. Восстановите пропуск. Ответ: Куплеты Евы. Комментарий: Куплеты Евы - иронические куплеты, исполняемые под гитару, характеризующиеся тем, что слова в конце рифмуемых строк не соответствуют ожидаемым: во второй строке после паузы предполагается нецензурная рифма к первой строке, однако вместо нее следует другое слово (получается рифмовка ABAB или ABBA). В раздаточном материале приведены куплеты из песни Евы Польны "Я тебя тоже нет". Источник: 1. http://ru.wikipedia.org/wiki/Куплеты_Евы 2. http://www.sentido.ru/songs.php?id_song=3741 3. http://webkind.ru/text/52810130_01090179p922043029_text_pesni_kuplety-evy-menya-prostite-boga-radi-ya-dumal-zdes.html Автор: Александр Плотников, Сергей Вознесенский, Варвара Секова Вопрос 21: В романе Владислава Крапивина "Кораблики" упоминается вымышленный город с необычным десятибуквенным названием. Коренные горожане ставят ударение на второй слог, намекая на растущие повсюду многолетние деревья. Залётные же гости делают акцент на первом слоге, упоминая находящийся неподалеку космодром. А еще название города всего двумя буквами отличается от названия одного из южных городов России. Восстановите название города. Ответ: Стартополь. Источник: В. Крапивин. Кораблики. Автор: Сергей Вознесенский Вопрос 22: Обсуждая книгу Лиона Наделя и Нинель Шаховой "Мыло - голым!", пользователь Феликс Рахлин утверждает, что это - [ПРОПУСК], и НИ МОРД, НИ ЛАП у него быть не может. Восстановите пропуск. Ответ: "... палиндром...". Комментарий: "Ни морд ни лап" - анаграмма слова "палиндром". Источник: http://www.proza.ru/2011/12/05/148/ Автор: Сергей Вознесенский Вопрос 23: [На столы раздаются по одному прямоугольному листку чистой бумаги.] Внимание, в вопросе есть замена. При наличии фантазии можно предположить, что перед вами - МАКЕДОНСКИЙ КРЕАТИВ. Известно, что МАКЕДОНСКИЙ писал КРЕАТИВЫ, однако автору вопроса не удалось найти ни одного МАКЕДОНСКОГО КРЕАТИВА, принадлежащего МАКЕДОНСКОМУ. Что мы заменили словами "МАКЕДОНСКИЙ КРЕАТИВ"? Ответ: Белый стих. Комментарий: Как известно, Андрей Белый писал стихи, однако автор вопроса не нашел ни одного белого стиха, принадлежащего Андрею Белому. Автор: Сергей Вознесенский Вопрос 24: Внимание, в вопросе есть замены. Героиня Натальи Солнцевой заявила своему любовнику, не желающему жениться на ней, что готовится ДЕЛАТЬ ПЕРВОЕ, и пыталась ДЕЛАТЬ ВТОРОЕ, демонстрируя компрометирующие его фотографии. Напишите буквы, на которые слово, замененное на "ДЕЛАТЬ ПЕРВОЕ", короче, чем слово, замененное на "ДЕЛАТЬ ВТОРОЕ". Ответ: УГ. Комментарий: Очень она хотела замуж, поэтому отказывалась делать аборт и наняла частного детектива. Ничем хорошим для нее это не закончилось. Надеемся, что ваша оценка нашего пакета не совпадет с ответом. Источник: Наталья Солнцева, Магия венецианского стекла. http://www.loveread.ec/read_book.php?id=6134&p=15 Автор: Андрей Алдашев Тур: 11 тур. "Пингвины" (Архангельск) Вопрос 1: Дуплет. 1. С середины декабря 2013 года на рекламных билбордах города появилось обращение к горожанам от информационно-новостного портала, где в стандартное предложение была добавлена буква. Воспроизведите измененное предложение. 2. С середины декабря 2013 года на рекламных билбордах города появилось обращение к горожанам от компании, занимающейся производством бытовых фильтров воды, где в стандартное предложение был добавлен символ. Воспроизведите измененное предложение. Ответ: 1. Happy News Year. 2. С Н2Овым годом. Комментарий: Наша команда поздравляет всех с наступившими новогодними праздниками и желает хорошего отыгрыша нашего пакета (бла-бла-бла). Источник: ЛОАВ. Автор: Александра Головина (Архангельск) Вопрос 2: Город Кочабамба, где вырос Марио Варгас Льоса, не [ПРОПУСК 1]. Роман "Город и псы" о военном училище в Лиме [ПРОПУСК 2] Марио Варгас Льосы. На слух пропуски, состоящие из двух слов, совершенно не отличаются друг от друга. Восстановите любой из пропусков. Ответ: Принадлежит Перу. Зачет: Принадлежит перу. Источник: 1. http://ru.wikipedia.org/wiki/Варгас_Льоса,_Марио 2. http://ru.wikipedia.org/wiki/Кочабамба Автор: Денис Никитенко (Архангельск - Москва) Вопрос 3: Одна из статей сайта openleft.ru рассказывает об Обелиске в Александровском саду, на котором изначально были выбиты имена царей и императоров из дома Романовых, а затем, в 1918 году, эти имена были сбиты и на их место помещен список 19 социальных мыслителей и политических деятелей. Интересно, что совсем недавно Обелиску вернули первоначальный вид, сбив теперь уже имена мыслителей-революционеров и нанеся имена Романовых. "Но надо признать, - пишет А. Новоженова, - что памятник-КУЛЬТУРА - вещь обычная". Какое слово греческого происхождения мы заменили словом "КУЛЬТУРА"? Какое слово греческого происхождения мы заменили словом "КУЛЬТУРА"? Ответ: Палимпсест. Комментарий: Палимпсест - рукопись на пергаменте (реже папирусе) поверх смытого или соскобленного текста. А как известно из творчества Псоя Галактионовича: "Культура - это палимпсест <...> припев два раза, припев два раза". Источник: 1. http://www.openleft.ru/?p=423 2. Псой Короленко "Припев два раза". 3. http://ru.wikipedia.org/wiki/Палимпсест Автор: Александр Цыганов (Архангельск) Вопрос 4: Африканский пастор Френк Кабеле в 2006 году объявил, что сумеет явить чудо прихожанам - сможет пересечь пешком устье реки Комо, путь, который занимает как минимум 20 минут на катере. Так, он стал первым в истории священником, который СДЕЛАЛ ЭТО. Сомнительное достижение, ничего не скажешь. Ответьте с упоминанием имени собственного: СДЕЛАЛ ЧТО? Ответ: Получил премию Дарвина. Источник: http://www.d-awards.ru/745-premiya-darvina-2006-rukopolozhenie-obyazyvaet.html Автор: Григорий Теплых (Архангельск) Вопрос 5: Во время одной из своих лекций профессор Соловьев раскритиковал Мао за то, что тот сделал вершину сходящейся. В одном из слов предыдущего предложения мы пропустили три буквы. Напишите это слово в первоначальном варианте. Ответ: Маслоу. Комментарий: Потребности все-таки безграничны. Источник: ЛОАВ. Автор: Алевтина Некипелова (Архангельск) Вопрос 6: Судя по всему, лондонский "Челси" никогда не был главным соперником ни, например, ташкентского "Пахтакора", ни "Нефтчи" из Ферганы. Однако узбекские производители ЭТОГО всё же неуважительно разместили эмблему "Челси" на упаковке своего продукта. А по данным "ЦБК экспресс", в 2013 году в России было произведено чуть менее 19 единиц ЭТОГО на человека. Назовите ЭТО. Ответ: Туалетная бумага. Источник: 1. http://www.isport.ua/football/europe/news/280721.html 2. http://ru.wikipedia.org/wiki/Туалетная_бумага 3. "ЦБК экспресс", 2013, N 4 (512). - С. 2. Автор: ??? Вопрос 7: Продолжим футбольную тематику. Этот восьми- или десятикратный чемпион страны по футболу имеет на теле множество татуировок. Среди них, например, красный дракон, два туза, надпись "Only god can judge me" (Только Бог может судить меня). Остальные - примерно восемь штук, - по мнению автора вопроса, образуют ЕЕ. Назовите ЕЕ устойчивым выражением из двух слов. Ответ: Шведская семья. Комментарий: Он - Златан Ибрагимович, шведский футболист. Остальные его татуировки так или иначе посвящены ему самому и членам его семьи: его сыновьям, братьям и сестрам, родителям. А вот восемь или все-таки десять раз он был чемпионом - зависит от того, за "Интер" вы болеете или за "Ювентус". :-) Ведь именно "Интеру" были отданы два титула "Ювентуса" в результате Кальчополи. Источник: http://ru.wikipedia.org/wiki/Ибрагимович,_Златан#.D0.A2.D0.B0.D1.82.D1.83.D0.B8.D1.80.D0.BE.D0.B2.D0.BA.D0.B8 Автор: Владимир Ермолин (Архангельск) Вопрос 8: Статья на сайте Лента.ру, рассказывающая о неудачных экономических последствиях легализации марихуаны в штате Колорадо, называется "Рынок ИКСОВ". Назовите ИКС. Ответ: Косяк. Комментарий: Косяк на жаргоне - не только папироса с марихуаной, но и обозначение проступка или оплошности. Источник: 1. http://lenta.ru/articles/2014/01/15/weedbusiness/ 2. http://ru.wikipedia.org/wiki/Косяк Автор: Алексей Амосов (Архангельск) Вопрос 9: Огорченный лирический герой одной песни в день рождения возлюбленной планирует совершить некое действие под ее окном. Это же действие совершил известный человек в первом веке н.э. Другой поступок этого человека нашел свое отражение в текстах песен таких групп, как "Metallica" и "Stratovarius". Назовите этот поступок, упомянутый в песнях, одним словом. Ответ: Поцелуй. Комментарий: Лирический герой, как и Иуда в свое время, хотел повеситься. А ранее, о чем и поется в песнях, Иуда свои поцелуем предал Иисуса. Источник: 1. "Сатана печет блины", песня "Сегодня я повешусь". 2. "Metallica", песня "The Judas Kiss". 3. "Stratovarius", песня "The Kiss of Judas". Автор: Алексей Кузнецов (Архангельск) Вопрос 10: Как известно, пиар-менеджеры иногда преподносят сюрпризы. Так, на одном плакате рядом с НИМИ изображены родители. Мы не просим назвать ИХ. Назовите изображенных родителей, постараясь не спорить о правильной последовательности. Ответ: Курица и петух. Комментарий: Таким образом рекламируется шоколадное яйцо - Kinder Surprise. "Преподносят сюрпризы" - подсказка, как и "не спорить о последовательности" - отсылка к спору о том, что было раньше: курица или яйцо. Источник: http://pikabu.ru/story/kinder_syurpriz_1153467 Автор: Алевтина Некипелова (Архангельск) Вопрос 11: В вопросе есть замены. 21 апреля 1912 года некто ИКС написал директору Британского музея: "Сэр! Обращаюсь к Вам с просьбой о выдаче мне билета на право выхода в читальный зал Британского музея. Я прибыл из России для изучения КВАРТИРНОГО вопроса". Мы не спрашиваем вас, какое слово в тексте мы заменили словом "КВАРТИРНОГО". Назовите человека, скрывавшегося под псевдонимом ИКС. Ответ: В.И. Ленин. Комментарий: ИКС - Якоб Рихтер: именно под этим псевдонимом в Лондон прибыл В.И. Ленин для изучения аграрного вопроса. Источник: В. Ерофеев. Москва - Петушки (с комментариями Э. Власова). Автор: Павел Столяров (Архангельск) Вопрос 12: Надеемся, этот вопрос вызовет восторженные отклики. Существует мнение, что это междометие появилось в Средние века и происходит от выражения "Hierusalem est perdita" - "Иерусалим разрушен". Другие исследователи полагают, что его история уходит корнями во времена древних греков и посвящалось оно богине. Воспроизведите его максимально точно. Ответ: "Гип-гип-ура!". Комментарий: Исследователи полагают, что "hep-hep!" - боевой клич крестоносцев. Другие ученые относят это междометие к словам "Hyper Urania" (греч. Ourania) - "Небесная", одно из имен, эпитетов Афродиты. А мы радостными восклицаниями отмечаем экватор нашего пакета. Источник: 1. http://traditio-ru.org/wiki/Гип-гип 2. http://ru.wikipedia.org/wiki/Гип-гип_ура! Автор: ??? Вопрос 13: Когда у Евгения Ловчева спросили, кто бы мог получить Золотой мяч 2013 года от России, он в присущей ему ироничной манере ответил, используя название бального танца. А теперь и вы опробуйте воспроизвести ответ язвительного Ловчева. Ответ: Ирина Шейк. Комментарий: Российская модель, подруга португальского футболиста Криштиану Роналду. Источник: 1. http://www.sovsport.ru/news/text-item/675266 2. http://dic.academic.ru/dic.nsf/enc3p/328142/ 3. http://ru.wikipedia.org/wiki/Шейк,_Ирина_Валерьевна Автор: Владимир Ермолин (Архангельск) Вопрос 14: Автор вопроса поздравляет участников марафона с приближающимся днем Святого Валентина и искренне надеется, что всем вам есть, кого любить. Внимание, дуплет! 1. Умирая, бывший президент США Джон Адамс с горечью констатировал, что ЭТОТ ЧЕЛОВЕК еще жив. Это оказалось неправдой: он умер в тот же день, но на четыре часа раньше. Назовите фамилию этого человека. 2. Напишите состоящее из одного слова название специального счета "Альфа-банка", позволяющего копить мили в программе "Аэрофлот Бонус". Ответ: 1. Джефферсон. 2. Аэроплан. Комментарий: "Somebody to love" - главный хит американской рок-группы "Jefferson Airplane". Источник: 1. http://ru.wikipedia.org/wiki/Адамс,_Джон_(президент) 2. http://www.alfabank.ru/press/news/2013/3/1/30420.html 3. http://ru.wikipedia.org/wiki/Jefferson_Airplane Автор: Алексей Кузнецов (Архангельск), Денис Никитенко (Архангельск - Москва) Вопрос 15: Герой современной молодежной комедии, глядя в камеру, говорит: "Сейчас вы увидите самую безбашенную и крутейшую жесть на свете". В одной из последующих сцен он и его друг ДЕЛАЮТ ЭТО, а чуть позже удивляются увиденным цифрам. Ответьте максимально точно, что мы заменили в тексте вопроса на "ДЕЛАЮТ ЭТО". Ответ: Выкладывают видео на Youtube. Зачет: Заливают видео на Youtube; выкладывают видео в Интернет. Источник: Фильм "Movie 43". Автор: Алексей Кузнецов (Архангельск) Вопрос 16: Отрывок из романа Иэна Макьюэна "Искупление": "... Но мне казалось, что в ее облике есть что-то от сценической злодейки - костлявая фигура, темная шуба, кроваво-красный рот. Ей бы еще сигарету в длинном мундштуке и комнатную собачку под мышку - получилась бы ни дать ни взять...". Заканчивается фраза именем героини известного мультфильма. Назовите этот мультфильм. Ответ: "101 далматинец". Комментарий: Круэлла де Виль. Источник: 1. Иэн Макьюэн. Искупление. 2. Мультфильм "101 далматинец". Автор: Ирина Аксенова (Архангельск) Вопрос 17: В седьмом эпизоде мультсериала "Мой маленький пони" одна из героинь говорит о застенчивой и пугливой пони Флаттершай, что та боится даже АЛЬФЫ. В последующей сцене Флаттершай, в подтверждение этих слов, увидев АЛЬФУ, в страхе резво бьет по земле копытами и прыгает в кусты. Назовите АЛЬФУ двумя словами. Ответ: Собственная тень. Зачет: Своя тень. Комментарий: Аллюзия на Буцефала, любимого коня Александра Македонского. Источник: "Мой маленький пони", s01e07. Автор: Алексей Кузнецов (Архангельск) Вопрос 18: Электронная книга Амазон Киндл позволяет получить бесплатный доступ к сайтам Википедии, включая, например, саму энциклопедию или проект Вики-Тревел, практически из любого города мира, что позволило авторам комикса xkcd сравнить ее с другой книгой. А что написано у той книги на обложке? Ответ: "Don't panic". Зачет: "Не паниковать", "Справочник путешествующего автостопом по галактике" и т.п. по смыслу. Источник: 1. http://ru-xkcd.livejournal.com/78063.html 2. http://xkcd.com/548/ Автор: Александр Цыганов (Архангельск) Вопрос 19: Внимание, в вопросе есть замены. "КОЛЕБРИ" - название южноамериканской футбольной команды. КОЛИБРИ чудесным образом тоже имеет отношение к спорту. Так, в одном литературном произведении XIX века КОЛИБРИ используются в некой спортивной игре. Назовите эту игру. Ответ: Крокет. Незачет: Крикет. Комментарий: Заменены слова "Фламенго" и "фламинго". В "Алисе в Стране чудес" описывается игра в крокет, где в роли клюшек выступали фламинго. Источник: 1. http://ru.wikipedia.org/wiki/Фламенго 2. http://lib.ru/CARROLL/alisa_zah.txt Автор: Алексей Кузнецов (Архангельск) Вопрос 20: В современном комиксе "Малость подавленный парень" для героя, ожидающего звонка возлюбленной, ИКСЫ на линиях электропередач звучали, словно рингтоны мобильника. Роман, в заглавии которого упоминается ИКС, получил Пулитцеровскую премию в 1961 году. Назовите автора этого романа. Ответ: Харпер Ли. Источник: 1. Little deppresed boy, том 0, стр. 43. 2. http://ru.wikipedia.org/wiki/Пулитцеровская_премия_за_художественную_книгу Автор: ??? Вопрос 21: Коронный прием рестлера Эйджа, в ходе выполнения которого тот разбегается и наносит сбивающий с ног удар плечом в корпус противника, называется "ИКС". Впервые ИКС начали использовать в эпоху палеолита. Назовите ИКС словом голландского происхождения. Ответ: Гарпун. Источник: 1. http://www.youtube.com/watch?v=_KTg5lzdzJ8 2. http://ru.wikipedia.org/wiki/Гарпун Автор: Алексей Кузнецов (Архангельск) Вопрос 22: На данный момент этот показатель равен пяти. В 1953 году зафиксирован минимальный показатель - две, а в 1991 году максимальный - семнадцать. А всего лишь через одну минуту Вам следует ответить: что гипотетически произойдет, если этот показатель будет равен нулю? Ответ: Ядерная катастрофа. Зачет: Ядерная/атомная война, ядерный/атомный катаклизм и т.п. по смыслу. Комментарий: Они - Часы Судного дня: проект журнала Чикагского университета "Bulletin of Atomic Scientists", демонстрирующий уровень международной напряженности в зависимости от прогресса развития ядерного вооружения. Указанные значения - время до полуночи. Источник: http://ru.wikipedia.org/wiki/Часы_Судного_дня Автор: Александра Головина и Владимир Ермолин (Архангельск) Вопрос 23: Американский промышленник Джордж Вестингауз прокомментировал первое использование ИКСА следующими словами: "Лучше бы они воспользовались топором". ИКС изображен на обложке альбома "Ride The Lightning" группы "Metallica". Назовите ИКС двумя словами. Ответ: Электрический стул. Комментарий: Выражение "Ride The Lightning" (оседлать молнию) означает "быть казненным на электрическом стуле". Надеемся, ваши стулья к концу нашего пакета кажутся вам всё такими же удобными, как и в начале. Источник: http://ru.wikipedia.org/wiki/Электрический_стул Автор: Алексей Кузнецов (Архангельск) Вопрос 24: По мнению многих, в игре "Fallout: Tactics" самая жестокая формулировка "ЭТОГО": "Кровь хлещет из каждой Вашей раны, пока Вы осознаёте, что умрете девственником". Назовите россиянку, прозвищем которой является ЭТО. Ответ: [Екатерина] Гамова. Комментарий: ЭТО - фраза "game over", а наша игра тоже подошла к концу. :-) Источник: 1. http://joyreactor.cc/post/1124672 2. http://ru.wikipedia.org/wiki/Гамова,_Екатерина_Александровна Автор: Алексей Амосов (Архангельск) Тур: 12 тур. "Зигдиан+" (Серпухов - Ярославль) Вопрос 1: <раздатка> Буги-вуги Юрий Клинских определял жанр ряда песен группы "Сектор Газа" как "колхозный панк-рок". Какие две буквы мы заменили на одну в названии розданного вам альбома? Ответ: Пл. Комментарий: "Плуги-вуги". Куда уж колхозней? Источник: 1. http://otvet.mail.ru/question/42425788 2. http://ru.wikipedia.org/wiki/Плуги-вуги Автор: Дмитрий Сахаров (Тутаев) Вопрос 2: В одной легенде рассказывается о крестьянине, который в жаркий день прилег отдохнуть под оливу вместе со своим питомцем. На территории какой современной страны происходит действие этой легенды? Ответ: Хорватия. Комментарий: У крестьянина был белый пес, который в итоге загорел пятнами в форме оливок. Так объясняется появление породы далматинов. Далмация - историческая область в составе Хорватии. Источник: "Вокруг света", 2013, N 9. - С. 78. Автор: Дмитрий Сахаров (Тутаев) Вопрос 3: В разделе одной статьи Википедии рассказывается о победе Ким Клейстерс на US Open в 2009 году, одержанной вскоре после рождения дочери. Назовите футбольную команду, которая упоминается в том же разделе. Ответ: [Мужская] сборная Дании. Комментарий: Клейстерс не хватало очков рейтинга, чтобы принять участие в таком престижном турнире. Поэтому организаторы персонально пригласили ее. В 1992 году сборная Дании также получила своего рода Wild Card и также триумфально выиграла турнир. Источник: http://ru.wikipedia.org/wiki/Уайлд-кард Автор: Дмитрий Сахаров (Тутаев) Вопрос 4: В статье, вышедшей в начале декабря, сообщается, что "племена бавенда <...> обычно просто оставляют умерших в их домах, пока их не высушит солнце и горячий воздух, а насекомые не закончат начатое". Далее в тексте упоминается возможность возведения ЕГО. Назовите ЕГО двумя словами, начинающимися на одну и ту же букву. Ответ: Мавзолей Манделы. Комментарий: Между смертью и погребением Манделы прошло достаточно времени - больше десяти дней. За это время появилось достаточно много версий насчет того, как будет погребено тело. Источник: http://www.sports.ru/tribuna/blogs/bazdrevboxing/539480.html Автор: Дмитрий Сахаров (Тутаев) Вопрос 5: В антиутопии Курта Воннегута, действие которого происходит в конце XXI века, 212-я ОНА предписывает красивым носить маски. Назовите ЕЕ максимально точно. Ответ: Поправка к конституции США. Зачет: По смыслу. Комментарий: В рассказе "Гаррисон Бержерон" 2081 году в США всеми силами устанавливается общество равных возможностей. Источник: http://domino-books.narod.ru/lib/vonnegut_harrison_bergeron.htm Автор: Дмитрий Сахаров (Тутаев) Вопрос 6: В 2012 году ОН, с состоянием в 62 миллиарда долларов, возглавил список "Forbes Fictional". Надеемся, что вы сможете назвать ЕГО имя. Ответ: [Дракон] Смог. Источник: http://ru.wikipedia.org/wiki/15_богатейших_вымышленных_персонажей_по_версии_Forbes Автор: Дмитрий Юрлов (Астрахань), в редакции Дмитрия Сахарова (Тутаев) Вопрос 7: Внимание, в вопросе есть замены. Пауло Коэльо утверждает, что в мире нет ничего совершенно ненужного или ошибочного, даже сломанные ОНИ ДЕЛАЮТ ЭТО. Через минуту как можно точнее ответьте, что мы заменили на "ДЕЛАЮТ ЭТО". Ответ: Два раза в день показывают точное время. Источник: http://www.zemlemer-67.ru/news/5265-paulo-koelo Автор: Тимур Муратов (Астрахань) Вопрос 8: В экранизации 1994 года главный герой несколько раз подряд остается на второй год за отличную успеваемость. Назовите автора этого экранизированного произведения. Ответ: [Курт] Воннегут. Комментарий: Как вы знаете из пятого вопроса... Источник: К/ф "Гаррисон Бержерон". Автор: Дмитрий Сахаров (Тутаев) Вопрос 9: Геннадий Орлов во время трансляции из Порту заявил: "Дожди всегда были АЛЬФОЙ". На одном региональном форуме был задан вопрос "Влияет ли на вас АЛЬФА?", отвечая на который участники разделились во мнениях. Кого-то беспокоит хандра, болезнь горла, депрессия, а другие утверждают, что АЛЬФА красива, индивидуальна и непредсказуема. Назовите АЛЬФУ двумя словами, начинающимися на одну и ту же букву. Ответ: Питерская погода. Комментарий: Геннадий Орлов - известный болельщик санкт-петербургского "Зенита". Упоминание регионального сайта должно было вас натолкнуть, что погода не плохая, а питерская. Источник: 1. Трансляция матча "Порту" - "Зенит", 22.10.2013 г. 2. http://www.spbtalk.ru/index.php?showtopic=9248 Автор: Игорь Гужвин (Астрахань) Вопрос 10: Некоторое время назад в результате взрыва погиб заместитель муфтия Ставропольского края Курман Исмаилов. Примечательно, что муфтий погиб на улице ЕГО имени. Урбанонимы, носящие ЕГО фамилию, есть также в Орле и Кисловодске. Назовите ЕГО. Ответ: [Генерал] [Алексей Петрович] Ермолов. Комментарий: Род Ермоловых происходит из Орловской губернии, а уж его связь со Ставрополем известна всем. Источник: http://lenta.ru/news/2012/02/13/muftiy/ Автор: Павел Бордачев (Серпухов) Вопрос 11: Двое российских рэперов - Змей и Жара решили на пару выпустить сборник песен. Название сборника созвучно девизу американской компании, основанной в 1964 году. Назовите эту компанию. Ответ: "Nike". Зачет: "Найк". Комментарий: Альбом называется "ЖаЗ Дуэт". Слоган компании Nike - "Just do it...". Источник: http://www.kasta.ru/releases/73-zhaz-duet Автор: Алексей Абросимов (Астрахань), в редакции Дмитрия Сахарова (Тутаев) Вопрос 12: Персонаж Стругацких, рассуждая о слабом месте в своей теории, упоминает бублик с маком вместо одного из НИХ. Назовите ИХ тремя словами. Ответ: Звенья логической цепи. Зачет: Звенья цепи логики. Комментарий: "Не просчитался, нет, пока еще никто не вправе кинуть в меня камень, но видно уже, что стальная цепь логики моей содержит одно звено не металлическое, а так, бублик с маком". Источник: http://e-libra.ru/read/318040-otyagoshennie-zlom,-ili-sorok-let-spustya.html Автор: Дмитрий Сахаров (Тутаев) Вопрос 13: Просим вас не обращать внимание на возможную неряшливость в написании вопроса. На обложке одного фильма ужасов можно увидеть ЕГО, хотя в этом фильме ведьма убивает подростков, а не наоборот. Назовите ЕГО одним словом. Ответ: Топор. Комментарий: Вопрос написан грубо, поспешно, то есть топорно; ведьма с топором - как "Преступление и наказание" наоборот. Источник: 1. http://www.kinopoisk.ru/film/363534/ 2. Ф.М. Достоевский. Преступление и наказание. Автор: Дарья Бычкова (Серпухов) Вопрос 14: Герой романа "Страдания юного Вертера" просит девушку не совершать некоего действия с письмами, адресованными ему, так как герою не нравится ДЕЛАТЬ ЭТО. В отличие от героя песни, который был твердо намерен ДЕЛАТЬ ЭТО. Какие два слова мы заменили словосочетанием "ДЕЛАТЬ ЭТО"? Ответ: Целовать песок. Комментарий: "Об одном только прошу вас: не посыпайте песком писем, адресованных мне. Сегодня я сразу же поднес записочку к губам, и у меня захрустело на зубах". Источник: 1. И.В. Гёте. Страдания юного Вертера. - М.: Государственное издательство художественной литературы, 1957. - С. 101. 2. Песня Владимира Маркина "Я готов целовать песок". Автор: Дарья Бычкова (Серпухов) Вопрос 15: Комментируя телекартинку с финишем пловца, на корпус опережающего мировой рекорд, Василий Уткин упомянул ЭТО. Можно сказать, что ЭТО находится справа от нуля. Назовите ЭТО. Ответ: [Нижнее] подчеркивание. Комментарий: Телекартинка упомянута не зря. По традиции график мирового рекорда показан движущейся белой линией. В данном случае спортсмен обогнал ее. Источник: "Олимпийское эхо", ночь с 4 на 5 августа 2012 г. Автор: Ярослав Киприанов (Ярославль) Вопрос 16: Иконка приложения "Maps" на iPhone автора вопроса представляет собой стилизованный ОН. Назовите ЕГО тремя словами, начинающимися на одну и ту же букву, одно их которых имя собственное. Ответ: Кусок карты Купертино. Зачет: Кусочек карты Купертино. Комментарий: В калифорнийском городе Купертино находится штаб-квартира Apple. Источник: iPhone автора вопроса. Автор: Павел Бордачев (Серпухов) Вопрос 17: Крупнейшими государствами, полностью лежащими [ПРОПУСК 1], являются Демократическая Республика Конго, Индонезия и Судан. [ПРОПУСК 2] была написана "Черная весна". Заполните любой из незначительно отличающихся пропусков. Ответ: Между тропиком Рака и тропиком Козерога. Зачет: Между "Тропиком Рака" и "Тропиком Козерога". Комментарий: Во втором случае имеются в виду скандально известные романы Генри Миллера, вместе с "Черной весной" составляющие автобиографическую трилогию. Источник: 1. http://ru.wikipedia.org/wiki/Северный_тропик 2. http://ru.wikipedia.org/wiki/Южный_тропик 3. http://ru.wikipedia.org/wiki/Миллер,_Генри Автор: Дмитрий Сахаров (Тутаев) Вопрос 18: Комик Энди Кауфман знаменит пародиями на Элвиса Пресли. В репертуаре какой группы есть песня "Энди Кауфман"? Ответ: "Король и Шут". Зачет: "Северный флот". Комментарий: Про Энди можно сказать, что он шут, пародировавший короля. Источник: 1. http://ru.wikipedia.org/wiki/Кауфман,_Энди 2. http://ru.wikipedia.org/wiki/Театр_демона Автор: Арсений Глазовский (Ярославль) Вопрос 19: Клуб "Ливерпуль" очень неудачно начал сезон 2012/13, поэтому появилась шутка, что новый ОН придется по душе болельщику "Ливерпуля". Назовите ЕГО. Ответ: iPhone. Зачет: Айфон. Комментарий: Экран нового iPhone был длиннее, чем у предыдущей версии. В одной из вариаций текст шутки звучит так: "С iPhone 5 Вам не понадобится листать вниз, чтобы найти "Ливерпуль" в турнирной таблице". Источник: (pic: 20140048.jpg) Автор: Арсений Глазовский (Ярославль) Вопрос 20: Герой Акунина видит незнакомую женщину. В описании этой женщины фигурируют "посверкивающий серебром мех" и "надменный требовательный взгляд". Напишите фамилию, которая также упоминается в этом описании. Ответ: Крамской. Комментарий: Цитата: "У Фандорина возникло ощущение, что он эту красавицу уже где-то видел. Быть может, на картине Крамского?". "Неизвестная" - название картины Крамского. Источник: Б. Акунин. Внеклассное чтение. Автор: Арсений Глазовский (Ярославль) Вопрос 21: Научно-популярный сайт называет ванну Архимеда "тренажером для приобретения чувства меры". Какие два слова мы заменили двумя другими? Ответ: Чаша Пифагора. Зачет: Чашка/кубок Пифагора. Комментарий: Оригинальный сосуд, изобретение которого приписывают Пифагору. Сосуд можно заполнить только небольшим количеством жидкости, попытка заполнить сосуд доверху приводит к потере всей жидкости. Источник: http://www.parakalo.ru/chasha-pifagora-kak-polzovatsya/ Автор: Дмитрий Сахаров (Тутаев) Вопрос 22: Курт Воннегут в фильме "Снова в школу" сыграл в эпизодической роли самого себя. Ученик нанимает Воннегута для определенной работы, однако дальнейшие события напоминают байку о будущей писательнице и писателе. Назовите или писательницу, или писателя. Ответ: [Дарья] Донцова. Зачет: [Валентин] Катаев. Комментарий: Герой фильма нанимает писателя для написания реферата по творчеству Курта Воннегута, однако написанный реферат не устраивает преподавателя. В школе будущей писательнице первой величины Дарье Донцовой задали написать сочинение "О чем думал Валентин Петрович Катаев, когда писал повесть "Белеет парус одинокий"". Катаев - друг семьи писательницы - сам написал сочинение, которое, тем не менее, получило неудовлетворительную оценку: комментарий учительницы гласил: "Катаев совсем не об этом думал!". Источник: 1. http://ru.wikipedia.org/wiki/Снова_в_школу 2. http://ru.wikipedia.org/wiki/Донцова,_Дарья Автор: Дмитрий Сахаров (Тутаев) Вопрос 23: Агент Малдер, перед тем как укрыться от погони в пчелином зимовнике, ДЕЛАЕТ ЭТО. Кавалер ордена Томаша Масарика, так же как Пламен Горанов, СДЕЛАЛ ЭТО перед трагическим событием. Какие три слова мы заменили на "СДЕЛАЛ ЭТО"? Ответ: Облил себя бензином. Зачет: По смыслу. Комментарий: Малдер обливается бензином, чтобы запах отпугнул пчел. Герой Чехии Ян Палах сжег себя на Вацлавской площади в Праге в знак протеста против оккупации Чехословакии войсками Варшавского договора. Пламен Горанов - болгарский общественный деятель, который 20 февраля 2013 года совершил акт самосожжения перед мэрией Варны. Источник: 1. "Секретные материалы", s04e01. 2. http://ru.wikipedia.org/wiki/Палах,_Ян 3. http://ru.wikipedia.org/wiki/Горанов,_Пламен Автор: Дмитрий Сахаров (Тутаев) Вопрос 24: Несмотря на то, что у девочки из книги Урсулы Ле Гуин одна рука была повреждена, она научилась изготавливать ЕЕ. "ОНА" помогла получить ученую степень магистра. Назовите ЕЕ тремя словами. Ответ: Колыбель для кошки. Источник: 1. http://www.litmir.net/br/?b=79303&p=10 2. http://ru.wikipedia.org/wiki/Колыбель_для_кошки Автор: Арсений Глазовский (Ярославль) Тур: 13 тур. "Рукалицо" (Саранск) Вопрос 1: Этот писатель создал серию романов, в названии которых встречается одно животное. Также он написал приквел к "Гамлету" Шекспира. Его же перу принадлежит и роман, который неоднократно экранизировали и ставили на сцене в качестве мюзикла. Назовите фамилию писателя. Ответ: Апдайк. Источник: http://en.wikipedia.org/wiki/John_Updike Автор: Ксения Козлова (Саранск) Вопрос 2: По словам лингвиста Щербы, писать безграмотно - значит посягать на ИКС людей, к которым мы адресуемся, а потому совершенно недопустимо в правильно организованном обществе. Что мы заменили в вопросе на ИКС? Ответ: Время. Источник: http://www.ruthenia.ru/apr/textes/sherba/sherba20.htm Автор: Ксения Козлова (Саранск) Вопрос 3: Аболютно удивительный сигнал был услышан 15 августа 1977 года специальным поисковым прибором, указывающий на возможное неодиночество. Удивителен он был настолько, что даже приобрел собственное имя, совпадающее с аббревиатурой названия известной компьютерной игры. Какое имя из двух слов имел большой прибор, уловивший этот сигнал? Ответ: Большое ухо. Комментарий: Сигнал "Wow!", предположительно инопланетного происхождения, был обнаружен радиотелескопом Большое ухо. Компьютерная игра - "World of Warcraft". Источник: http://ru.wikipedia.org/wiki/Сигнал_%C2%ABWow!%C2%BB Автор: Евгений Крюков (Рузаевка) Вопрос 4: У нее три "родителя". Двое из них - "близнецы", которые названы в ее же честь. Третий же родитель обязан своим именем той, которая связывает беспечную молодежь с филателистами. Назовите "ребенка". Ответ: Вода. Комментарий: Близнецы - водород, который назван в честь самой воды, третий - кислород, названием обязанный кислоте. Автор: ??? Вопрос 5: Качество ИХ определяется отчасти проволокой. В самых дешевых для разделения используют круглую проволоку. В более дорогих для разделения используется трехгранная проволока. В профессиональных используется тончайшая разделительная проволока. А непрофессионалы иногда используют в качестве ЕЕ фотографию. Назовите ЕЕ. Ответ: Мишень для игры в дартс. Источник: http://ru.wikipedia.org/wiki/Дартс Автор: Николай Хвастунов (Саранск) Вопрос 6: Лингвисты, внимательно изучив надпись, обнаружили пять грамматических ошибок на постаменте этого памятника. Кому установлен этот памятник? Ответ: Кириллу и Мефодию. Источник: http://www.intomoscow.ru/pamyatnik-kirillu-i-mephodiyu.html Автор: Николай Хвастунов (Саранск) Вопрос 7: Этот знак имеет четкое место в известной системе, а в фильме 2006 года его можно заметить практически в каждом кадре. Напишите этот знак, ставший популярным после исполнения политиком двадцатого века. Ответ: V. Комментарий: V означает ванадий в Периодической системе химических элементов, повторяется почти в каждом кадре фильма "V - значит вендетта" и стал популярным жестом после исполнения Черчиллем. Источник: 1. http://www.kinopoisk.ru/film/86208/ 2. http://www.blokadaleningrada.ru/content/view/id-1297/ Автор: Михаил Пивкин (Рузаевка) Вопрос 8: Для транслитерации этой фразы англичане используют двухцветный общественный транспорт. А кому посвящена ария в опере по произведению Пушкина, начинающаяся с этих слов? Ответ: Ольге. Комментарий: "Я люблю вас". Источник: 1. http://answers.yahoo.com/question/index?qid=20080928195408AA6klRd 2. http://clubs.ya.ru/4611686018427399632/replies.xml?item_no=109443 Автор: Николай Хвастунов (Саранск) Вопрос 9: В.Н. Гололобов является автором ЭТОГО на паяльнике, хотя нам более привычен другой инструмент. Эта книга не только о том, как правильно держать паяльник, но и о... естественном любопытстве и стремлении сделать что-то свое. Назовите ЭТО двумя словами. Ответ: Самучитель игры. Источник: http://rutracker.org/forum/viewtopic.php?t=4173103 Автор: Николай Хвастунов (Саранск) Вопрос 10: Пит Конрад, рост которого - 169 см, находясь в специфической местности, можно сказать, отождествил себя с человечеством. А какое имя он при этом произнес? Ответ: Нил. Зачет: Нейл. Комментарий: Находился Пит на луне, вспоминая известную фразу Нила Армстронга. Источник: http://en.wikipedia.org/wiki/Pete_Conrad Автор: Николай Хвастунов (Саранск) Вопрос 11: Как известно, многие фильмы доходят до нас не в том виде, в котором они была задуманы сценаристом и режиссером, в том числе из-за того, что нужный актер отказался от роли. Так, финальная сцена продолжения одного культового фильма должна была выглядеть примерно так: "... из космического корабля вышло существо, которое было больше своих собратьев и вместо того, чтобы убить главного героя, оно вручило ему кремниевый пистолет и сняло маску, под которой был известный актер". Назовите его первую роль. Ответ: Геркулес. Источник: 1. http://www.filmz.ru/pub/1/20107_1.htm 2. http://ru.wikipedia.org/wiki/Карьера_Арнольда_Шварценеггера_в_кинематографе Автор: Михаил Пивкин (Рузаевка) Вопрос 12: Однажды один кинорежиссер предпринял попытку художественного переосмысления понятий Французской революции 1789 года и снял серию фильмов, общее название которых складывалось в один из символов этой революции, хотя, перепутав очередность фильмов, можно было подумать, что фильмы посвящены другой европейской стране. Назовите фильмы в правильной очередности. Ответ: "Три цвета: Синий", "Три цвета: Белый", "Три цвета: Красный". Источник: http://www.kinopoisk.ru/review/947553/ Автор: Михаил Пивкин (Рузаевка) Вопрос 13: После вручения "Оскара-2013" в Интернете появилась видео, сообщающее о съемках нового фильма Стивена Спилберга про жизнь Барака Обамы. На нем актер, представившийся Дэниелом Дей-Льюисом, в образе Барака Обамы рассказывает про фильм и делает несколько "фирменных" жестов президента США. Видео оказалось уткой, и никакого фильма про Обаму Спилберг снимать не собирается. Да и вообще, сложно белому сыграть чернокожего, как бы ни был хорош грим. Мы не просим назвать человека, сыгравшего Обаму. Назовите режиссера этого ролика, который номинировался в том же году на "Оскар", но не получил его, в отличие от исполнителя главной роли в его картине. Ответ: Спилберг. Источник: http://slon.ru/fast/world/video-dnya-barak-obama-snyalsya-v-filme-stivena-spilberga-938030.xhtml Автор: Михаил Пивкин (Рузаевка) Вопрос 14: В кинематографе очень популярны фильмы, где зрителя пугают чем-то, с чем он может столкнуться в жизни. Так, есть фильмы про цунами, землетрясения, нападения волков, львов, нападения инопланетян и гигантских крабов. ПЕРВОЕ ежегодно уносит около сотни человеческих жизней в одной только США, а ВТОРЫЕ убивают ежегодно около десятка человек. Студия "Асилум" решила пойти дальше и совместила ПЕРВОЕ и ВТОРЫХ в одном фильме. Причем ПЕРВОЕ нисколько не мешало ВТОРЫМ истреблять людей, нападая на них из совершенно неподходящей для ВТОРЫХ среды, то есть с неба. Назовите название этого фильма. Ответ: "Sharknado". Зачет: Любое объединение слов "торнадо" и "акулы". Источник: http://www.kinopoisk.ru/film/740360/ Автор: Михаил Пивкин (Рузаевка) Вопрос 15: Уэйн Оллвайн работал актером озвучания 32 года. В 1991 году он женился на Русси Тейлор, которая работает актером озвучания и по сей день. А каких персонажей они озвучивали? Ответ: Микки Маус и Минни Маус. Источник: http://ru.wikipedia.org/wiki/Оллвайн,_Уэйн Автор: Николай Хвастунов (Саранск) Вопрос 16: После матча чемпионата Англии между "Арсеналом" и "Фулхэмом" в одном из блогов на портале sports.ru появилась заметка с говорящим названием из трех слов, абсолютно точно характеризующим состояние команды гостей в этом розыгрыше чемпионата. Причем пробел между первым и вторым словом вполне можно заменить на еще одну букву, не изменив при этом сути заголовка. Восстановите любой из вариантов заглавия заметки. Ответ: "Не дачный сезон". Зачет: "Неудачный сезон". Комментарий: Прозвище "Фулхэма" среди болельщиков - "дачники". Источник: http://www.sports.ru/tribuna/blogs/arsarsh/555026.html Автор: Евгений Крюков (Рузаевка) Вопрос 17: Ошибки перевода - обычное дело для Интернета. Но иногда ошибки носят абсолютно парадоксальный характер. Так, например, одно из подразделений РАН на официальном сайте в английской версии долгое время имело абсолютно некорректный перевод. 20 сентября 2010 года после огласки в прессе данный казус был исправлен, но в памяти интернет-пользователей он остался надолго, а вот была ли связана эта ошибка с алкоголем или нет - неизвестно до сих пор. Назовите это подразделение РАН. Ответ: Институт белка. Комментарий: В изначальном переводе на англоязычной версии сайта РАН он был назван Squirrel Institute. Источник: http://www.wikireality.ru/wiki/Squirrel_Institute Автор: Евгений Крюков (Рузаевка) Вопрос 18: На одной юмористической фотографии запечатлена известная серия книг американского писателя с несколькими десятками закладок. Если же делать подобную фотографию для другой, не менее известной, серии книг английской писательницы, таких закладок получится гораздо меньше, если не брать в расчет последнюю книгу серии. Что они символизируют? Ответ: Гибель главных героев. Комментарий: Речь о сериях "Песнь льда и пламени" и "Гарри Поттер". Источник: http://pikabu.ru/story/kazhdaya_smert_v_serii_knig_quotigra_prestolovquot_vyidelena_zakladkoy_1712083 Автор: Евгений Крюков (Рузаевка) Вопрос 19: [Ведущему: читать стих четко и внятно.] Там и мат, И кабакИ, И казакИ, Кем чучмек Закопан [ПРОПУСК]. Заполните пропуск словом, которым называется один из разделов сайта lookatme.ru [лук эт ми точка ру], посвященный моде. Ответ: Напоказ. Комментарий: Каждая из строк стихотворения - палиндром. Источник: 1. http://khanmagomedovy.narod.ru/index/0-24 2. http://www.lookatme.ru Автор: Евгений Крюков (Рузаевка) Вопрос 20: На основе приключений этих российских персонажей "1С-СофтКлаб" выпустила серию компьютерных игр: "Погоня", "Круглый счет", "Догонялки" и другие. Назовите этих персонажей, если их латинские названия различаются только одной гласной буквой. Ответ: Заяц, волк. Комментарий: Lepus - заяц, Lupus - волк. Источник: http://www.bambini.by/articles/chto-mi-znaem-o-multfilme-nu-pogodi/ Автор: Евгений Крюков (Рузаевка) Вопрос 21: <раздатка> мэн, тис, фейс Автор одного из переводов Евгений Синельщиков использовал слова "мэн", "тис", "фейс". Назовите любимый напиток главного героя произведения. Ответ: Молоко. Комментарий: В произведении "Заводной апельсин", автор использовал транслитерированные русские слова. переводчик на русский сделал наоборот - написал некоторые английские слова русскими буквами. Автор: Евгений Крюков (Рузаевка) Вопрос 22: Правила этой игры настолько сложны, что наглядно представить их практически невозможно. Цель игры состоит в том, чтобы найти глубинную связь между предметами, которые относятся к совершенно разным на первый взгляд областям науки и искусства, а также выявить их теоретическое сходство. Изначально в игре использовались игровые кости, которые были похожи на костяшки счетов или на камни, которые используются для игры в го, но позднее игровые кости уже не использовались, так как считались излишними. Назовите эту игру. Ответ: Игра в бисер. Источник: http://ru.wikipedia.org/wiki/Игра_в_бисер#.D0.9F.D1.80.D0.B0.D0.B2.D0.B8.D0.BB.D0.B0_.D0.B8.D0.B3.D1.80.D1.8B Автор: Евгений Крюков (Рузаевка) Вопрос 23: "Дабы могли рабы твои На сладостных струнах Возглашать твои дивные деяния, Очисти от грехов Их бренные уста, Святой Иоанн!". Этот гимн послужил прообразом системы, впоследствии получившей широкое распространение и дошедшей практически неизменной до наших дней. Единственное усовершенствование было предпринято неким Джованни Дони. Какое именно? Ответ: Замена ноты ут на ноту до. Зачет: Введение ноты до. Источник: http://www.countries.ru/library/middle_ages/srvmusic.htm Автор: Евгений Крюков (Рузаевка) Вопрос 24: Этот персонаж в литературе представляется то тайным агентом Рима, то жалким и слабоумным человеком. А где он находится в известной комедии? Ответ: В девятом кругу ада. Комментарий: Иуда Искариот. Источник: 1. А. и Б. Стругацкие. Отягощенные злом, или Сорок лет спустя. 2. М. Булгаков. Мастер и Маргарита. 3. Данте. Божественная комедия. Автор: Ксения Козлова (Саранск) Тур: 14 тур. "Кашалот и петунии" (Москва) Вопрос 1: Мы начинаем наш тур. Перед вами игрок команды "Кашалот и петунии" Максим Александров. По его мнению, если бы известный человек был живым, то он бы во многом не одобрил экипировку Максима. Назовите имя и отчество этого человека. Ответ: Александр Васильевич. Автор: Максим Александров, Амаль Имангулов Вопрос 2: Команда "Кашалот и петунии" сильно пожалела, что купила несколько очень плохих ИКСОВ. В 30-х - 40-х годах XX века ИКС использовался в качестве эвфемизма для чего-то очень плохого. Назовите ИКС двумя словами. Ответ: Еврейский вопрос. Автор: Амаль Имангулов, по вдохновению уфимских знатоков Вопрос 3: (pic: 20140049.jpg) Напишите слово, которое мы дважды закрыли от вас на этой фотографии. Ответ: Sherlock. Зачет: Шерлок. Комментарий: Был он пустым. Началом нового сезона "Шерлока" и был эпизод "Пустой катафалк". Автор: Амаль Имангулов Вопрос 4: Внимание, в этом вопросе мы заменили на ИКС два слова. Название статьи в журнале "Секрет фирмы" о революции в обмене информации в Интернете и быстро растущих компаниях-агрегаторах контента лишь одной буквой отличается от ИКСА. Стекло, согласно Википедии, - это тоже ИКС. Напишите название статьи. Ответ: "Агрегатное достояние". Источник: 1. "Секрет фирмы", сентябрь 2013 г. - С. 50. 2. http://ru.wikipedia.org/wiki/Агрегатное_состояние#.D0.A1.D1.82.D0.B5.D0.BA.D0.BB.D0.BE Автор: Артемий Анцупов Вопрос 5: По одной из легенд, Нобель однажды прочитал ЭТО и создал премию. Кто, после того как прочитал ЭТО, внес исправления? Ответ: Марк Твен. Комментарий: ЭТО - собственный некролог. Автор: Амаль Имангулов Вопрос 6: Участок национальной дороги номер 85 между Лазурным берегом и Греноблем из-за большого количества табличек и памятников напоминает гигантский мемориальный ИКС. Назовите ИКС двумя словами, одно из которых - имя собственное. Ответ: Комплекс Наполеона. Комментарий: Путь, по которому двигался Наполеон после бегства с острова Эльба, ныне является участком французского национального маршрута N 85, и рядом с ним располагается множество памятных знаков. Автор: Никита Поверинов Вопрос 7: Пьер Байяр в книге "Искусство рассуждать о книгах, которые вы не читали" пишет, что иногда нечтение может быть на пользу. В главе, рассказывающей о создании мнения о непрочитанной книге, упоминается имя. Ответьте четырьмя буквами: какое? Ответ: Розы. Зачет: Роза. Комментарий: Иногда знать содержание книги, не читая ее саму, весьма полезно. Например, когда страницы книги отравлены, как у Эко. Источник: В вопросе. Автор: Амаль Имангулов Вопрос 8: Персонаж британского фильма "Хокинг" говорит, что не верит в некую теорию и намеренно дал ей название, которое звучит несерьезно, мультяшно. Напишите название этой теории по-английски или по-русски. Ответ: The big bang theory. Зачет: Теория большого взрыва. Автор: Амаль Имангулов Вопрос 9: <раздатка> wimoweh Перед вами одно из названий песни, которою обычно связывают с Королем Лиром. В этом вопросе мы немного изменили одно из слов. Как оно звучало первоначально? Ответ: Львом. Комментарий: Так называется песня "The Lion Sleeps Tonight". Действительно, слова "o-wimoweh, o-wimoweh, o-wimoweh" повторяются на протяжении всей песни. Она играла и в "Короле Льве", и в "Короле Льве 3/2: Тимон и Пумба", и еще много где. Автор: Амаль Имангулов Вопрос 10: Создатель первой аналитической машины Чарльз Бэббидж говорил, что его машина будет обладать "мельницей и складом". А как "мельница и склад" называются сейчас? Ответ: Мельница - оперативная память (ОЗУ), склад - жесткий диск. Источник: Введение в программирование. - Минск: Харвест, 1997. - Глава 1. Автор: Николай Яковлев Вопрос 11: Уважаемые знатоки! Что бы ни произошло, когда вы сдадите ответ на этот вопрос, - он будет или правильным, или неправильным. Tertium non datur. Известный российский математик А.П. Ершов говорил, что "программист должен обладать способностью первоклассного математика к абстракции и логическому мышлению в сочетании с эдисоновским талантом сооружать всё, что угодно, из гайки и винта". Какие слова мы заменили словами "гайка и винт"? Ответ: Ноль и единица. Комментарий: Представьте себе гайку и винт: они внешне напоминают ноль и единицу. Источник: Введение в программирование. - Минск: Харвест, 1997. - Предисловие. Автор: Николай Яковлев Вопрос 12: Константина Арбенина зачастую называют преемником традиций ленинградского рок-клуба. Закончите его четверостишие четырьмя словами: "Я возьму не свое пальто, Я допью не свое саке И уйду туда, где никто Знать не знает...". Подскажем лишь, что в ответе дважды должна звучать частица "ни". Ответ: Ни Б, ни Г. Источник: http://www.zzverey.spb.ru/list.php?id=23&text=15 Автор: Николай Яковлев Вопрос 13: Дмитрий Шадрин, следуя легендарной фразе, говорит, что ему плевать на нормы социума, а общественное мнение, несмотря ни на что, ДЕЛАЕТ ЭТО. Ответьте одним словом: делает что? Ответ: Вертится. Автор: Амаль Имангулов Вопрос 14: Джим Джеффрис рассказывает, как смотрит порно на своем ноутбуке и почти всегда в итоге ПОЛУЧАЕТ ПЯТЬДЕСЯТ ОЧКОВ. Какие три слова мы заменили в тексте этого вопроса? Ответ: Попадает в яблочко. Автор: ??? Вопрос 15: Самый невероятный герой "Мастера и Маргариты" удивлялся отсутствию у людей ЭТОГО. Цинциннат же говорил, что ЭТИМ компенсируется смертный приговор. Что это? Ответ: [Точное] знание смертного часа. Комментарий: Воланд удивлялся, что люди (в частности, Берлиоз) не знают, когда они умрут. Источник: 1. М. Булгаков. Мастер и Маргарита. 2. В. Набоков. Приглашение на казнь. Автор: Николай Яковлев Вопрос 16: Курт Воннегут пишет, что нельзя оглядываться на свое прошлое и ностальгически вспоминать его. Далее он пишет, что книга выходит неудачной, если ее автор - ИКС. Назовите ИКС двумя словами, начинающимися на одну и ту же букву. Ответ: Соляной столб. Источник: К. Воннегут. Бойня N 5, или Крестовый поход детей. Автор: Амаль Имангулов Вопрос 17: В мультфильме "Песнь о буревестнике", действие которого происходит в школе, ОН ДЕЛАЕТ ЭТО. Ответьте четырьмя словами на одну и ту же букву, что мы заменили словами "ОН ДЕЛАЕТ ЭТО". Ответ: Пингвин прячется под партой. Автор: Александр Карясов, Амаль Имангулов Вопрос 18: По наблюдениям автора, скоро орка на ролевых играх сложно будет представить без "Оки". На вопрос о том, как ведет себя иномарка автора вопроса в холода, заданный другом, автор ответил: "Как Ока". То, что мы заменили на "Ока", нашло место на флагах нескольких стран. Назовите это. Ответ: Автомат Калашникова. Источник: ЛОАВ. Автор: Григорий Воронков Вопрос 19: Героев песни одного исполнителя авторской песни всегда рассудит ОНО. Синее ОНО куда легче обнаруживается на кассе в супермаркете, чем на карте. Назовите его. Ответ: Сигареты "More". Комментарий: Сигареты "More" в России очень часто называют "море". Соответственно, на кассе в супермаркете (где обычно продаются сигареты), просят дать "Синий море" или "Синее море". Синего моря нет на карте мира. Источник: 1. http://teksty-pesenok.ru/rus-kancler-gi/tekst-pesni-romans-olafa-kaldmeera/1818984/ 2. http://www.prosigareti.ru/sigareti-more.html 3. http://www.okidoki.ee/ru/item/449135/ Автор: Григорий Воронков Вопрос 20: Внимание, в вопросе есть замены. В одной из сцен фильма "Ночной портье" полуобнаженная девушка танцует на пирушке. Затем она получает в подарок АЛЬФУ, от чего приходит в шок. Назовите АЛЬФУ. Ответ: Голова. Зачет: Отрубленная голова, голова обидчика. Автор: Дарья Бычкова Вопрос 21: Персонаж Бернарда Корнуэлла, датский морской торговец, с презрением называет шотландское березовое вино АЛЬФОЙ, вероятно, за его вкус и цвет. Настоящая АЛЬФА перед выведением проходит путь примерно в полметра, а также содержит, в том числе, немало рыбьего жира. Назовите АЛЬФУ двумя словами на одну и ту же букву. Ответ: Моржовая моча. Автор: Никита Поверинов Вопрос 22: Согласно сайту анекдотов, ИКС по-оркски означает "криворукий охотник". Первого октября мы празднуем всемирный день ИКСОВ. Ответьте, кто такие ИКСЫ, если точно известно, что в книге Льва Толстого "Первая ступень" появление ИКСОВ служит признаком того, что стремление к нравственному совершенствованию человека серьезно и искренно. Ответ: Вегетарианцы. Источник: 1. http://права-животных.рф/library/philosophy/tolstoy_the_first_step.htm 2. http://ru.wikipedia.org/wiki/Всемирный_день_вегетарианства Автор: Полина Саханевич Вопрос 23: Не спрашивайте, почему ответ на этот вопрос такой. Рэпер Дёрти Монк шуточно угрожает, что его враги всплывут, как испорченные ИКСЫ. В поздравлении ИКСАМ екатеринбургский журнал "Красная бурда" желает им путь, утыканный ИГРЕКАМИ. Мы не спрашиваем, какое слово мы заменили на ИКС. Ответьте, какое слово мы заменили на ИГРЕК. Ответ: Гладиолус. Автор: Амаль Имангулов Вопрос 24: Это последний вопрос нашего тура, перед ним у вас есть совсем немного времени, чтобы покурить, поесть и сделать много других вещей. В анекдоте будущий лауреат Нобелевской премии спрашивает у учительницы разрешения выйти. Воспроизведите ее ответную реплику из семи слов. Ответ: "Только в уборную, и сразу же возвращайся". Автор: Амаль Имангулов Тур: 15 тур. "Поцелуй Фрейи" (Москва) Вопрос 1: На недавнем турнире по "Что? Где? Когда?" одна из команд провалила первый тур, взяла все вопросы во втором, а в третьем снова опустилась. Комментируя этот факт, циничная команда автора вопроса предложила купить букетик цветов и упомянула имя. Какое? Ответ: Элджернон. Зачет: Чарли Гордон. Комментарий: Чарли Гордон, герой рассказа Дэниэла Киза "Цветы для Элджернона", также вышел на пик своих интеллектуальных способностей, которые затем угасли. Источник: ЛОАВ. Автор: Кирилл Новокщёнов (Москва), в редакции Александра Берелехиса (Уфа) Вопрос 2: Автор вопроса настолько любит соевый соус, что сравнивает его с первой, хотя последние исследования показывают, что такое сравнение уже неуместно. Какие два слова мы пропустили? Ответ: Группой крови. Комментарий: Соевый соус подходит к любому блюду. Раньше считалось, что первая группа крови подходит всем, однако сейчас реципиентам стараются переливать лишь кровь их группы. Источник: ЛОАВ. Автор: Кирилл Новокщёнов (Москва), в редакции Александра Берелехиса (Уфа) Вопрос 3: Героиня песни Киры Малыгиной обвиняет известного персонажа, человека благородного происхождения, собирающегося отправиться в дорогу весьма необычным способом, в том, что он продавал ей мечту, а она поверила. В песне есть такие строки: "На ветер брошены слова и листья ворохом, Как яду я подсыплю вам...". Закончите последнюю строку двумя словами. Ответ: "... сырого пороху". Зачет: "... сырого пороха". Комментарий: Песня рассказывает о несчастной любви героини к любителю вранья барону Мюнхгаузену. В конце фильма "Тот самый Мюнхгаузен" Марта, наоборот, признаётся барону, что ему насыпали сырой порох. Источник: Песня Киры Малыгиной "Барон Мюнхгаузен". Автор: Кирилл Новокщёнов (Москва), в редакции Александра Берелехиса (Уфа) Вопрос 4: Карьера Ника Палмиери в НХЛ не задалась, поскольку игрок не показывал достаточно результативную и зажигательную игру. Тем не менее, он успел получить прозвище ПАЛЬМИТИН. Какое слово мы заменили на ПАЛЬМИТИН? Ответ: Напалм. Комментарий: Слово "зажигательная" - подсказка. Напалм получил свое название в т.ч. и благодаря пальмитиновой кислоте. Источник: 1. http://en.wikipedia.org/wiki/List_of_ice_hockey_nicknames 2. http://ru.wikipedia.org/wiki/Напалм Автор: Кирилл Новокщёнов (Москва), в редакции Александра Берелехиса (Уфа) Вопрос 5: [Ведущему: читать медленно и размеренно, чтобы команды всё смогли понять.] В финале олимпийского хоккейного турнира в Ванкувере Канада обыграла США. Канадец Райан Гецлаф после этого несколько месяцев подкалывал своего партнера по "Анахайму" американца Бобби Райана. Гецлаф постоянно стремился напомнить Райану о том, что именно Гецлаф стал олимпийским чемпионом, это было видно даже по его вещам. В итоге Райан сравнил Гецлафа с ИКСОМ. ИКС - талисман баскетбольной команды "Бостон Селтикс". Какое слово мы заменили на ИКС? Ответ: Лепрекон. Комментарий: Гецлаф окружил себя золотыми вещами. В Бостоне живут много выходцев из Ирландии, прозвище "кельты" тоже появилось не случайно. Источник: http://www.youtube.com/watch?v=xMSHf79zlUk Автор: Кирилл Новокщёнов (Москва) Вопрос 6: Главный тренер "Ливерпуля" Брендан Роджерс как-то сказал: "Работать тренером - все равно что строить самолет". Какие два слова мы пропустили? Ответ: На лету. Зачет: В полете, в воздухе. Источник: http://www.telegraph.co.uk/sport/football/teams/liverpool/10536272/Liverpool-manager-Brendan-Rodgers-deserves-our-praise.html Автор: Кирилл Новокщёнов (Москва) Вопрос 7: "Он очень разносторонен - он и [ПРОПУСК], и джентльмен, и поэт, и мечтатель, а в общем это одинокое существо, мечтающее о красивой любви и приключениях. ... В то же время он готов подобрать с тротуара окурок или отнять у малыша конфету. И, разумеется, при соответствующих обстоятельствах он способен дать даме пинка в зад, - но только под влиянием сильного гнева". Мы не просим вас восстановить пропущенное слово. Назовите автора этой цитаты. Ответ: Чарли Чаплин. Комментарий: Чаплин так говорил о Бродяге - одном из самых ярких своих образов. Источник: http://ru.wikipedia.org/wiki/Чаплин,_Чарльз Автор: Кирилл Новокщёнов (Москва) Вопрос 8: Павел Зиновьев рассказывает о выдающемся мастерстве футболиста Франка Рибери и отмечает, что отобрать мяч у Рибери - это АЛЬФА. Множество ИХ можно найти в книге Марка, а книгу, в свою очередь, - на полках старшеклассников. Какие три слова мы заменили на АЛЬФУ? Ответ: Задача со звездочкой. Комментарий: Марк - Сканави. Источник: http://www.sports.ru/football/154030548.html Автор: Кирилл Новокщёнов (Москва) Вопрос 9: Вот далеко не полный список пропаж: март 1918 года - "Циклоп", октябрь 1931 года - "Ставентер", декабрь 1948 года - "Дакота-3", март 1950 года - "Глоубмастер", июнь 1950 года - "Сандра", февраль 1963 года - "Марин Салфер Куин", ноябрь 1970 года - "Джилли Бин" и т.д. Согласно широко известному источнику, в 70-е годы прошлого века группа граждан, находившаяся тогда в Подмосковье, была очень встревожена и возмущена этими событиями. Эти граждане решили обратиться с посланием, в котором требовали разъяснений. Однако, не слишком надеясь на ответ, они также собирались отправить еще одно письмо. Назовите адресата второго письма. Ответ: "Спортлото". Комментарий: Бермудский треугольник и песня В. Высоцкого. Источник: http://lib.ru/WYSOCKIJ/v77.txt Автор: Владислав Виноградов (Уфа) Вопрос 10: ИКС упомянут еще в Книге Царств. Однако в России ИКС появился гораздо позже. В Москве - более 200 лет назад, в Уфе - в начале XX века, а в Ижевске - в 1934 году. С появлением и развитием ИКСА постепенно исчезла профессия, один из представителей которой задавался вопросом о причинах избрания им такой профессии и сам же давал на него ответ. Воспроизведите этот ответ. Ответ: "Потому что без воды - ни туды и ни сюды". Комментарий: Песня из кинофильма "Волга-Волга". Источник: 1. http://ru.wikipedia.org/wiki/Водопровод 2. http://www.songkino.ru/songs/volga.html Автор: Владислав Виноградов (Уфа) Вопрос 11: Гуляющий по городу герой Переса-Реверте назвал ЭТО плутовским факультетом. ЭТО переводится на английский словом "parvise" [пэрвайз], которое, по одной из версий, этимологически связано со слово "paradise" [пэрэдайз] - рай. В прошлом, в середине ЭТОГО обычно устраивали бассейн. Назовите ЭТО. Ответ: Паперть. Комментарий: На паперти побиралось много плутов, притворявшихся больными и нищими на все лады. Источник: 1. А. Перес-Реверте. Корсары Леванта. 2. http://ru.wikipedia.org/wiki/Паперть Автор: Кирилл Новокщёнов (Москва) Вопрос 12: Великий хоккеист Марио Лемье родился в Монреале и с юношеских лет был олицетворением франкоязычной Канады. Под каким номером играл Марио Лемье? Ответ: 66. Комментарий: Лемье противопоставлялся другому великому хоккеисту - англоязычному выходцу из эмигрантской семьи Уэйну Гретцки. Гретцки играл под 99-м номером, Лемье перевернул это число. Источник: http://ru.wikipedia.org/wiki/Лемье,_Марио Автор: Павел Кебец (Москва), в редакции Кирилла Новокщёнова (Москва) Вопрос 13: На одном фото видно, как солдаты ДЕЛАЮТ ЭТО в противогазе. Советуют также поставить рядом свечу, прежде чем ДЕЛАТЬ ЭТО. Какие два слова мы заменили? Ответ: Чистить лук. Комментарий: Клетки лука содержат вещества летучей серы, которые выпускаются в воздух, когда в них вонзается нож. Пламя от свечи поможет окислить серу. Источник: 1. https://www.facebook.com/photo.php?fbid=197738223738166 2. http://intesto.com/school/697-chistim-luk-bez-slez.html Автор: Кирилл Новокщёнов (Москва) Вопрос 14: В шуточной лекции под названием "Политбюро учит нас переходить к управлению с помощью АЛЬФЫ" есть фраза "Ускорение темпов роста научно-технического прогресса - наша основная задача!". Какие два слова, начинающиеся на одну и ту же букву, мы заменили словом "АЛЬФА"? Ответ: Пятая производная. Комментарий: Производная в математике - функция, выражающая скорость возрастания другой функции. Соответственно, пятая производная - скорость возрастания скорости возрастания скорости возрастания скорости возрастания скорости возрастания функции. :-) "Ускорение темпов роста научно-технического прогресса - наша основная задача", говорю. "Давайте считать: ускорение - вторая, темп - третья, рост - четвертая, прогресс - пятая!". Шум и начинают ржать. "Потому что, когда всё летит в тартарары, в смысле производные с первой по четвертую отрицательны, но ПЯТАЯ положительна - у нас есть шанс вынырнуть из пике. Хотя вряд ли. Лекция окончена. Вопросы есть?". Источник: http://www.dxdy.ru/topic14151.html Автор: Борис Белозёров (Москва) Вопрос 15: Молнии Кататумбо - появляющиеся ночью над местом впадения реки Кататумбо в озеро Маракайбо 140-160 раз в год разряды молний без сопровождающих акустических эффектов, длящиеся около десяти часов. Их также называют "ИКС Маракайбо". "ИКС" был основан в 1964 году для "противовеса западным "голосам". Назовите ИКС. Ответ: Маяк. Комментарий: Молнии настолько яркие, что их видно даже с расстояния 400 километров, поэтому они использовались для навигации. Решением ЦК КПСС в 1964 году была основана радиостанция "Маяк". Источник: 1. http://ru.wikipedia.org/wiki/Молнии_Кататумбо 2. http://ru.wikipedia.org/wiki/Маяк_(радиостанция) Автор: Борис Белозёров (Москва) Вопрос 16: Внимание, в вопросе есть замена. Когда недавно пополневший автор вопроса прогуливался по Москве, он увидел магазин "Фантастическая четверка" и подумал, что теперь это магазин для него. Что мы заменили словами "Фантастическая четверка"? Ответ: "Люди Икс". Комментарий: Автор вопроса сменил размер L на XL, а "Фантастическая четверка" - такой же экранизированный комикс, как и "Люди Икс". Источник: Личный опыт достаточно толстого автора вопроса в естественной среде. Автор: Кирилл Железнов (Москва) Вопрос 17: Страдавшему дислексией человеку тяжело давалось запоминание числовых артикулов товаров, что отразилось в политике основанной им компании. Назовите эту компанию. Ответ: "IKEA". Комментарий: Именно поэтому товары из "ИКЕА" носят названия растений, животных, имена людей и т.д. Источник: http://www.aif.ru/money/business/36203 Автор: Александр Карчевский (Саратов - Москва) Вопрос 18: В 1967 году Ким Гроув влюбилась в своего будущего мужа Роберто Касали в лыжном клубе, где они оба брали уроки. Однажды Ким оставила Роберто записку, в которой вместо подписи нарисовала свой стилизованный портрет с веснушками, большими глазами и длинными светлыми волосами. Что благодаря инициативе Роберто Касали появилось в апреле 1970 года? Ответ: Жевательная резинка "Love is...". Комментарий: Жевательные резинки с вкладышами-рисунками, идею которых предложила Ким Касали в своих любовных записках к мужу. "Всё начиналось, как невинный флирт". :-) Источник: http://en.wikipedia.org/wiki/Kim_Casali Автор: Павел Кебец (Москва) Вопрос 19: <раздатка> "Боже мой! Это же была шутка, не более того. Дурацкое хобби. Игра обезумевшего конструктора - попытка создать компьютер, который невозможно построить на Земле. Ну, может быть, часть идеи и удастся реализовать, но работать машина все равно не будет. Напоминает [ПРОПУСК] - на бумаге выглядит впечатляюще, а в жизни не бывает". Перед вами цитата из романа Роджера Желязны "Карты Судьбы". Восстановите два слова, которые мы пропустили. Ответ: Рисунки Эшера. Зачет: Картины Эшера, наброски Эшера и т.п. Источник: В вопросе. Автор: Кирилл Железнов (Москва) Вопрос 20: Персонаж романа Джеймса Кейбелла попадает в Рай, представившись ИМ, потому что приготовленный там особняк для НЕГО действительно пустует. ИМ в 2005 году провозгласил себя итальянец Роберто Карневале. Назовите ЕГО. Ответ: [Папа Римский] Иоанн XX. Незачет: Иоанн. Комментарий: Возможно из-за легенды о папессе Иоанне, возможно из-за церковных расколов и многочисленных антипап в 1276 году, новоизбранный Папа Римский Педру Жулиан решил взять себе имя Иоанн XXI вместо полагавшегося "Иоанн XX", посчитав нумерацию неверной. Позже его ошибку исправили, но имя "Иоанн XX" в итоге не носил ни один из реально существующих пап, и оно стало своеобразным историческим фантомом. Источник: http://ru.wikipedia.org/wiki/Иоанн_XX Автор: Борис Белозёров (Москва) Вопрос 21: Одного из них снимали, последовательно поставив перед микрофоном два ящика: один побольше, второй - поменьше. Последний из них, с небольшими допущениями, является тезкой знаменитого актера. Назовите этого актера. Ответ: Леонардо ди Каприо. Комментарий: Они - это львы, рычащие в заставке фильмов студии "Metro-Goldwyn-Mayer". Последнего из них зовут Лео - подходящее имя для льва. Источник: 1. http://www.adme.ru/istoriya-brenda/istoriya-lva-iz-legendarnoj-zastavki-metro-goldwyn-mayer-335455/ 2. http://en.wikipedia.org/wiki/Leo_the_Lion_(MGM) Автор: Кирилл Новокщёнов (Москва) Вопрос 22: В заглавии статьи о футболистах, вернувшихся в свой клуб после перерыва, упоминается ОН. А для какого уроженца штата Нью-Йорк ОН не стал последним? Ответ: Франклин [Делано] Рузвельт. Комментарий: ОН - второй срок. Источник: 1. http://www.sports.ru/football/156156832.html 2. http://en.wikipedia.org/wiki/Franklin_D._Roosevelt Автор: Кирилл Новокщёнов (Москва) Вопрос 23: [Ведущему: в начале и конце вопроса прочитать слово "вольтИж" с ударением на "И", а в стихотворном отрывке - с ударением на "О"!] Внимание, в вопросе слова "Вольта", "вольтиж" и "заплотишь" являются заменами. Прослушайте отрывок из стихотворения Евгения Евтушенко "Тихая поэзия": "Без спросу превращая Вольта в вольтиж, бубенчики бренчат с прогнивших дуг: "Эстрада, ты за всё еще заплотишь. Ты горлопанством унижаешь дух". Какое слово мы заменили на слово "вольтИж"? Ответ: ФетИш. Комментарий: "Без спросу превращая Фета в фетиш, бубенчики бренчат с прогнивших дуг: "Эстрада, ты за всё еще ответишь. Ты горлопанством унижаешь дух". А.А. Фет является одним из ярчайших представителей "тихой поэзии" в русской и мировой литературе. У Евтушенко "фетиш" произносится с ударением на первый слог, однако по нормам русского языка оно должно произноситься с ударением на второй слог, как и слово "вольтиж". Это было небольшой дополнительной подсказкой. Источник: В вопросе. Автор: Павел Кебец (Москва) Вопрос 24: В одном вымышленном виде спорта используются, например, пламягасительный или одурительный мячи. А роль своеобразных ворот исполняет ОН. Какие четыре буквы мы пропустили в тексте вопроса? Ответ: ДРАК. Комментарий: Вид спорта называется драконбол - очки команде начисляются за заброшенные в пасть дракону команды противников мячи. Источник: Серия книг о Тане Гроттер. :-) Автор: Борис Белозёров (Москва) Тур: 16 тур. "Какие-то лохи" (Санкт-Петербург) Вопрос 1: Статья в журнале "Огонек", посвященная выставке шедевров Франсиско Гойи в Эрмитаже, называется "Испанцы ЛОХАНУЛИСЬ". Какие два слова мы заменили на "ЛОХАНУЛИСЬ"? Ответ: Дали "Маху". Зачет: Дали Маху, дали маху. Источник: "Огонек", 1998, N 37. Автор: Михаил Дворкин Вопрос 2: Сюжет в выпуске новостей "Euronews", посвященный причинам временного закрытия аэропорта Мехико, назывался "ПЕРВЫЙ пошел по пути ВТОРОГО". Назовите ПЕРВЫЙ и ВТОРОЙ хотя бы приблизительно. Ответ: Попокатепетль и Эйяфьятлайокудль. Зачет: Любые буквосочетания, позволяющие заключить, что речь идет об означенных вулканах. Источник: Выпуск новостей "Euronews" от 05.07.2013 г. Автор: Александр Топаж Вопрос 3: <раздатка> История армянского книгопечатания Восканяновская типография издавала как религиозные, так и светские книги.   Золотой храм (роман) В романе в форме псевдоисповеди предлагается авторская версия...   Сосновка (Мурманская область) В 2007 году на побережье Белого моря было начато сооружение пяти ветроагрегатов. Автор статьи "Охота на слова" на портале Хабрахабр приводит самые длинные слова, обладающие некоторым свойством, найденные в процессе анализа всей русскоязычной Википедии и, в частности, розданных вам цитат. Запишите любое другое слово, обладающее тем же свойством. Ответ: Мама. Зачет: Любое слово, в котором каждая буква встречается ровно два раза. Источник: http://habrahabr.ru/post/189236/ Автор: Михаил Дворкин Вопрос 4: Дженни Лорен устроила пьяный дебош на борту самолета, вследствие чего он произвел экстренную посадку в ирландском аэропорту Шаннон. Женщина предстала перед судом в соседнем с аэропортом городке Киллало. Так как в этом городке нет зала суда, заседание прошло... Где? Ответ: В пабе. Зачет: В баре, кабаке и др. питейном заведении. Источник: http://ria.ru/world/20140107/988177938.html Автор: Вера Круз Вопрос 5: Дуплет. Преамбула. Когда пишут в единственном числе о золотом ИКСЕ, то скорее всего, используется существительное мужского рода, а когда пишут во множественном числе о красных ИКСАХ, то скорее всего, среднего. 1. Запишите существительное ИКС мужского рода в именительном падеже из трех букв. 2. Запишите существительное ИКС мужского рода в именительном падеже из пяти букв. Ответ: 1. Век. 2. Телец. Источник: 1. http://yandex.ru/yandsearch?text="золотом%20веке"&lr=2 2. http://yandex.ru/yandsearch?text="красных%20веках"&lr=2 3. http://yandex.ru/yandsearch?text="золотом%20тельце"&lr=2 4. http://yandex.ru/yandsearch?text="красных%20тельцах"&lr=2 Автор: Михаил Дворкин Вопрос 6: Порошок - модная стихотворная форма - четверостишие, первые три строки которого написаны четырехстопным ямбом, а последняя короткая строка состоит из двух слогов и рифмуется со второй. Прослушайте порошок Марии Беляевой: подайте черного монаха кричит аркадий и ему официант в слезах приносит муму Какие два слова мы заменили в этом порошке на "черного монаха"? Ответ: Жалобную книгу. Комментарий: Один рассказ Чехова заменен на другой. Источник: http://vk.com/perawki?w=wall-28122932_24542 Автор: Александр Топаж Вопрос 7: Внимание, в вопросе есть две замены. Рискованная шутка автора вопроса: "Ух, как мы сейчас с тобой пошалим, Малыш! - сказал Карлсон, пролетая над Стокгольмом". Какое слово мы заменили на "Карлсон"? Ответ: Толстяк. Комментарий: Речь идет о бомбах, сброшенных на Хиросиму и Нагасаки. Источник: Жесткий юмор автора вопроса. Автор: Мария Топаж Вопрос 8: Она снималась в таких популярных фильмах, как "Бриллиантовая рука" и "Служебный роман". При этом главный герой "Служебного романа" Новосельцев ее не бросил, а главный герой "Бриллиантовой руки" Горбунков ее, напротив, отверг. А по какой причине? Ответ: Она была с крыльями. Зачет: Нужна была без крыльев и прочие синонимичные ответы с упоминанием того, что крылья были лишними. Комментарий: Речь идет о тяжелой металлической скульптуре крылатой лошади, к которой Горбунков приценивался в магазине, а Новосельцев таскал по просьбе Шурочки, "чтобы юбиляр заранее не обрадовался". Это одна и та же скульптура из реквизиторской "Мосфильма". Источник: http://mik-kiev.livejournal.com/68658.html Автор: Александр Топаж Вопрос 9: Если вы обратитесь к своим записям, то найдете, что такое порошок. Завершите порошок автора под ником "Oleg Oleg": их спросишь что такое совесть и сразу опускают взгляд не обольщайтесь им не стыдно ... Ответ: гуглят Источник: http://vk.com/perawki?w=wall-28122932_25366 Автор: Александр Топаж Вопрос 10: <раздатка> I have sex daily. I mean <...>. Fcuk. Ведущий несколько раз прочитал этот вопрос, чтобы не допустить ни одной ошибки. Вам роздана англоязычная шутка: "У меня ежедневный секс. Ой, то есть [ПРОПУСК]. Видоизмененное ругательство". Восстановите пропущенное слово на русском или английском языке. Ответ: Дислексия. Зачет: Dyslexia. Комментарий: Расстройство чтения, при котором буквы не выстраиваются в правильном порядке. Источник: http://www.finetshirtdesigns.com/view/1028670/i-have-sex-daily-i-mean-dyslexia-fcuk-t-shirt Автор: Михаил Дворкин Вопрос 11: Во второй половине XIX века в США ненадолго возник литературный жанр под названием "Анти-ИКС". Авторами книг были писатели-южане, многие книги были написаны женщинами. Какое короткое имя мы заменили на ИКС? Ответ: Том. Комментарий: Такой была реакция на "Хижину дяди Тома". Источник: 1. http://ru.wikipedia.org/wiki/Хижина_дяди_Тома#.D0.90.D0.BD.D1.82.D0.B8-.D0.A2.D0.BE.D0.BC 2. http://en.wikipedia.org/wiki/Uncle_Tom's_Cabin#Anti-Tom_literature Автор: Вера Круз Вопрос 12: Согласно шутке, после очередного кругосветного путешествия Федор Конюхов приобрел пять холодильников. И всё ради НИХ. Назовите ИХ. Ответ: Магнитики. Зачет: По смыслу. Источник: http://otvet.mail.ru/question/11851556 Автор: Александр Топаж Вопрос 13: Закончите наблюдение автора вопроса: "Нет в мире совершенства: если месяц начинается с воскресенья, то...". Ответ: "... в нем есть пятница, 13-е". Зачет: По смыслу. Источник: Принятая система времяисчисления. Автор: Александр Топаж Вопрос 14: Английский посланник в Неаполе в начале XIX века говорил: "Лучше иметь 50 процентов в хорошем предприятии, нежели 100 процентов в плохом". Мы не спрашиваем фамилию этого посланника. Назовите его, так сказать, "компаньона" в упомянутом "хорошем предприятии". Ответ: [Горацио] Нельсон. Комментарий: "Хорошее предприятие" - леди Гамильтон, жена этого посланника. Источник: Марк Алданов. Чертов мост. Автор: Александр Топаж Вопрос 15: Осип Михайлович был известен в Одессе, но похоронен на Смоленском кладбище в Санкт-Петербурге. Назовите его фамилию. Ответ: Дерибас. Зачет: Де Рибас. Комментарий: Родился он в Неаполе, происходил из знатного каталонского рода, поэтому в разных странах его называли также Хосе, Жузеп и Джузеппе. Источник: http://ru.wikipedia.org/wiki/Дерибас,_Осип_Михайлович Автор: Вера Круз Вопрос 16: Прослушайте цитату из статьи Сергея Алискерова, заведующего отделом Московского зоопарка, в которой слова "ФЕН" и "ИКС" - замены: "В коллекциях зоопарков белоплечий ФЕН ИКС 1990 года был весьма редок". Какие два слова в этой цитате мы заменили словами "ФЕН ИКС"? Ответ: Орлан до. Комментарий: Логика замены - разные американские города. Источник: http://www.earaza.ru/wps/wp-content/uploads/vest_orlan.pdf Автор: Михаил Дворкин Вопрос 17: Как называется отель на улице Маркина в Санкт-Петербурге? Ответ: Mark Inn. Зачет: С пробелом или без пробела латиницей или кириллицей. Источник: http://www.markinn.ru/ Автор: Вера Круз Вопрос 18: Он - самый быстрый среди российских ПЕРВЫХ. Его скорость достигает 320 км/час. Он - самый быстрый среди российских ВТОРЫХ. Его скорость достигает 250 км/час. Мы не спрашиваем, кто он. Назовите ПЕРВЫХ и ВТОРЫХ словами, начинающимися на одну и ту же букву. Ответ: Птицы, поезда. Зачет: В любом порядке. Комментарий: Он - Сапсан. Источник: 1. http://otvet.mail.ru/question/22281233 2. http://www.sapsan.su/train-info.htm Автор: Александр Топаж Вопрос 19: Абу-ль-Аббас был отправлен Гаруном аль Рашидом в 801 году к Карлу Великому в сопровождении еврея Исаака. Он первым из своего отряда достиг современной Германии, чтобы принимать участие во всех походах императора и погибнуть в 810 году в битве с королем данов. А как сейчас называется отряд Абу-ль-Аббаса? Ответ: Хоботные. Комментарий: Это был слон. Источник: Юрий Аршанский. Про белого слона Абу-ль-Аббаса. Автор: Дмитрий Топаж Вопрос 20: Вы уже знаете, что такое порошок. В одном порошке в третьей строке описывается шахматный дебют, в котором белая пешка делает ход на пять клеток вправо и на три вперед, после чего следует ОН. Восстановите четвертую строку, которую мы заменили словом "ОН". Ответ: о аш. Комментарий: Упомянутый ход - це два аш пять. Источник: http://vk.com/sandporohok?w=wall-32681168_214242 Автор: Михаил Дворкин Вопрос 21: (pic: 20140866.jpg) В этой фразе, придуманной автором вопроса, слово "пожалуй" взято в скобки в соответствии с решением, принятым астрономическим сообществом в 2006 году. Восстановите абсолютно точно пропущенные слова, учитывая, что одно из них - женское имя. Ответ: "... мне Моника...". Комментарий: Это мнемоническая фраза для запоминания последовательности размеров планет Солнечной Системы. Источник: http://www.iau.org/public_press/news/detail/iau0602/ Автор: Михаил Дворкин Вопрос 22: В 1905 году в Вене большую популярность приобрело предприятие семейства Фигльмюллер. При этом некоторые венцы, не зная точного адреса, находили его по характерному стуку, звучащему из подвала здания. Назовите продукт, прославивший это предприятие. Ответ: [Венский] шницель. Комментарий: В подвале отбивали мясо. Источник: http://www.figlmueller.at Автор: Дмитрий Топаж Вопрос 23: <раздатка> Дракон по имени кошмар Звездный мост Ищу тебя Любовь нас выбирает Мир без любимого Он пришел, этот добрый день От грешных страстей С детства я хотела быть первой Твердят, верить не надо Хэппи энд Чарльстон В 1978 году в СССР был снят фильм "31 июня". Песни из него в исполнении Татьяны Анциферовой перечислены в розданном Вам списке. Многие стали настоящими шлягерами. Но одну из них, начиная с 1984 года, певице настоятельно не рекомендовали включать в концертные программы. Какую? Ответ: "Мир без любимого". Комментарий: После отъезда Юрия Любимова из СССР. Источник: "Биография", 2014, N 1. Автор: Анна Топаж Вопрос 24: Внимание, в вопросе есть замены. В ЧУДОВИЩЕ "девочка съела пирожок без СНА РАЗУМА" всё звучит адекватно, а в ЧУДОВИЩЕ "девочка съела пирожок со СНОМ РАЗУМА" концовка режет слух. Какое слово мы заменили на "СОН РАЗУМА"? Ответ: Спрос. Комментарий: СОН РАЗУМА = спрос, ЧУДОВИЩЕ = предложение. Источник: 1. http://ru.wikipedia.org/wiki/Закон_спроса_и_предложения 2. http://ru.wikipedia.org/wiki/Сон_разума_рождает_чудовищ Автор: Михаил Дворкин Тур: 17 тур. "Мы все умрем" (Зеленоград) Редактор: Роман Царегородцев Инфо: Редактор благодарит за помощь в подготовке пакета Максима Евланова, Григория Смыслова, Михаила Лаврина, Степана Кузнецова, Андрея Кудрявцева, Анастасию Ронжину, Алену Абдрахманову, Сергея Шорина, команды "Апостериори" и "Питер ФМ". Вопрос 1: Елена Ларина в своей статье назвала беспилотники армии США ЕЮ военных роботов. Назовите ЕЕ двумя словами. Ответ: Первая ласточка. Комментарий: Дроны - летающие роботы. Источник: "Комсомольская правда", 16-23 января 2014 г. Автор: Михаил Лаврин Вопрос 2: В эпизоде фильма "Астерикс и Обеликс в Британии" Цезарь, уговаривая Астерикса перейти на его сторону, сообщает ему тайну. Назовите человека, который услышал подобные слова в 3 году новой эры. Ответ: Люк Скайуокер. Комментарий: Данный эпизод пародирует серию фильмов "Звездные войны". Цезарь сообщает Астериксу, что тот является его сыном. Во вселенной "Звездных войн" летоисчисление ведется с битвы при Явине (события из самого первого фильма киносаги). Источник: 1. http://ru.wikipedia.org/wiki/Астерикс_и_Обеликс_в_Британии 2. http://ru.wikipedia.org/wiki/Звёздные_войны Автор: Роман Царегородцев Вопрос 3: На обложке одного из номеров журнала "The Economist" [зи икОномист] изображена огненная река, в которой находятся МахмУд АхмадинежАд, БашАр АсАд, Ким Чен Ын и некоторые другие персоны, а ЭТОТ ПОЛИТИК смотрит на них сверху. Назовите ЭТОГО ПОЛИТИКА. Ответ: Ангела МЕркель. Комментарий: Огненная река символизирует ад, среди которого, как считает редакция журнала, находятся упомянутые люди. Логично было бы предположить, что лидер Христианско-демократического союза Германии Ангела МЕркель смотрит на это сверху в образе ангела, но в данном случае это не так. :-) Источник: http://www.newsru.com/world/20dec2012/hell.html Автор: Григорий Смыслов Вопрос 4: По словам историка образования Алексея Любжина, в немецких университетах романтики сначала отставали от реалистов, но затем догоняли и перегоняли их. В этом вопросе мы заменили одно слово. Назовите его. Ответ: Гимназисты. Комментарий: Реалистами называли учащихся или, в данном случае, выпускников реальных училищ, существовавших в России и по сей день существующих в Германии. Источник: http://www.polit.ru/article/2013/04/28/school/ Автор: Денис Иванов Вопрос 5: Пользователь одного из интернет-ресурсов не согласен с предположением о том, что нынешний президент в дальнейшем будет придерживаться более агрессивной политики. При этом он употребляет словосочетание "ТАКОЙ ИКС". Что мы заменили на "ТАКОЙ ИКС"? Ответ: Мягкий путь. Комментарий: "... ведь "дзюдо" - это "мягкий путь"". Комментарий не вполне серьезен. Источник: 1. http://www.odnako.org/blogs/show_35021/comments/ 2. http://ru.wikipedia.org/wiki/Дзюдо Автор: Андрей Кудрявцев Вопрос 6: Комментатор телеканала "Спорт", отмечая уверенное выступление спортсменки, предположил, что она чувствовала себя СОЛОМОНОМ В ПОСУДНОЙ ЛАВКЕ. Какие слова мы заменили на "СОЛОМОН В ПОСУДНОЙ ЛАВКЕ"? Ответ: Королева на льду. Комментарий: Речь шла о выступлении фигуристки. Источник: Трансляция чемпионата России по фигурному катанию в женском одиночном разряде на телеканале "Спорт" от 26.12.2013 г. Автор: Андрей Кудрявцев Вопрос 7: Повествуя о детстве Василия Григорьевича Давиденко, автор статьи о нем упоминает, что его первой книгой был букварь. Какие четыре буквы мы заменили в предыдущем предложении? Ответ: Кобз. Комментарий: Давиденко - украинец, и первой его книгой был, естественно, "Кобзарь" Т. Шевченко. Источник: http://www.odesskiy.com/d/davidenko-vasilij-grigorevich.html Автор: Степан Кузнецов Вопрос 8: В 1981 году в Югославии проводилась перепись населения. Какую национальность указали в анкетах 5,4% опрошенных? Ответ: Югословены. Зачет: Югославы. Комментарий: Остальные назывались конкретнее - сербы, хорваты, черногорцы и т.д. Источник: http://ru.wikipedia.org/wiki/Югославы Автор: Степан Кузнецов Вопрос 9: В вопросе есть две замены. Русский броненосец "Император Павел I" стал первым в мире кораблем, оснащенным гиперболоидными пушками конструкции инженера Гарина. Поняв, что мы заменили в этом вопросе, назовите того, кого мы заменили на Гарина. Ответ: Владимир Григорьевич Шухов. Комментарий: Броненосец "Император Павел I" был оснащен гиперболоидными мачтами конструкции инженера Шухова, напоминающими Шаболовскую башню. Источник: http://ru.wikipedia.org/wiki/Император_Павел_I_(броненосец) Автор: Денис Иванов Вопрос 10: Фигуристка Елена Водорезова стала участницей Олимпийских игр в очень юном возрасте. Рассказывают, что именно из-за нее пришлось вводить в анкете дополнительный вариант в графЕ. Назовите эту графу. Ответ: Партийная принадлежность. Комментарий: В советское время, для того чтобы попасть на международные соревнования, нужно было заполнять огромное количество анкет. В 1976 году Водорезовой было 12 лет, и в графе "Партийная принадлежность" она указала "Пионер". Источник: Репортаж о соревнованиях по фигурному катанию среди женщин на чемпионате Европы 2014 года на канале "Россия 2" от 17.01.2014 г. Автор: Михаил Лаврин Вопрос 11: ОН входил в сервировку будапештских ресторанов 1980-х и помогал скрасить ожидание заказа. Назовите ЕГО двумя словами. Ответ: Кубик Рубика. Комментарий: В это время данная головоломка была невероятно популярна. Источник: http://ru.wikipedia.org/wiki/Кубик_Рубика Автор: Степан Кузнецов Вопрос 12: Петр Великий создал первую в России тайную полицию. Говоря о Петре Андреевиче Толстом, Леонид Парфёнов назвал его первым человеком с ЭТИМ. Назовите ЭТО шестью словами. Ответ: Холодная голова, горячее сердце, чистые руки. Зачет: Указанные три словосочетания в любом порядке. Комментарий: Тайная полиция была фактически прародителем различных спецслужб. Источник: Цикл "Российская империя", часть вторая, 29-30-я минуты. Автор: Григорий Смыслов Вопрос 13: Дуплет. В одном абсурдистском мультфильме показывается восстание морских обитателей против людей. 1. В одном из эпизодов ОН уходит из суши-бара, даже не поблагодарив. Назовите ЕГО. 2. В одном из эпизодов ОНА избивает сама себя. Назовите ЕЕ. Ответ: 1. Дельфин. 2. Русалка. Комментарий: 1. В произведении Дугласа Адамса "Автостопом по галактике" дельфины хотя бы поблагодарили за рыбу. 2. Рыбий хвост русалки начинает душить человеческую половину. В песне Игоря Николаева и Наташи Королёвой дельфин и русалка были друг другу не парой, а у нас встретились в одном дуплете. :-) Источник: http://www.youtube.com/watch?v=_TWlarOIGT4 Автор: Григорий Смыслов Вопрос 14: В вопросе есть замены. Перед первым показом в СССР название ОДНОГО фильма перевели дословно и добавили слова "но достойно". В рецензии на сайте kinopoisk.ru говорится, что ДРУГОМУ фильму, вышедшему за два месяца до ввода войск в Чехословакию, ПРИШЛОСЬ ТЯЖЕЛО. Какие три слова мы заменили на "ПРИШЛОСЬ ТЯЖЕЛО"? Ответ: Досталось на орехи. Комментарий: Фильм с Брюсом Уиллисом в видеосалонах прошел под названием "Умри тяжело, но достойно", а одноименному фильму Теодора Вульфовича досталось на орехи от цензоров. Источник: 1. http://www.kinopoisk.ru/film/471/ 2. http://www.kinopoisk.ru/review/838060/ Автор: Роман Царегородцев Вопрос 15: Призыв ко всему "рабочему народу" появился только в русской версии одной революционной песни. А к кому он был обращен в оригинале? Ответ: К жителям Варшавы. Комментарий: "Варшавянка" же. Источник: http://ru.wikipedia.org/wiki/Варшавянка Автор: Степан Кузнецов Вопрос 16: Описывая разницу между двумя языками, знакомый автора вопроса пошутил, что в АЛЬФЕ целых четыре ИКСА. Какие слова мы заменили на АЛЬФУ и на ИКС? Ответ: Решетка, плюс. Комментарий: Имелись в виду языки программирования C# и C++. Знак решетки можно разбить на четыре плюса. Автор: ??? Вопрос 17: Самолет президента Аргентины имеет позывной Тэ01. Зачитывая этот вопрос, я кое-что произнес неправильно. Напишите слово, которое мне следовало сказать. Ответ: Танго. Комментарий: Буква Т в позывном президентского самолета означает "Транспорт". В фонетическом алфавите НАТО, используемом в авиационном радиообмене, она читается как Танго. Танго - национальный танец Аргентины. Источник: http://en.wikipedia.org/wiki/Agrupaci%C3%B3n_A%C3%A9rea_Presidencial Автор: Денис Иванов Вопрос 18: Рассказывают, что один актер после спектакля утверждал, что смог бы отлично сыграть роль Ивана Грозного, если бы у него был нужный костюм. Назовите фамилию персонажа, которого играл актер в этом спектакле. Ответ: Нагой. Комментарий: Михайло Нагой - брат царицы Марии Федоровны, персонаж пьесы А.К. Толстого "Смерть Иоанна Грозного". Источник: 1. http://subscribe.ru/archive/rest.funny.news.baiki/201311/10234948.html 2. http://ru.wikipedia.org/wiki/Смерть_Иоанна_Грозного_(пьеса) Автор: Михаил Лаврин Вопрос 19: В одном мультфильме парочка людей пытается спастись на крыше от восставших. Однако их давит ОН. Назовите ЕГО коротким словом. Ответ: Кит. Комментарий: Падает с неба и давит. Это всё тот же абсурдистский мультфильм про восстание морских обитателей. :-) Здесь снова прослеживается аллюзия на роман Дугласа Адамса "Автостопом по галактике". Источник: http://www.youtube.com/watch?v=_TWlarOIGT4 Автор: Григорий Смыслов Вопрос 20: По одной из версий, ИМ был Сальери. Назовите ЕГО двумя словами, начинающимися на одну и ту же букву. Ответ: Черный человек. Комментарий: Не убил Моцарта буквально, но опосредовано поспособствовал. Источник: 1. Фильм "Амадей", реж. М. Форман. 2. ЛОАВ. Автор: Григорий Смыслов Вопрос 21: На самом деле известный флаг состоит из шести полос. Автор вопроса удивился отсутствию на нем именно этого цвета. Какого? Ответ: Голубого. Источник: http://ru.wikipedia.org/wiki/ЛГБТ-движение Автор: Степан Кузнецов Вопрос 22: На одном сайте психологические точки опоры назвали ИМИ, которые не нужны здоровому человеку. В этом вопросе ИХ нет. Точнее, один есть. Назовите ЕГО. Ответ: Костыль. Комментарий: Костыль - это тоже точка опоры. Фраза "точнее, один есть" сама по себе является в этом вопросе костылем. Источник: http://www.improve-me.ru/10490 Автор: Григорий Смыслов Вопрос 23: Персонаж романа Айн Рэнд - банкир, всегда успешно вкладывавший капитал в предприятия, которые впоследствии приносили ему большую прибыль. Герой даже официально поменял свое имя на полученное от журналистов прозвище ИКС. В историях о ИКСЕ часто упоминают купание в реке. Какое слово мы заменили на ИКС? Ответ: Мидас. Комментарий: Любой бизнес, к которому он прикладывал руку, становился источником богатства. По одной из легенд, Мидас искупался в реке Пактол и излечился, а в ней с тех пор находят частички золота. Источник: 1. А. Рэнд. Атлант расправил плечи. 2. http://ru.wikipedia.org/wiki/Мидас Автор: Алена Абдрахманова Вопрос 24: <раздатка> За несколько лет Шухов спроектировал и построил их сотни, что привело к частичной типизации самой конструкции и ее отдельных элементов - лестниц и резервуаров. При этом АЛЬФ у Шухова не было. Перед вами фрагмент статьи из журнала "Наука и жизнь". Другие АЛЬФЫ различались по одному из параметров на два метра. Какое слово мы заменили на АЛЬФЫ? Ответ: Башни-близнецы. Комментарий: Несмотря на широкое распространение знаменитой конструкции, у Шухова нельзя найти двух одинаковых башен. В раздаточном материале речь идет о водонапорных башнях. Башни-близнецы ВТЦ были очень похожи друг на друга, хоть и различались по высоте на два метра. Источник: 1. "Наука и жизнь", 2013, N 12. - С. 104. 2. http://ru.wikipedia.org/wiki/Всемирный_торговый_центр Автор: Андрей Кудрявцев Тур: 18 тур. "Запущинный ИКС" (Пущино - Серпухов) Вопрос 1: (pic: 20140050.jpg) Надеемся, что все вы отключили мобильные телефоны. Восстановите подпись под демотиватором, состоящую из двух слов. Ответ: Батарея разряжена. Источник: http://pikabu.ru/story/batareya_razryazhena_1709301 Автор: Алексей Дубровский Вопрос 2: Как ни странно, стандарт записи компакт-дисков, разработанный компаниями "Philips" и "SONY" в 1980 году, весьма распространен. Напишите его название двумя русскими словами, начинающимися на одну и ту же букву. Ответ: Красная книга. Источник: 1. http://ru.wikipedia.org/wiki/Звуковой_компакт-диск 2. http://ru.wikipedia.org/wiki/Красная_книга Автор: Ирина Алисова Вопрос 3: В комиксе на сайте http://kolyan.net в уста известного мультипликационного героя вложено четверостишие, одна из строк которого звучит так - "Из-за женщин даже логика загнется". Назовите этого героя. Ответ: Крошка Енот. Комментарий: Полностью строфа звучит так: "А от женщин станет всё сложней, Из-за женщин даже логика загнется, Ты расстался с бывшею своей, Но она еще не раз к тебе вернется". Источник: http://www.kolyan.net/index.php?newsid=49467 Автор: ??? Вопрос 4: (pic: 20140051.jpg) Ответьте двумя словами, начинающимися на одну и ту же букву, что, по мнению автора вопроса, нежелательно делать при помощи приспособления, изображенного на картинке. Ответ: Солить свинину. Комментарий: Расположение дырочек в крышке напоминает Звезду Давида. Солить свинину из такой солонки не кошерно. :-) Источник: ЛНА. Автор: Алексей Дубровский Вопрос 5: Статья на сайте sports.ru, посвященная хоккейному матчу между женскими сборными командами Канады и США, в котором вспыхнула массовая драка, была озаглавлена "[ПРОПУСК] тебе выдеру. Как женские сборные США и Канады готовятся к Олимпиаде". Заполните пропуск укороченным названием печатного органа. Ответ: Cosmo. Источник: http://www.sports.ru/tribuna/blogs/hockeyisoutthere/519583.html Автор: Алексей Дубровский Вопрос 6: Его русское название идет от заимствованного глагола со значением "открывать", хотя это было прерогативой его предшественника. Его турецкое название нам известно из популярного источника. Напишите оба названия. Ответ: Апрель, нисан. Комментарий: Год раньше открывался мартом. Про месяц нисан знает всякий, кто читал "Мастера и Маргариту". Автор: ??? Вопрос 7: По версии "Каталога невозможных объектов" Карельмана, ОН может быть прозрачным, чтобы не мешать любоваться окружающим пейзажем, раздвигаться как купол астрономической обсерватории, чтобы просунуть трубу и продолжить наблюдения, а также, благодаря своей форме, собирать драгоценную воду в засушливом климате. Назовите ЕГО. Ответ: Зонт. Источник: http://kvant.mccme.ru/1992/07/iz_kataloga_nevozmozhnyh_obekt.htm Автор: Ирина Алисова Вопрос 8: (pic: 20140052.jpg) Назовите двумя словами устройство, на котором можно видеть скрытую от вас надпись. Ответ: Струйный принтер. Комментарий: Скрытая надпись - "Чернила заканчиваются". Источник: (pic: 20140053.jpg) Автор: Алексей Дубровский Вопрос 9: Название спектакля нижегородского театра "Комедiя", поставленного по ранним рассказам русского писателя, совпадает с названием детской подвижной игры. Первая и последняя буквы этого названия совпадают с первой и последней буквой слова, с которым упомянутый писатель совершил любопытную манипуляцию. Напишите оба слова, загаданные в вопросе. Ответ: "Чехарда", чепуха. Источник: (pic: 20140054.jpg) Автор: Данила Аладин Вопрос 10: Короткое существительное в названии одного череповецкого вино-водочного магазина наводит на мысль о возможных последствиях благодаря выражению, в котором это существительное превращается в другую часть речи. В этом же выражении упоминается герой. Назовите этого героя. Ответ: Ихтиандр. Комментарий: Магазин - "Вечный зов". Сильно перепив, порой зовут Ихтиандра. Автор: ??? Вопрос 11: Заголовок статьи на сайте serpregion.ru, посвященной сухому поражению волейбольной команды "Надежда", отличается от устойчивого выражения добавлением отрицания. Воспроизведите название статьи. Ответ: "Опять не двадцать пять". Комментарий: Партии в волейболе проходят до 25 очков. При сухом поражении "Надежда" так ни разу 25 очков не набрала. Источник: http://www.serpregion.ru/content/view/14772/ Автор: Олег Холодов Вопрос 12: (pic: 20140055.jpg) Название известного романа отличается от подписи к данной картинке так: первое слово осталось без изменения, второе заменено антонимом, третье разбито на два с почти полным сохранением созвучия. Назовите автора романа. Ответ: Милан Кундера. Комментарий: Произведение - "Невыносимая легкость бытия", подпись к картинке - "Невыносимая тяжесть быть Иа". Источник: 1. http://pikabu.ru/story/nevyinosimaya_tyazhest_1406660 2. http://ru.wikipedia.org/wiki/Невыносимая_лёгкость_бытия Автор: Алексей Дубровский, Мария Томашевская Вопрос 13: Прослушайте шуточное стихотворение Джеймса Линдона: "Плотная пища у жен [ПРОПУСК] Только на пользу им шла, не иначе. Весили жены, согласно молве, Каждая - как предыдущие две". Заполните пропуск прозвищем. Ответ: Фибоначчи. Автор: Глеб Крутинин Вопрос 14: Ампутировав собственную руку, в которую вселилось нечто, Эш, главный герой фильма "Армия тьмы", обнаружил, что возможности функционировать это ее не лишило. Руку Эш засунул под ведро, на которое для надежности наложил книжек. Назовите англоязычный роман, который находился сверху стопки. Ответ: "Прощай, оружие!". Комментарий: По-английски роман называется "A Farewell to Arms". Источник: 1. Фильм "Армия тьмы". 2. http://lurkmore.to/Фильмы_эпохи_VHS Автор: Глеб Крутинин Вопрос 15: Некто закачал себе на iPad приложение Google Sky Map, позволяющее рассматривать звездное небо. Сообщение об этом на сайте http://bash.im прокомментировано известной песенной строкой. Какой временной промежуток упоминается в названии песни? Ответ: 14 минут. Комментарий: "Заправлены в планшеты космические карты". iPad представляет собой планшетный компьютер. Источник: http://bash.im/quote/422791/ Автор: Данила Аладин Вопрос 16: (pic: 20140056.jpg) Перед вами несколько необычная фотография Диты фон Тиз. Назовите место рождения той, чье имя указано в подписи к данной фотографии. Ответ: Кипр. Комментарий: На данном фото Дита изображена в виде негритянки, поэтому она афро-Дита. Источник: http://pikabu.ru/story/afrodita_718758 Автор: Мария Томашевская, Алексей Дубровский Вопрос 17: Знакомый автора вопроса пошутил, что будет звать свою кошку Фабия. Воспроизведите аргументацию знакомого двумя именами. Ответ: Маленькая шкода. Комментарий: "Фабия" - это небольшая модель автомобиля "Шкода". Ну а кошка, видимо, частенько шкодит. Имена - существительное и прилагательное. Автор: Данила Аладин Вопрос 18: Левый глаз Гора был выбит в схватке с Сетом и рассеян на части. Все они были собраны Тотом, и лишь самая маленькая не нашлась. А что изображали египтяне при помощи частей глаза Гора? Ответ: Дроби. Источник: http://ru.wikipedia.org/wiki/Уаджет Автор: Ирина Алисова Вопрос 19: В Индии была распространена словесная система нумерации, в которой цифрам соответствовали слова. Воспроизведите число, записанное таким способом, как "глаза-небо-луна-касты". Ответ: 2014. Источник: А.И. Володарский. Очерки истории средневековой индийской математики. - М.: Либроком, 2009. - С. 24. Автор: Ирина Алисова Вопрос 20: <раздатка> У быдла есть особенность: оно - всегда на ты. И это восхищает. Какую букву мы заменили в высказывании Олега Ладыженского? Ответ: е. Комментарий: "У быдла есть особенность: оно - всегда не ты. И это восхищает". Источник: Олег Ладыженский. Мост над океаном. Автор: Глеб Крутинин Вопрос 21: Среди многочисленных потомков известного человека есть и Карл Филипп Эммануил, известный как петух гамбургский. Какие буквы и на какие мы заменили в этом вопрос? Ответ: "Ба" на "пету". Комментарий: На самом деле он Бах Гамбургский. Источник: http://ru.wikipedia.org/wiki/Бах,_Карл_Филипп_Эммануил Автор: ??? Вопрос 22: Аллюзия на вопрос телефонного чемпионата 1994 года. Этих двоих отличает нота, а также то, что один уместен на детском празднике, а другой - на взрослом. Назовите их обоих. Ответ: Аниматор и реаниматор. Автор: ??? Вопрос 23: То, что стало, по мнению американской компании "SplashData", худшим паролем 2013 года, более 30 лет назад принесло успех. Назовите источник, благодаря которому стало известно об этом успехе. Ответ: [Фильм] "Спортлото-82". Комментарий: Худшим паролем был признана комбинация 123456, но эти же шесть цифр, выбранных героями популярной комедии, принесли выигрыш в лотерею. Источник: http://lenta.ru/news/2014/01/20/worst1/ Автор: ??? Вопрос 24: Он дал свое имя термину, означающему полный цикл работы с клиентом, что включало и длительный душевный разговор. Результат этой работы также может носить его имя и в любом случае сопровождается емкой формулой. Какой? Ответ: "Помой и иди". Источник: http://www.1tvnet.ru/content/show/vidal-sassun-biografiya-vidayuschegosya-parikmahera_11430.html Автор: ??? Тур: 19 тур. "Выезд апартаменты" (Москва) Редактор: Дмитрий Дягилев Инфо: Редактор благодарит за помощь в подготовке пакета, тестирование, ценные советы и замечания Павла Зельтера и Григория Калихмана (Самара), Элинор Гейстрих (Ришон ле-Цион), Юлию Лободу (Томск), Екатерину Дубровскую (Омск), Алексея Батькова (Екатеринбург), Владимира Богданова (Бремен), Елену Анциферову (Томск), Дарью Паренкину (Тюмень), Галину Воловник (Рыбница), Андрея Кокуленко (Омск). Вопрос 1: В статье на сайте tjournal.com [ти джоурнал точка ком] рассказывается о математике, который, используя статистические методы на сайте знакомств, смог найти себе жену. Один из пользователей прокомментировал статью, назвав произошедшее с математиком устойчивым выражением из трех слов. Воспроизведите это выражение. Ответ: Брак по расчету. Комментарий: Герой очень долго высчитывал, какой должна быть идеальная жена, прежде чем вступить в брак. Источник: https://tjournal.ru/49444-lovehacker Автор: Дмитрий Дягилев Вопрос 2: В песне Ланы дель Рей поется об эффектной девушке, к ногам которой мужчины падают один за другим. С чем в песне сравниваются эти мужчины? Ответ: С костяшками домино. Зачет: По слову "домино". Комментарий: "Эффектная" и "один за другим" - подсказки. Источник: http://www.amalgama-lab.com/songs/l/lana_del_rey/lolita.html Автор: Игорь Яшков Вопрос 3: Ведущие программы "Говорим по-русски" для запоминания разницы между ЭТИМИ глаголами рекомендуют вспоминать слова "резервИровать" и "никелировАть". Напишите любой из этих глаголов. Ответ: Бронировать. Комментарий: Так рекомендуется запоминать ударение: бронИровать - резервИровать и бронировАть- никелировАть. Источник: Программа "Говорим по-русски" на радио "Эхо Москвы" от 03.11.2013 г. Автор: Павел Зельтер Вопрос 4: (pic: 20140057.jpg) Перед вами афиша фильма "Да здравствуют антиподы!". Какие два символа на ней мы скрыли? Ответ: ¡ ! Зачет: Перевернутый восклицательный знак, восклицательный знак. Комментарий: Фильм снят в Аргентине, афиша написана на испанском, а название фильма явно представляет собой восклицательную фразу. Пропущенные знаки тоже являются в некотором смысле антиподами. Источник: http://www.kinopoisk.ru/film/578942/posters/ Автор: Игорь Яшков Вопрос 5: Анна ДыбО, рассказывая о базовой докультурной лексике языка, упоминает слова "птица", "рыба" и "АЛЬФА". Средневековый поэт воспел АЛЬФ на кудрях своей возлюбленной. Назовите АЛЬФУ. Ответ: Вошь. Зачет: Вша, блоха. Комментарий: Вошь - чуть ли не самый древний спутник человека (вплоть до того, что изменение генома вши плотно коррелирует с изменением генома представителей отряда homo). Неудивительно, что слово "вша" появилось в языке довольно рано. В средневековье проблема оставалась достаточно актуальной. Источник: 1. http://www.youtube.com/watch?v=0LwBXRNHQqU 2. http://ru.wikipedia.org/wiki/Вши Автор: Дмитрий Дягилев Вопрос 6: В одном произведении герой обещает бросить курить, но на самом деле ОБМАНЫВАЕТ. Какие четыре слова мы заменили в предыдущем предложении? Ответ: Пускает дым в глаза. Комментарий: "Пускать дым в глаза" - синоним слова "обманывать". Источник: http://www.ershik.com/issues/2-july-2013/July%202013%20-%20Ershik.pdf Автор: Дмитрий Дягилев Вопрос 7: (pic: 20140058.jpg) Перед вами иллюстрация к статье о реорганизации компании. Напишите название этой компании. Ответ: "Microsoft". Комментарий: На логотипе "Microsoft" изображены четыре квадрата: красный, синий, зеленый и желтый. Источник: http://habrahabr.ru/company/apps4all/blog/184478/ Автор: Игорь Яшков Вопрос 8: В пьесе "Собака на сене" Тристан, переодетый иностранным купцом, говорит, что его отец увез детей в ЭТУ далекую страну. Возможно, именно это повлияло на выбор актера для советского фильма. Назовите эту страну. Ответ: Армения. Комментарий: Тристана в фильме играет Армен Джигарханян. Источник: 1. http://lib.ru/DEVEGA/vega2.txt 2. http://ru.wikipedia.org/wiki/Собака_на_сене_(фильм) Автор: Павел Зельтер Вопрос 9: В одном рассказе при описании момента, следовавшего за разоблачением главного героя, упоминается упавшая на ковер английская булавка. В известной песне в описании аналогичного момента упоминается ОН. Первый ОН появился в Англии. Назовите ЕГО двумя словами. Ответ: Вагон метро. Зачет: Вагон метрополитена. Комментарий: Главного героя-обманщика разоблачили на глазах у большого круга людей, у которых от удивления пропадает дар речи. В воцарившейся тишине, по словам автора, можно было услышать, как английская булавка падает на ковер. В песне "Романс" группы "Сплин" лирический герой, описывая воцарившуюся тишину, говорит, что слышит, как идет на глубине вагон метро. Первое метро появилось в Англии во второй половине XIX века. Источник: 1. Э. По. Вильям Вильсон. 2. http://megalyrics.ru/lyric/splin/romans.htm 3. http://en.wikipedia.org/wiki/London_Underground Автор: Дмитрий Дягилев Вопрос 10: По воспоминаниям Юрия Никулина в цирке выступали воздушные гимнасты, которые, несмотря на свою малообразованность, подобрали для своего номера подходящее короткое название. По одной версии, название родилось в пивной после того, как гимнасты увидели отражение в стекле. Напишите это название. Ответ: "Икар". Комментарий: В пивной они увидели отражение надписи "Раки". Источник: Ю. Никулин. Почти серьезно. Автор: Павел Зельтер Вопрос 11: <раздатка> Vertigo Самое известное ЕЕ изображение появилось в книге издательства "Vertigo" [вЕртиго]. Дэвид Ллойд сравнил ЕЕ со знаменитой фотографией Че Гевары, которая долгое время была символом протеста молодежи. Назовите ЕЕ. Ответ: Маска Гая Фокса. Комментарий: "Vertigo" тоже начинается на букву "V". Маска Гая Фокса стала одним из символов протестного интернет-движения Anonymous. Источник: 1. http://en.wikipedia.org/wiki/Alan_Moore 2. http://en.wikipedia.org/wiki/Guy_Fawkes_mask Автор: Дмитрий Дягилев Вопрос 12: По одной версии, идея ЕЕ создания была взята из романа "Криптономикон". На данный момент с НЕЙ работают два казино в Лас-Вегасе. Напишите название самой распространенной из НИХ. Ответ: Биткоин. Комментарий: ОНА - цифровая валюта, или, по-другому, криптовалюта. Пока массового хождения она не получила, поэтому с ней согласились работать только лишь два казино. Объемы сделок на бирже биткоинов - самые большие из всех цифровых валют. Источник: 1. http://habrahabr.ru/post/99100/ 2. http://www.bfm.ru/news/244364?doctype=article Автор: Дмитрий Дягилев Вопрос 13: Внимание, слово "АЛЬФА" в вопросе является заменой. Лев Гумилев пишет о том, что аристократическая верхушка Хазарского каганата исповедовала иудаизм, в то время как большая часть населения была язычниками. В связи с этим Гумилев уподобляет Хазарский каганат АЛЬФЕ. По одной легенде, родиной АЛЬФЫ является курортный город. Назовите этот город. Ответ: Кемер. Комментарий: Гумилев сравнивает Хазарский каганат с этнокультурной химерой, у которой голова исповедует иудаизм, а остальное тело - язычество. Родиной Химеры, по одной из легенд, является созвучный с ее именем город Кемер. Источник: 1. http://www.flibusta.net/b/196882/read 2. http://ru.wikipedia.org/wiki/Химера Автор: Дмитрий Дягилев Вопрос 14: В труде Тацита "О происхождении германцев..." описывается некий обычай германских племен казнить предателей и самоубийц. Этот обычай впоследствии послужил причиной феномена ИХ. ОНИ стали фигурировать в отечественной прессе после 5 декабря 2011 года. Назовите ИХ двумя словами. Ответ: Болотные люди. Комментарий: "Благодаря" такому обычаю в двадцатом веке на территории Северной Европы было найдено более тысячи прекрасно сохранившихся мумий (самая известная - человек из Толлунда), получивших общее название - болотные люди. Болотными людьми в отечественной прессе стали называть участников митингов на Болотной площади, первый из которых состоялся 5 декабря 2011 года. Источник: 1. http://www.hist.msu.ru/ER/Etext/tacit.htm 2. http://www.nkj.ru/archive/articles/21728/ 3. http://ru.wikipedia.org/wiki/Протестное_движение_в_России_(2011%E2%80%942013) Автор: Дмитрий Дягилев Вопрос 15: Согласно легенде, двое влюбленных никак не могли найти уединенное место на узких улочках. Над ними сжалился месяц, и так появилась ОНА. Назовите ЕЕ. Ответ: Гондола. Незачет: Лодка. Комментарий: Согласно венецианской легенде, месяц спустился на воду к влюбленным, предоставив им место для уединения. Источник: "Вокруг света", 2013, N 7. Автор: Игорь Яшков Вопрос 16: Одно из значений термина "bootstrapping" [бутстрэппинг] - способ финансирования бизнеса без привлечения каких-либо внешних источников. Назовите фамилию персонажа, которая упоминается в статье Википедии, объясняющей этот термин. Ответ: Мюнхгаузен. Комментарий: Термин описывают процесс, который поддерживает и развивает сам себя - подобно Мюнхгаузену, вытащившему себя за волосы из болота. Источник: http://en.wikipedia.org/wiki/Bootstrapping Автор: Игорь Яшков Вопрос 17: В одном из отзывов о картине Шишкина, найденном в Интернете, подчеркивается спокойствие и умиротворение, [начало цитаты] "будто бы в месте, изображенном на полотне, никогда ничего не происходило" [конец цитаты]. Напишите название этой картины. Ответ: "Камень в лесу". Зачет: "Камни в лесу", "Валаам. Камни в лесу". Комментарий: "Камень в лесу" - известный твиттер-аккаунт, который якобы ведет лесной булыжник. Каждый день в этом твиттере появляется равно одна запись - "Сегодня ничего не произошло". Шишкин, как известно, рисовал лесные пейзажи. Картина представляет собой изображение нескольких камней в лесу, покрытых толстым слоем мха. Источник: 1. http://www.museum-online.ru/Peredvizhniki/ivan_ivanovich_shishkin 2. https://twitter.com/kamen_v_lesu Автор: Дмитрий Дягилев Вопрос 18: Владимир Найдин в своей книге рассказывает о чемпионке мира по некоему виду спорта. Чемпионка утверждала, что могла по запаху безошибочно определить, когда в магазине тканей ее пытались обмануть, подсунув акрил. Назовите вид спорта, которым она занималась. Ответ: Парашютный спорт. Комментарий: Так как она постоянно имела дело с шелком, который она определяла по запаху. Источник: В. Найдин. Реанимация. Автор: Павел Зельтер Вопрос 19: (pic: 20140059.jpg) Перед вами норвежская гора Тургхаттен, в которую, согласно легенде, превратилась АЛЬФА. Назовите АЛЬФУ двумя словами. Ответ: Шляпа тролля. Комментарий: Гора по форме напоминает шляпу; "hatten" - по-норвежски "шляпа"; очевидно, каменеют на солнце не только норвежские тролли, но и их предметы одежды. Источник: 1. http://www.vokrugsveta.ru/vs/article/8481/ 2. http://ru.wikipedia.org/wiki/Тургхаттен Автор: Игорь Яшков Вопрос 20: В конце новогодних праздников автор вопроса наблюдал очередь у входа в НЕГО. Название одного из НИХ - "Раскольников". Назовите ЕГО. Ответ: Ломбард. Комментарий: К концу длинных новогодних праздников люди пропили все деньги и выстроились в очередь, чтобы дать в залог какую-либо ценную вещь. Ломбард "Раскольников" действительно существует! Источник: 1. ЛОАВ. 2. http://ru-marazm.livejournal.com/2862672.html Автор: Дмитрий Дягилев Вопрос 21: Иридиевые аномалии - тонкие пласты глины на большой глубине, с содержанием иридия в 300 раз превышающим норму. По одной версии, причиной этих аномалий являются ОНИ. Согласно шутке, появившейся около года назад, с утра лучше всего бодрит ОН. Назовите ЕГО. Ответ: Метеорит. Комментарий: Некоторые ученые объясняют такие точечные скопления иридия в почве падением металлических метеоритов. "Ничего так не бодрит, как с утра метеорит" - шутка, появившаяся в феврале прошлого года после падения метеорита в Челябинске. Источник: 1. http://www.bibliotekar.ru/index.files/2/0-87.htm 2. http://www.anekdot.ru/id/624882/ Автор: Дмитрий Дягилев Вопрос 22: Автор статьи в журнале "Вокруг света", рассказывая о способе нанесений наскальной живописи, упоминает полые кости птиц в качестве прототипа ЕГО. Назовите ЕГО. Ответ: Аэрозольный баллончик. Зачет: Баллончик с краской, пульверизатор и т.п. по смыслу. Комментарий: Древние художники, предположительно, набирали краску в рот и распыляли через полые трубочки, сделанные из птичьих костей. Источник: "Вокруг света", 2013, N 7. Автор: Игорь Яшков Вопрос 23: Внимание, ИКС и КСИ в вопросе являются заменами. На последних парламентских выборах ИКС прошел в КСИ, набрав 14%. Назовите лидера ИКСА. Ответ: [Виталий] Кличко. Незачет: Владимир Кличко. Комментарий: Партия "Удар" прошла в украинскую раду. Лидер партии "Удар" - известный боксер. Источник: http://ru.wikipedia.org/wiki/Парламентские_выборы_на_Украине_(2012) Автор: Дмитрий Дягилев Вопрос 24: В название гран-при фестиваля авангардного видео, проводящегося в ЭТОМ городе Красноярского края, входит слово "секатор". Напишите название этого города. Ответ: Канск. Комментарий: В городе Канске проводится Международный Канский Видео Фестиваль. В противовес кинофестивалю в Каннах главным призом фестиваля является "Золотой пальмовый секатор". Источник: http://ru.wikipedia.org/wiki/Международный_канский_фестиваль_видео Автор: Павел Зельтер Тур: 20 тур. "Вересковая пустошь" (сборная) Редактор: Николай Лёгенький (Минск) Вопрос 1: Закончите двумя словами короткое стихотворение с нестандартным ритмом: пришел бетховену по почте какой-то странный коробок с письмом дарю тебе нужнее ... Ответ: ван гог. Комментарий: Безухий - глухому. Источник: http://vk.com/perawki?w=wall-28122932_33606 Автор: Александр Фокин (Челябинск) Вопрос 2: <раздатка> 7 5 3 1 9 0 2 4 6 8 Перед вами расположение цифровых клавиш в раскладке Дворака, созданной в 1932 году. Причиной асимметрии в расположении четных и нечетных цифр, по одной из версий, стал ОН. Назовите ЕГО, используя два одинаковых символа. Ответ: XX век. Комментарий: Подавляющее большинство дат начиналось с сочетания цифр 1 и 9, поэтому они и были расположены рядом. Источник: 1. http://en.wikipedia.org/wiki/Dvorak_Simplified_Keyboard#Original_Dvorak_layout 2. http://habrahabr.ru/post/210186/ Автор: Дмитрий Карякин (Москва) Вопрос 3: Лавку, находящуюся на пересечении улиц, Гоголь называет ТАКОЙ. "ТАКАЯ рыба" - другое название черной трески. Какое слово мы заменили словом "ТАКАЯ"? Ответ: Угольная. Комментарий: Вместо "угловая" Гоголь написал "угольная", но его все равно считают русским писателем. Источник: Н. Гоголь. Мертвые души. http://www.flibusta.net/b/140158/read Автор: Станислав Алимпов (Витебск - Минск) Вопрос 4: Черная треска считается ценной промысловой рыбой. Ее спина черного цвета, а живот - почти белый. Другое название черной трески - рыба-маршрутка. Какое слово мужского рода мы заменили словом "маршрутка"? Ответ: Соболь. Комментарий: Окрас черной трески напоминает соболя. "Соболь" - марка автомобилей ГАЗ. Источник: http://www.ooozm.by/ryba-svezhemorozhenaya/item/chernaya-treska.html Автор: Николай Лёгенький (Минск) Вопрос 5: Уважаемые знатоки! Задумывались ли вы о том, чего у вас никогда не будет? "Эйфелевой башней" называется фигура, которая получится, если два парня, занимающиеся сексом с одной девушкой в определенном положении, СДЕЛАЮТ ЭТО друг с другом. Джимми Карр не советует ДЕЛАТЬ ЭТОГО с раввином. Ответьте двумя короткими английскими словами: СДЕЛАТЬ ЧТО? Ответ: High five. Комментарий: Во втором случае это может напомнить нацистское приветствие. В первом случае парень, которому девушка делает минет, делает high five парню, который входит в эту девушку сзади. Источник: 1. http://www.urbandictionary.com/define.php?term=the%20eiffel%20tower 2. https://twitter.com/JoeRaggett/status/253219190725550081 Автор: Алексей Бороненко (Челябинск - Москва) Вопрос 6: Герой комикса "Малыш Эбнер" очень переживал, что его дочь превратится в старую деву. Именем этой дочери в США называют мероприятия, состоящие только из НИХ. Назовите ИХ двумя словами. Ответ: Белые танцы. Источник: 1. http://en.wikipedia.org/wiki/Sadie_Hawkins_dance 2. https://web.archive.org/web/20130425224828/http://www.lil-abner.com/sadiehawk.html Автор: Егор Игнатенков (Москва) Вопрос 7: Персонаж Джеффа Бриджеса собирается убить гениального инженера, чье последнее на тот момент изобретение он называет его ЛЕБЕДИНОЙ ПЕСНЕЙ. Что мы заменили словами "ЛЕБЕДИНАЯ ПЕСНЯ"? Ответ: Девятая симфония. Комментарий: Намек на так называемое проклятие девятой симфонии: многие композиторы умирали, начиная создавать свою десятую или так и не закончив девятую симфонию. Источник: 1. "Iron Man" (2008), реж. Дж. Фавро. 2. http://en.wikipedia.org/wiki/Curse_of_the_ninth Автор: Алексей Бороненко (Челябинск - Москва) Вопрос 8: Герой исторического романа - монах посыпает место привала предусмотрительно собранной дорожной пылью. В какой стране он ее собрал? Ответ: В Ирландии. Комментарий: Монаху казалось, что земля Ирландии будет защищать его от змей даже в Альпах. Источник: Е. Водолазкин. Лавр. http://www.flibusta.net/b/310583/read Автор: Егор Игнатенков (Москва) Вопрос 9: Персонаж Нила Геймана в результате сделки получает возможность жить до тех пор, пока ему не надоест. Назовите фамилию человека, которого он встречает в трактире в 1589 году. Ответ: Марло. Комментарий: Примерно такая же история произошла и с Фаустом, о котором Марло написал пьесу. Источник: N. Gaiman "Sandman", #12, p. 9. Автор: Дмитрий Карякин (Москва) Вопрос 10: Рассказывают, что в Средние века киевляне умышленно продалбливали колдобины на дороге, проходящей через Золотые Ворота. Какую пословицу упомянул в связи с этим писатель Евгений Водолазкин? Ответ: Что с воза упало, то пропало. Комментарий: По законам того времени, упавшее на землю переходило в собственность хозяина земли. Поэтому неровные дороги были на руку киевлянам. Источник: Е. Водолазкин. Лавр. http://www.flibusta.net/b/310583/read Автор: Егор Игнатенков (Москва) Вопрос 11: Иногда привычные нам метафоры оказываются совершенно неизвестными в других странах. Так, на американской открытке про отца семейства говорится, что он "могучий, благородный и с каждым годом становится всё лучше". Также на этой открытке изображены ОНИ. Назовите ИХ двумя словами. Ответ: Оленьи рога. Комментарий: Рога благородного оленя тоже растут с каждым годом. Автор: Дарья Бер (Москва) Вопрос 12: Теглайн фильма 1997 года можно перевести следующим образом: "Скоро на экранах. Честно-честно". Назовите этот фильм. Ответ: "Лжец, лжец". Источник: 1. http://www.imdb.com/title/tt0119528/taglines 2. Перевод автора вопроса. Автор: Алексей Бороненко (Челябинск - Москва) Вопрос 13: При определении источника вспышки холеры в 1854 году Джон Сноу сравнивал минимальные расстояния от домов до водоразборных колонок. Это считается одним из первых примеров практического применения метода, названного в честь Овсова. Какую фамилию мы заменили в предыдущем предложении? Ответ: Вороной. Комментарий: Хотя сам Георгий Вороной родился лишь спустя 14 лет после описанных событий, всё же этот метод схож с разбиением плоскости, названным в его честь. Источник: http://en.wikipedia.org/wiki/Voronoi_diagram#Applications Автор: Дмитрий Карякин (Москва) Вопрос 14: Героиню романа "Облачный атлас" зовут Луиза Рей. Кого в интервью журналу "Париж Ревю" Дэвид Митчелл назвал своим любимым писателем? Ответ: [Торнтона] Уайлдера. Комментарий: Известный роман Уайлдера называется "Мост короля Людовика Святого", в оригинале - "The Bridge of San Luis Rey". Источник: http://www.theparisreview.org/interviews/6034/the-art-of-fiction-no-204-david-mitchell Автор: Иделия Айзятулова (Москва) Вопрос 15: [Ведущему: так и сказать - "эээ писательницы", ее-то можно назвать писательницей с большим трудом!] Дипломная работа какой эээ писательницы была посвящена семье Николая II? Ответ: [Эрика Леонард] Джеймс. Зачет: Эрика Митчелл. Комментарий: Поэтому героиню романа "50 оттенков серого" и зовут Анастасия. Источник: http://www.50shadesofgray.ru/nekotorye-fakty-o-trilogii-50-ottenkov.html Автор: Мария Кленницкая (Минск) Вопрос 16: Героиня произведения Томаса Пинчона входит в роль задумчивой и печальной девушки, а спустя некоторое время выбрасывает заколки и шпильки. С кем эту героиню сравнивает Пинчон? Ответ: Рапунцель. Комментарий: Героиня, подобно Рапунцель, стала пленницей и ждет спасителя. Она радостно вынет заколки, шпильки отбросит и вниз беспечально обрушит лавину волос. Источник: 1. Томас Пинчон. Выкрикивается лот 49. http://www.flibusta.net/b/315033/read 2. http://ru.wikipedia.org/wiki/Рапунцель Автор: Вадим Германенко (Витебск - Минск) Вопрос 17: Дипломатическое событие, произошедшее 3-4 июля 1961 года в Вене, Леонид Парфёнов сравнивает с приездом стариков-родителей в столицу на свадьбу разбогатевшего сына и первой красавицы. О каком событии идет речь? Ответ: Встреча Хрущёва и Кеннеди. Комментарий: Речь идет о встрече лидеров двух супердержав и их жен. По словам Парфёнова, советский лидер выглядел потрепанным, в то время как Кеннеди был молодым и красивым. Источник: Л. Парфёнов. Намедни. 1961-1970. Наша эра. Автор: Вадим Германенко (Витебск - Минск) Вопрос 18: [Ведущему: не озвучивать кавычки.] По данным Википедии, примерно каждый пятый житель нашей планеты - китаец. По данным той же Википедии, каждый пятый мусульманин - индонезиец. Согласно сайту президента республики Беларусь, каждый десятый ОН в мире - "беларус". Назовите ЕГО. Ответ: [Новый колесный] трактор. Комментарий: Каждый 10-й выпускаемый в мире трактор носит название "Беларус". Источник: 1. http://ru.wikipedia.org/wiki/Население_Земли 2. http://ru.wikipedia.org/wiki/Хань_(народ) 3. http://ru.wikipedia.org/wiki/Ислам 4. http://ru.wikipedia.org/wiki/Индонезия 5. http://president.gov.by/ru/economy_ru/ Автор: Станислав Алимпов (Витебск - Минск) Вопрос 19: Однажды Александр Лукашенко предложил наложить ПЕРВОЕ на ВТОРОЕ. Назовите и ПЕРВОЕ, и ВТОРОЕ несклоняемыми словами одинаковой длины. Ответ: Вето, табу. Зачет: В любом порядке. Источник: http://www.4vlada.net/vneshnyaya-politika/lukashenko-pora-prinyat-mery-i-nalozhit-veto-na-tabu Автор: Леонид Климович (Гомель) Вопрос 20: Какой ученый, по словам Регины Спектор, помог ей найти няньку для детей и оформить выгодную страховку? Ответ: [Бенджамин] Франклин. Комментарий: Портрет Бенджамина Франклина изображен на банкноте в 100 долларов. Источник: http://www.amalgama-lab.com/songs/r/regina_spektor/chemo_limo.html Автор: Иван Сапего (Брест - Минск) Вопрос 21: Герой комикса "Хранители", узнав о смерти персонажа по имени Комедиант, высказывает надежду, что тот смог достичь ИКСА. По одной из версий, происхождение слова "ИКС" связано с происхождением комиксов. Какое слово мы заменили ИКСОМ? Ответ: Панчлайн. Комментарий: Вполне вероятно, что само слово "панчлайн" произошло от названия журнала "Punch", в котором публиковались карикатуры, ставшие прародителями комиксов. Источник: 1. Watchmen, #4. 2. http://en.wikipedia.org/wiki/Punch_line 3. http://en.wikipedia.org/wiki/Punch_(magazine) Автор: Дмитрий Карякин (Москва) Вопрос 22: Рисунок на каристийском мраморе может представлять собой концентрические окружности. Другое название этой горной породы - имя с итальянскими корнями. Напишите это имя. Ответ: Чиполлино. Зачет: Циполин; чиполин; циполла; чиполла. Комментарий: Мрамор напоминает луковицу в разрезе, поэтому и был назван итальянским словом, обозначающим лук. Источник: http://ru.wikipedia.org/wiki/Циполин Автор: Николай Лёгенький (Минск) Вопрос 23: [Раздаются вырезанные из колоды карт символы пик.] Возьмите раздаточный материал. Команда "Эрудиты", решив играть чемпионат Украины по "Что? Где? Когда?", столкнулась с настороженным и даже слегка враждебным отношением к себе. Впервые попав в игровой зал, игроки команды обнаружили на своем игровом месте только ИКС. Что мы заменили ИКСОМ? Ответ: Два стула. Комментарий: Во многих герметичных коллективах новичков испытывают с помощью каверзного вопроса о двух стульях. Источник: ЛОАВ. Автор: Дмитрий Карякин (Москва) Вопрос 24: В фильме "Крик тишины" всего три персонажа, поэтому и сцены там в основном безлюдные. По сюжету героев в загородной поездке преследует огромное ОНО. Назовите ЕГО, используя дефис. Ответ: Перекати-поле. Комментарий: В голливудских вестернах в качестве метафоры безлюдного места используется перекати-поле, катящееся по городу. В фильме "Крик тишины" тоже было довольно безлюдно. Источник: http://ru.wikipedia.org/wiki/Крик_тишины Автор: Николай Лёгенький (Минск) Тур: 21 тур. "Неловкий трубкозуб" (Москва) Вопрос 1: Искусствовед Елена Дунаева пишет про пьесы, содержание которых рвется наружу, потому что ему становится тесно в условиях единства места. Такие пьесы Дунаева сравнивает с НИМ. В классическом источнике ОНО предлагалось в качестве альтернативы. Чему? Ответ: "Женитьбе Фигаро". Зачет: Чтению "Женитьбы Фигаро", перечитыванию "Женитьбы Фигаро". Комментарий: Такие пьесы искусствовед сравнивает с шампанским. У Пушкина шампанское как раз и предлагается как альтернатива классической пьесе: "Как мысли черные к тебе придут, Откупори шампанского бутылку Иль перечти "Женитьбу Фигаро"". Источник: 1. http://www.flibusta.net/b/198844/read 2. http://lib.ru/LITRA/PUSHKIN/p4.txt Автор: Сергей Спешков и Николай Крапиль (Москва) Вопрос 2: Героиня Малгожаты Мусерович вспоминает день, когда у себя дома она впервые увидела человека, с которым у нее впоследствии завязались отношения. В тот день - отмечает героиня - ОНА воспользовалась звонком. Назовите ЕЕ. Ответ: Судьба. Комментарий: Обычно судьба стучится в двери, в данном случае она предпочла дверной звонок. Источник: http://lib.ru/TALES/MUSEROWICH/celestina.txt Автор: Николай Крапиль (Москва) Вопрос 3: Злую эпиграмму, написанную суеверным Пушкиным на Андрея Муравьева, объясняли тем, что Муравьев был ИМ. На форуме "Женский клуб" существует раздел с анекдотами про НИХ. Назовите ИХ. Ответ: Блондины. Зачет: Белокурые, мужчины со светлыми волосами. Комментарий: Пушкину было предсказано, что он погибнет от руки блондина. На женском форуме самое место анекдотам про глупых мужчин. Источник: 1. http://www.hrono.info/biograf/bio_m/muravev_andr.html 2. http://pushkin.niv.ru/pushkin/vospominaniya/vospominaniya-69.htm 3. http://lady.webnice.ru/forum/viewtopic.php?t=31 Автор: Сергей Спешков (Москва) Вопрос 4: Японская поговорка утверждает: для влюбленных оспины - все равно что АЛЬФЫ. АЛЬФЫ упоминаются в описании нескольких героинь Куприна. Назовите АЛЬФЫ тремя словами. Ответ: Ямочки на щеках. Комментарий: Автор повести "Яма" писал не только про ямы, но и про ямочки. Источник: 1. http://www.livelib.ru/book/1000604293/quotes 2. http://readr.ru/aleksandr-kuprin-igrushka.html 3. http://modernlib.ru/books/aleksandr_ivanovich_kuprin/krovat/read_1/ 4. http://www.russkiymir.ru/russkiymir/ru/magazines/archive/2010/08/article0006.html Автор: Николай Крапиль (Москва) Вопрос 5: Стрелявший в Гарри Трумэна пуэрториканец Оскар Гольязо на суде рассказал, что перед покушением купил ЭТО. Клементина Лёвингер пишет, что никак не может свыкнуться с мыслью, что однажды ей придется купить ЭТО. У кого? Ответ: У Харона. Комментарий: Это - билет в один конец. Источник: 1. http://www.flibusta.net/b/312588/read 2. http://clem-lewinger.livejournal.com/57411.html Автор: Сергей Спешков (Москва) Вопрос 6: В начале одной из экранизаций ОН в гневе рвет карту пополам. Назовите ЕГО. Ответ: [Король] Лир. Комментарий: В начале пьесы Лир делит свои земли между тремя дочерями. Но после сдержанного ответа Корделии лишает ее наследства. В фильме Козинцева Лир при этом символически разрывает на две части карту королевства. Источник: Художественный фильм "Король Лир", реж. Григорий Козинцев, 14-я минута (http://www.youtube.com/watch?v=aVZirl-TPFs). Автор: Сергей Спешков (Москва) Вопрос 7: (pic: 20140060.jpg) В конце 90-х годов прошлого века подобные холодильники международной корпорации "Кока-Кола" получили в России прозвище, совпадающее с фамилией известного персонажа. Назовите этого персонажа. Ответ: [Альбус Персиваль Вульфрик Брайан] Дамблдор. Комментарий: Официально двухдверные холодильники называются Double-door. Свое неофициальное прозвище - Дамблдоры - такие холодильники получили в период быстро растущей популярности книг о "Гарри Поттере" из-за очевидного созвучия. Источник: Личный опыт автора вопроса. Автор: Николай Крапиль (Москва) Вопрос 8: Рассказ Макса Фрая "Вечерняя проповедь" написан по всем канонам русских народных сказок. В одном из слов предыдущего предложения мы пропустили букву. Напишите это слово в исходном виде. Ответ: Пропповедь. Комментарий: Рассказ написан в полном соответствии с учением самого известного советского филолога-фольклориста Владимира Яковлевича Проппа. Источник: http://www.loveread.ec/read_book.php?id=16560&p=22 Автор: Илья Бер (Москва) Вопрос 9: Персонажу исторического произведения говорят, что его сделают похожим на сыр и на карандаш. На какой именно карандаш? Ответ: Свинцовый. Комментарий: Персонажу угрожают огнестрельным оружием. И говорят, что он будет дырявым, как сыр и будет начинен свинцом, как карандаш. Источник: Телесериал "Blackadder the Third", эпизод "Amy and Amiability". Автор: Сергей Спешков (Москва) Вопрос 10: Автор текста одного онлайн-аудиогида по Эйфелевой башне говорит, что не понимает, кому и зачем может понадобиться имеющийся наверху башни ИКС. Какую аббревиатуру мы заменили на ИКС? Ответ: Wi-Fi. Комментарий: По всей видимости, текст писался не для онлайн-аудиогида, потому что как раз для его прослушивания вай-фай наверху Эйфелевой башни очень пригождается. Слово Wi-Fi произошло от английского словосочетания Wireless Fidelity, которое можно дословно перевести как "беспроводное качество". Источник: 1. http://vparige.ru/eyfeleva_bashnya_audio_gid_v_parizhe.html 2. http://ru.wikipedia.org/wiki/Wi-Fi Автор: Николай Крапиль (Москва) Вопрос 11: На картине "Музыка в Тюильри" среди множества персонажей Эдуар Мане нарисовал известных композиторов, поэтов, художников, в том числе и себя. Автор вопроса поставил в один ряд с "Музыкой в Тюильри" изображение, появившееся век спустя. Какая фамилия присутствует на этом изображении? Ответ: Пеппер. Зачет: Pepper. Комментарий: На обложке альбома "Битлз" "Sgt. Pepper's Lonely Hearts Club Band" есть и композиторы, и поэты, и художники, и сами участники четверки. Источник: 1. http://www.nationalgallery.org.uk/paintings/edouard-manet-music-in-the-tuileries-gardens 2. http://www.beatles.com/album/sgt-peppers-lonely-hearts-club-band Автор: Сергей Спешков (Москва) Вопрос 12: Большинство людей, не научившихся грамотно писать к концу школы, уже никогда не научатся это делать. Персонаж Стивена Кинга - преподаватель в школе для взрослых - отмечает, что большую часть АЛЬФЫ он расходует зря. Для приготовления АЛЬФЫ рекомендуется использовать свекольный сок. Назовите АЛЬФУ. Ответ: Красная паста. Комментарий: В первом случае имеется в виду паста в ручке, во втором - блюдо из макарон. Источник: 1. http://modernlib.ru/books/stiven_king/11_22_63/read_1/ 2. http://www.smak.ua/ru/recept/i-1732944/krasnaya_pasta_s_kalmarami.html Автор: Николай Крапиль (Москва) Вопрос 13: Суеверный мотогонщик Анхель Ньето - многократный чемпион мира. Напишите два числа, начинающиеся на одну и ту же цифру, которые входят в название документального фильма об его победах. Ответ: 12, 1. Комментарий: Он предпочитает говорить, что число его побед не 13, а 12 плюс 1. Источник: http://www.imdb.com/title/tt0758685/ Автор: Сергей Спешков (Москва) Вопрос 14: Заглавному герою одного произведения говорят, что Писемского звали Алексей Феофилактыч. Назовите этого героя. Ответ: Ионыч. Зачет: Дмитрий Ионович Старцев. Комментарий: Два довольно редких отчества. Источник: http://www.bibliotekar.ru/rusChehov/50.htm Автор: Сергей Спешков (Москва) Вопрос 15: В спектакле Камы Гинкаса Медея и Ясон вспоминают путешествие за золотым руном и спорят о некоторых деталях. Назовите короткую фамилию немецкого происхождения, носителя которой один из них цитирует в подтверждение своей правоты. Ответ: Кун. Комментарий: Николай Кун - автор известной книги "Легенды и мифы Древней Греции". Медея просто зачитывает Ясону фрагмент соответствующего мифа. Источник: МТЮЗ, "Медея", реж. К. Гинкас. Автор: Николай Крапиль (Москва) Вопрос 16: Героиня Иоанны Хмелевской попадает из XIX века в наши дни. В одном из эпизодов она успокаивается, поняв, что ИМ на самом деле ничего не грозит, хотя все равно удивляется, как вообще можно есть на улице. Назовите ИХ. Ответ: Собаки. Зачет: Горячие собаки. Комментарий: Узнав, что такое "хот доги", она понимает, что на самом деле люди не начали есть горячих собак, как можно было подумать. Источник: http://www.loveread.ec/read_book.php?id=8675&p=19 Автор: Николай Крапиль (Москва) Вопрос 17: В описании места, которое стало свидетелем легендарной трагедии, упоминаются два огромных камня, сужающиеся кверху. Напишите фамилию жертвы этой трагедии. Ответ: Баскервиль. Комментарий: "Посередине ее лежат два огромных камня, суживающиеся кверху и напоминающие гигантские гнилые клыки какого-то чудовища". Источник: А.Конан Дойл. Собака Баскервилей (http://lib.ru/AKONANDOJL/sh_baskr.txt). Автор: Сергей Спешков (Москва) Вопрос 18: Дуплет. 1. Персонаж детективного романа Тома Шарпа рассказывает про позеленевшие куски тела, которые полицейские доставали из ямок на НЕМ. Назовите ЕГО тремя словами. 2. В другом эпизоде того же романа полицейские предполагают, что женщина, которую они ищут, скрывается в НЕМ, хотя это предположение оскорбляет хозяина. Назовите ЕГО двумя словами. Ответ: 1. Поле для гольфа. 2. Дом викария. Зачет: 2. Дом священника и т.п. по смыслу. Комментарий: "Убийство на поле для гольфа" - один из самых известных рассказов Агаты Кристи об Эркюле Пуаро; "Убийство в доме викария" - первый рассказ Агаты Кристи, в котором действует мисс Марпл. Викарий гневно воспринимает само предположение, что он живет с женщиной. Источник: http://lib.ru/INPROZ/SHARP/wilt.txt Автор: Николай Крапиль (Москва) Вопрос 19: Герой Бена Элтона ест очень вкусный пончик и размышляет, есть ли в нем хоть что-то без НЕЕ. В одном детском стихотворении утверждается, что без НЕЕ вообще нельзя есть. Назовите ЕЕ. Ответ: [Буква] Е. Комментарий: Герой предполагает, что вкусовое удовольствие ему доставляют усилители вкуса и прочие добавки вида E-621 и ему подобных. Слово "есть" начинается с "е", поэтому есть без нее не получится. Источник: 1. http://www.e-reading.by/bookreader.php/150634/Elton_-_Do_poslednego_zvonka.html 2. http://www.readik.ru/str1_10_1_6.php Автор: Николай Крапиль (Москва) Вопрос 20: После того как персонаж рассказа Александра Королева СДЕЛАЛ ЭТО, он начал сомневаться в том, что земля круглая. Кто СДЕЛАЛ ЭТО в заглавии романа конца прошлого века? Ответ: Географ. Комментарий: СДЕЛАТЬ ЭТО - пропить глобус. Роман Алексея Иванова "Географ глобус пропил" написан в 1995 году. Источник: 1. http://samlib.ru/k/korolew_aleksandr_engelxsowich/maior.shtml 2. http://ru.wikipedia.org/wiki/Иванов,_Алексей_Викторович Автор: Николай Крапиль (Москва) Вопрос 21: В стихотворении Михаила Каневского для детей в море затонул танкер со снотворным. Напишите название этого стихотворения, состоящее из двух созвучных слов. Ответ: "Корабельная колыбельная". Источник: http://www.bintoshka.ru/korotkie-skazki-v-stixax-dlya-detej-na-noch/ Автор: Сергей Спешков (Москва) Вопрос 22: У Михаила Гаспарова отмечено, что как только ОН заговорил - расползся на социальные группы. Назовите ЕГО. Ответ: Народ. Комментарий: У Пушкина в "Борисе Годунове" народ безмолвствует. Источник: http://www.flibusta.net/b/244208/read Автор: Сергей Спешков (Москва) Вопрос 23: Александр Городницкий вспоминает, как в возрасте шестнадцати лет ему пришлось посетить несколько комнат, прикрываясь фиговым листом. Какое слово автор вопроса заменил в предыдущем предложении? Ответ: Обходным. Комментарий: "В этом доме в 1949 году помещался Райвоенкомат, где я вместе с другими допризывниками проходил приписку и необходимую при этом медкомиссию. В каждой из комнат бельэтажа сидели врачи, и мы, раздетые догола, ходили из комнаты в комнату, стыдливо прикрываясь обходным листом". Источник: Александр Городницкий. Атланты. Моя кругосветная жизнь (http://www.flibusta.net/b/323596/read). Автор: Сергей Спешков (Москва) Вопрос 24: Персонаж романа Брайт Поппи рассказывает, что в одиночной камере любое действие позволяло хоть как-то скоротать время, поэтому у него значительно уменьшился ИКС. ИКС упоминается в начале статьи об истории футбола. Назовите ИКС двумя словами. Ответ: Мочевой пузырь. Комментарий: Герою было настолько скучно, что он пользовался даже любой возможностью опорожнить мочевой пузырь, поэтому тот за время пребывания героя в тюрьме значительно сузился. Ранние прообразы футбольных мячей делали из свиных мочевых пузырей. Источник: 1. http://modernlib.ru/books/brayt_poppi/iziskanniy_trup/read/ 2. http://www.joma-sport.kz/page/istoriya-futbola Автор: Николай Крапиль (Москва) Тур: 22 тур. "Какая разница" Вопрос 1: Согласно современной городской легенде, раз в год ровно в полночь памятник Гагарину на одноименной московской площади поднимает руки вверх и произносит девятибуквенное слово. Какое? Ответ: Понаехали. Источник: http://www.liveinmsk.ru/places/a-656.html Вопрос 2: Согласно замечанию Талейрана, приведенному в книге "Истории простых вещей", ОНА - самая верная. Особенно не мудрите и назовите ЕЕ двумя словами. Ответ: Первая догадка. Зачет: Первая версия. Источник: http://lib.rus.ec/b/441675/read Вопрос 3: Актер Владлен Давыдов рассказывал, что с расколом МХАТа у его супруги Маргариты Анастасьевой начались проблемы со здоровьем, и тогда та решила совсем уйти из профессии. Давыдов, поддерживая супругу, пошутил, закончив реплику так: "... а дома у меня - ОНА". Какую фамилию носила самая, пожалуй, известная ОНА? Ответ: Жемчугова. Комментарий: "Мой дом - моя крепость". Вопрос 4: "В томатный сок добавить сок лимона (лучше свежевыжатый)...". Таково начало рецепта коктейля, который сайт povarenok.ru [поваренок точка ру] рекомендует как средство от похмелья. Какими двумя словами называется этот коктейль? Ответ: "Дева Мария". Зачет: "Virgin Mary". Комментарий: Коктейль-то безалкогольный. Источник: http://www.povarenok.ru/recipes/show/38389/ Вопрос 5: Увидев в программе одного известного отечественного телеканала три американских фильма подряд, разбавленных программой о покерном турнире, автор вопроса вспомнил о популярном в советские годы штампе. Назовите этот штамп из трех слов. Ответ: Тлетворное влияние Запада. Комментарий: Канал, о котором идет речь, - ТВ3. Но цифру можно принять за букву З [зэ]. Источник: http://tv.mail.ru/channel/1383/70/ Вопрос 6: [Ведущему: прочесть вопрос, отделяя каждый слог.] Вкладка "Множественный фильтр" "Ведомости по остаткам ТМЦ" модуля "Торговля и склад" программы 1С (версия 7.7) содержит шесть видов фильтра. Напишите абсолютно точно название того из фильтров, который идет в списке вторым. Ответ: По склАдам. Комментарий: А вопрос был прочтен "по складАм". Источник: В вопросе. Автор: Гай Кузнецов (Москва) Вопрос 7: Однажды автор вопроса в игровом чате загадал собравшимся слово, начинающееся на "з" и заканчивающееся на "ц". В качестве подсказки были предложены на выбор две анатомические характеристики, и сразу прозвучала версия, отличная от авторской, но подходящая ко всем заданным условиям. Мы не просим назвать ни авторское слово, ни версию ответа, напишите обе упомянутые характеристики. Ответ: Горб, уши. Комментарий: Загаданное слово - "Запорожец" (горбатый или ушастый), версия - заяц (существуют горбатые зайцы агути). Источник: ЛОАВ. Вопрос 8: Раневская рассказывала, что, когда заставала Ахматову в одиночестве на даче, та произносила реплику, вложенную в уста одного драматического персонажа. Напишите эту реплику из двух слов. Ответ: "Человека забыли". Вопрос 9: Фелиция, героиня ряда рассказов Жванецкого, употребляет по отношению к своему соседу Василию пятибуквенное ругательство. Напишите это ругательство. Ответ: Мразь. Зачет: Мурло. Комментарий: Нетрудно догадаться, что и Фелиция, и Василий - кошачьи имена. Вопрос 10: Если пассажиру "Красной стрелы" хочется чаю, он вешает на ручку дверцы своего купе соответствующую табличку. Начало надписи на оборотной стороне этой таблички напомнит о популярной телепередаче. Кто ее ведущий? Ответ: Дмитрий Дибров. Комментарий: А на обороте значится "Хочу стать миллионером. Принесите лотерейный билет". Источник: http://www.russian7.ru/2013/06/7-samyx-interesnyx-faktov-o-krasnoj-strele/ Вопрос 11: Когда пятилетняя Сара Бернар заболела туберкулезом, то попросила свою маму купить ЕГО. В спортивном ЧГК ОН - редкость. Назовите ЕГО двумя словами. Ответ: Красивый гроб. Комментарий: Свою экстравагантность великая актриса Сара Бернар проявляла еще в детстве. Гроб красного дерева, с атласной обивкой изнутри, сопровождал Сару всю жизнь. Источник: http://www.kalitva.ru/2007/03/04/sara_bernar_biografija.html Вопрос 12: ЕГО смерть оплакивали разве что обитатели Хитровки, лишившиеся заработка от перепродажи. Тот, чье имя родственно ЕГО фамилии, упоминается рядом со смертью в начале первой части хрестоматийного произведения. Какого? Ответ: "Мороз Красный Нос". Комментарий: Рисунки спивающегося, но еще не растерявшего свое мастерство Саврасова хитровцы перепродавали по 10-кратной цене. А "Смерть крестьянина" начинается со слов "Савраска увяз в половине сугроба". Вопрос 13: Комбинация, занявшая второе место в списке худших паролей 2013 года, напомнила в свое время журналисту Александру Гагину анекдот. Каким словом он заканчивается? Ответ: Проходи. Комментарий: Собственно, этим паролем является слово "password". Анекдот. "Скажи пароль" - "Пароль". Источник: http://www.rg.ru/2006/06/02/paroli.html Вопрос 14: Это понятие приводится в разделе "Заказ контракта" Кодекса преферанса. В определении к этому же понятию в бизнес-словаре упоминается новая ссуда. Назовите это понятие. Ответ: Перезаклад. Комментарий: Правда, Кодекс приводит это понятие, чтобы сказать о его несуществовании, а равно и отсутствии контракта "преферанс". Вопрос 15: Пора взбодриться. Что впервые получили из бычьей желчи Фридрих Тидеман и Леопольд Гмелин? Ответ: Таурин. Комментарий: Это одна из основных составляющих энергетиков. Вопрос 16: В заголовке материала о вручении вратарю "Спартака" Андрею Диканю премии "Джентльмен года" есть слова "СДЕЛАЛ ЭТО". Желая Андрею не ДЕЛАТЬ ТО, ответьте, какое существительное фигурирует в обеих исходных фразах. Ответ: Бабочка. Комментарий: СДЕЛАТЬ ЭТО - поймать бабочку (на смокинг), ДЕЛАТЬ ТО - пускать бабочку. Вопрос 17: Амбициозный и самолюбивый американский актер и продюсер Филип Майкл Томас придумал, преисполненный наполеоновских планов, аббревиатуру из четырех букв. Воспроизведите ее. Ответ: EGOT. Комментарий: Emmy, Grammy, Oscar, Tony. Источник: "Метро", 11.05.2011 г. - С. 22. Вопрос 18: В 1906 году Эрнест Сетон-Томпсон познакомился с одним лордом, вместе с которым стал активно пропагандировать идеологию жизни в гармонии с природой. Занятно, что инициалы фамилии лорда имеют расшифровкой и придуманный лордом девиз. Напишите и этот девиз, и фамилию лорда. Ответ: "Be prepared!" и Баден-Пауэлл. Вопрос 19: В Германии ОН, если разрешит курить, будет оштрафован на 1000 евро. Мы не просим назвать актера, получившего "Оскара" за исполнение роли ЕГО. Просто назовите ЕГО. Ответ: Таксист. Вопрос 20: Этот памятник представляет собой скамейку, на которой развалился довольный кот; еще бы, ведь он сбежал от сварливой старухи. А в каком городе установлен этот памятник? Ответ: В Йошкар-Оле. Вопрос 21: В своем цирковом детстве она носила ксилофон на голове, из-за чего, как ей казалось, и перестала расти. А в 1957 году эмигрировала в Польшу со своим мужем, оператором Леоном Жанно. Назовите ее самую известную роль. Ответ: Золушка. Вопрос 22: "Капуста тушится в котлах не меньше часа, С ней тушатся куски отборнейшего мяса, Покуда не проймет живые соки жаром, Покуда через край они не прыснут паром". А кто автор этих строк? Ответ: Адам Мицкевич. Комментарий: Приведен фрагмент приготовления бигоса, национального польского блюда. А кто "наше всё" польской поэзии? Источник: http://www.liveinternet.ru/users/astrahanka/post221818826/ Вопрос 23: (pic: 20140061.jpg) Перед вами, можно сказать, ИКС, можно сказать, ИКСА. Какое слово мы заменили на ИКС? Ответ: Скан. Комментарий: На раздатке отсканированный сканворд. Источник: 1. http://www.scanword.ru/news.asp - "скан" как "сканворд". 2. Значение "скан" как "копия, полученная путем сканирования исходного объекта", вызывать сомнений не должно. Вопрос 24: Вполне вероятно, вы не сможете назвать оперу, в которой присутствуют, среди прочих, Недда, Беппо и Сильвио. Но, скорее всего, вам знакомы последние два или четыре слова этой оперы. Процитируйте их. Ответ: "La commedia è finita". Зачет: "Комедия окончена". Незачет: "Финита ля комедия". Источник: 1. "Паяцы" Леонкавалло. 2. http://ru.wikipedia.org/wiki/Паяцы_(опера) Тур: 23 тур. "Всемирный потом" Редактор: Олеся Доронина Вопрос 1: (pic: 20140062.jpg) Недавно ФБР рассекретило документы почти семидесятилетней давности, имеющие отношение к этому человеку, включая несколько отредактированных версий его фотографии, подготовленных на случай, если он изменит внешность. Назовите этого человека. Ответ: [Адольф] Гитлер. Источник: http://www.ibtimes.com/hitler-files-fbi-hunts-nazi-killer-295179 Автор: Андрей Заболотский Вопрос 2: (pic: 20140063.jpg) Эта дверь изображала на телеэкране вход в жилище ПЕРВОГО. Неизменный атрибут ПЕРВОГО был не менее дорог и ВТОРОМУ, который из-за этого даже лишился жизни. Смерть ВТОРОГО роднит его с ТРЕТЬИМ, самым известным из этой достойной троицы. Назовите ТРЕТЬЕГО. Ответ: Иисус Христос. Комментарий: Этот дом изображал дом Бейкер-стрит, 221б в знаменитом советском телесериале о Шерлоке Холмсе; ПЕРВЫЙ - Холмс, он курил трубку; трубку (люльку) также курил ВТОРОЙ - Тарас Бульба, его схватили враги, когда он остановился ее поднять; Тарасу Бульбу на казни прибили руки, как и Христу. На последнего также намекает и слово "троица". Источник: 1. http://ru-sherlockiana.livejournal.com/383608.html 2. Н.В. Гоголь, Тарас Бульба. Автор: Андрей Заболотский Вопрос 3: Внимание, в вопросе есть замены. Его лето упоминается в песнях групп "Люмен", "Алиса", "Ляпис Трубецкой", "Немного нервно", "Сектор Газа" и других. Какие две буквы мы пропустили в тексте этого вопроса? Ответ: р, в. Комментарий: Первые два слова - Егор Летов. Сам приведенный факт намекает на русский рок. Источник: http://ru.wikipedia.org/wiki/Летов,_Егор Автор: Андрей Заболотский Вопрос 4: <раздатка> 1 937 j00Z 15 5|-|07 Ph34R 70937|-|3R В распространенном интернет-диалекте первая фраза из написанных означает "понял тебя", вторая - "застрелен", а третья - "бояться вместе". Переведите фразу id10t [ай-ди-десять-ти]. Ответ: Идиот. Зачет: Idiot. Комментарий: Это язык Leetspeak, в котором буквы латиницы заменяются похожими на них цифрами или сочетаниями символов, а также используются некоторые специфические суффиксы. Примеры переводятся как "I get you", "is shot" и "fear together". Источник: 1. http://en.wikipedia.org/wiki/Leet 2. http://lurkmore.to/Leet Автор: Александр Яковлев Вопрос 5: (pic: 20140064.jpg) Изображенный на фотографии аллигатор ДЕЛАЕТ ЭТО. В советском мультфильме 1947 года герой ДЕЛАЕТ ЭТО, танцуя. Ответьте двумя словами, начинающимися на одну и ту же букву: что делает? Ответ: Приманивает птицу. Зачет: Приманивает птиц. Комментарий: Крокодилы приманивают птиц, накладывая себе на морду ветки, которые служат пернатым материалом для постройки гнезд. В мультфильме 1947 года "Конек-горбунок", чтобы приманить захмелевшую Жар-птицу, Иван-дурак танцует перед ней. Источник: 1. http://lenta.ru/news/2013/12/13/crocs/ 2. http://video.yandex.ru/users/erihgeesuo/view/953/ Автор: Юрий Угольников Вопрос 6: Вероятно, с момента произнесения слов "Come here" и до сегодняшнего дня сотрудники правоохранительных органов регулярно ДЕЛАЮТ ЭТО. В 20-х годах прошлого века в Америке было позволено СДЕЛАТЬ ЭТО лингвисту в сугубо научных целях. Что мы заменили оборотом "ДЕЛАТЬ ЭТО"? Ответ: Прослушивать телефонные разговоры. Зачет: По смыслу. Незачет: Записывать телефонные разговоры. Комментарий: 'Mr. Watson - come here - I want to see you' или 'Mr. Watson - come here - I want you' - первая фраза, сказанная по телефону его вроде бы изобретателем Александром Беллом. Источник: 1. http://avva.livejournal.com/2681390.html 2. http://www.wired.com/science/discoveries/news/2008/03/dayintech_0310 Автор: Андрей Заболотский Вопрос 7: Блиц. 1. Последние пять человек, имевшие в сборной России по футболу этот статус, - это Егор Титов, Андрей Аршавин, Сергей Семак, Игорь Денисов и Роман Широков. Назовите этот статус одним словом. 2. "Весной 1897 года [ПРОПУСК] Лондон поддался "золотой лихорадке"". Какое слово в этой цитате из Википедии мы пропустили? 3. Если верить детской песне, в случае ЕГО приближения следует приготовить оружие. Назовите ЕГО одним словом. Ответ: 1. Капитан. 2. Джек. 3. Воробей. Источник: 1. http://ru.wikipedia.org/w/index.php?title=Сборная_России_по_футболу&oldid=60010159 2. http://ru.wikipedia.org/wiki/Джек_Лондон 3. http://www.pesni.net/text/Pesni-iz-multfilmov/Bremenskie-muzykanty-Oh-rano-vstaet-ohrana Автор: Андрей Заболотский Вопрос 8: На карикатуре художника Дана Пирраро, нарисованной в позапрошлом году, изображен необычно одетый человек, печально сидящий перед барной стойкой. Бармен протягивает ему стакан со словами "Взбодрись, приятель, это же не конец света". Назовите национальность печального персонажа. Ответ: [Индеец] Майя. Комментарий: В 2012 году, как мы помним, мир ждал конца света, якобы предсказанного Майя. Источник: http://bizarrocomics.com/2012/12/20/my-last-post-ever/ Автор: Андрей Заболотский Вопрос 9: Майя Котовская, более известная как Канцлер Ги, в своей песне "Terra Sancta I" от лица лирического героя сулит крестоносцам, что они добудут в Святой Земле не один ИКС, а сразу много ИКСОВ для личного пользования. Многие здесь присутствующих не раз имели дело с ИКСАМИ. Какое слово мы заменили словом "ИКС"? Ответ: Гроб. Комментарий: "И мы отвоюем, похоже, не гроб [Господень], а целый штабель гробов [для себя]". Источник: http://lyrics.wikia.com/Канцлер_Ги_(Kantsler_Gi):Terra_Sancta_I Автор: Андрей Заболотский Вопрос 10: На первом постере изображен человек и упоминаются бром и барий. На втором постере, шуточном, посвященном мероприятию, случившемуся минувшей осенью, также изображен человек и аналогичным образом упоминаются алюминий и натрий. Что это было за мероприятие? Ответ: Выборы мэра Москвы. Комментарий: Первый (официальный) постер - к сериалу "[Br]eaking [Ba]d", второй (народное творчество) - отредактированный предвыборный плакат [Ал]ексея [На]вального. Источник: 1. http://en.wikipedia.org/wiki/Breaking_Bad_(season_5) 2. http://navalny.livejournal.com/815625.html?thread=463706121#t463706121 Автор: Андрей Заболотский Вопрос 11: На форуме, посвященном различным языкам и переводам, иностранец попросил объяснить значение прилагательного в цитате Владимира Жириновского, указав, что словари дают явно неподходящее толкование: Владимир Вольфович явно говорил не о ручках. В ответ ему кратко пересказали сюжет классического произведения русской литературы XX века, название которого состоит из двух слов, начинающихся с одной и той же буквы. Напишите название этого произведения. Ответ: "Собачье сердце". Комментарий: "... дети "Шариковых", дети рабочих и крестьян...". Источник: 1. http://forum.wordreference.com/showthread.php?t=2427448 2. http://www.vesti7.ru/news?id=11944 Автор: Андрей Заболотский Вопрос 12: Дидье Дельсаль побывал на вершине Эвереста, высочайшей горы мира, не использовав никакого альпинистского снаряжения. Ответьте одним словом, какую вещь, для создания которой среди прочих материалов использовался бамбук, представила фирма "Луи Виттон" на крупном профильном мероприятии в Париже в 1909 году. Ответ: Вертолет. Комментарий: Из бамбука была сделана рама; мероприятие - Парижский авиасалон; мсье Дельсаль добрался до вершины Эвереста на вертолете (что тоже было нетривиально). Источник: 1. http://nostradamvs.livejournal.com/431644.html 2. http://nostradamvs.livejournal.com/465487.html Автор: Андрей Заболотский Вопрос 13: Наука считает чрезвычайно важным, что ни одного из грызунов под названием "голые землекопы" ни разу не убил ИКС. Но это неудивительно: ведь ИКСЫ обитают вовсе не под землей. Назовите ИКС. Ответ: Рак. Источник: http://lenta.ru/articles/2012/11/16/blidemolerat/ Автор: Андрей Заболотский Вопрос 14: И снова о голом землекопе. Еще одна характерная особенность этого грызуна состоит в том, что у взрослого животного ИХ около ста. Назовите ИХ. Ответ: Волосы. Зачет: Волоски. Источник: http://www.elementy.ru/news/430671 Автор: Андрей Заболотский Вопрос 15: В восьмидесятых годах прошлого века выходил малоизвестный мультсериал про необычного супергероя, его до сих пор знают взрослые и дети всего мира. Особенность его заключалась в том, что для использования силы ему нужно было собраться, что порой требовало немалой концентрации и времени. Однако Бог мог бы помочь ему в этом за двадцать шагов. Назовите этого героя двумя словами. Ответ: Кубик Рубика. Комментарий: Кубик Рубика, оживающий, будучи собранным, - главный герой мультсериала "Рубик - удивительный кубик". Доказано, что можно собрать любой кубик Рубика не более чем за двадцать шагов, соответствующий алгоритм сборки называется алгоритмом Бога. Источник: 1. http://ru.wikipedia.org/wiki/Рубик_%E2%80%94_удивительный_кубик 2. http://www.cube20.org/ Автор: Дарья Смирнова, Андрей Заболотский Вопрос 16: Внимание, в вопросе есть пропуск. Рассказывают, что для запоминания тройки использовалась фраза "пошел ты нааа [ПРООПУСК]". Мы просим вас не заполнять пропуск, но изобразить соответствующий ему знак. Ответ: Тире. Комментарий: Речь идет о так называемом мнемоническом коде для запоминания символа азбуки Морзе, в данном случае цифры 3 - три точки, два тире. Источник: 1. http://d-i-a-n-a-n-a-s.livejournal.com/56568.html?thread=1016568#t1016568 2. http://ru.wikipedia.org/wiki/Азбука_Морзе Автор: Андрей Заболотский Вопрос 17: Значение одной важной характеристики недавно открытого материала графена, измеренное экспериментально, отличается от значения, предсказанного большинством теорий, в определенное число раз. Эта проблема известна в англоязычной литературе, посвященной графену, как "загадка пропавшего пирога". Во сколько раз отличаются экспериментальное и теоретическое значение? Ответ: Пи. Зачет: 3,14... Комментарий: Пирог - pie - звучит как pi - пи. Источник: http://www.arxiv.org/abs/0906.2856 Автор: Андрей Заболотский Вопрос 18: Компания "Crane" более ста лет изготовляла свою продукцию из текстильных отходов, в частности, из обрезков джинсовой ткани. Продукция компании, в свою очередь, является основой для производства ИХ. ОНИ широко используются по всему миру, особенно в США. Однако в последние годы в джинсовую ткань стали добавлять эластичный материал спандекс, что сделало ее непригодной для дальнейшего использования компанией "Crane". Поэтому компания перешла на использование натурального, непереработанного хлопка, что увеличило ее собственную потребность в НИХ. Назовите ИХ точно. Ответ: Доллары США. Комментарий: "Crane" делает бумагу, на которой потом печатаются сами доллары. Источник: http://www.inosmi.ru/world/20131222/215931759.html Автор: Иван Соколов, Андрей Заболотский Вопрос 19: В мультфильме "ВАЛЛ-И" главный герой - робот. В мультфильме использовались четыре различные модели ВАЛЛ-И: исправный, сломанный, "свернутый в куб" и [пропуск]. Что мы заменили словом "пропуск"? Ответ: Мертвый. Зачет: Умерший, погибший. Источник: http://ru.wikipedia.org/wiki/ВАЛЛ-И Автор: Дарья Смирнова Вопрос 20: ЕЙ указывал путь сначала ПЕРВЫЙ, потом ВТОРОЙ. Китайцы используют ПЕРВОГО, а вьетнамцы - ВТОРОГО для одного и того же. Однако китайцы и вьетнамцы солидарны в вопросе о том, кто и кому только что передал вахту. Назовите ЕЕ. Ответ: Алиса. Зачет: Алиса в Стране чудес. Комментарий: Первый и второй - (белый) кролик и (Чеширский) кот, соответственно; они соответствуют одним и тем же годам по "восточному календарю". И по китайской, и по вьенамской версии календаря 31 января 2014 года год змеи сменяется годом лошади. Источник: 1. Л. Кэрролл. Алиса в Стране чудес. 2. http://ru.wikipedia.org/wiki/Китайский_зодиак Автор: Александра Пшеничная Вопрос 21: Менеджер по маркетингу фирмы "Signetics" Арт Фьюри посчитал, что это число в названии микросхемы будет особо запоминающимся и успешным на рынке. Неизвестно, знал ли он, что это число является палиндромом не только в десятичной, но и в девяти- и двенадцатеричной системах счисления, но продукт действительно оказался успешным, став одной из самых массовых микросхем в мире. Однако если приписать к этому числу еще несколько цифр, то мы получим, скорее всего, обозначение сугубо вымышленной сущности. Назовите это число. Ответ: 555. Комментарий: Число 555 получило широкую известность в качестве телефонного префикса североамериканского плана нумерации, повсеместно используемого в США при составлении вымышленных телефонных номеров для различных художественных произведений (в первую очередь, кинофильмов). Источник: http://ru.wikipedia.org/wiki/555_(число) Автор: Алексей Панов Вопрос 22: Сорок лет назад доктор Роберт Райнс и сэр Питер Скотт предположительно наблюдали это существо и предложили латинское название для вида, к которому оно принадлежало. Название состояло из двух слов, второе из которых - rhombopteryx, что значит "с ромбовидными плавниками". Однако было замечено, что предлагаемое название является анаграммой словосочетания Monster hoax by Sir Peter S - "Чудовищная фальшивка сэра Питера С.". Что это за существо? Ответ: Несси. Зачет: Лох-несское чудовище. Источник: http://ru.wikipedia.org/wiki/Лох-несское_чудовище Автор: Андрей Заболотский Вопрос 23: В сценке "Christmas Confusion" скрипач Алексей ИгУдесман и пианист Ричард Хьюнг-ки Джу дарят скрипачу Джошуа Беллу на Рождество ноты небольшой пьесы. Белл замечает, что он и Игудесман - ИКСЫ, и просит Алексея сделать ноты более ИКСОВЫМИ. Алексей делает с нотами нечто. Джу заявляет, что он единственный из них троих настоящий ИКС. После этого Игудесман исправляет ноты, и все трое исполняют произведение. Что первым делом сделал с нотами Игудесман после просьбы Белла? Ответ: Обрезал. Зачет: По слову "обрезал", либо "отрезал угол". Комментарий: Белл и Игудесман - евреи; Джу - Joo - звучит как Jew - еврей. Источник: http://www.youtube.com/watch?v=RyW-Jz3FECs Автор: Андрей Заболотский Вопрос 24: В 2008 году американский ученый Скотт Ааронсон в ответ на возражения против ЭТОГО ПРОЕКТА написал: "Современный уровень знаний не позволяют исключить возможность того, что в следующем году прилетят инопланетяне, обнаружат, что мы до сих пор не СДЕЛАЛИ ЭТО, сочтут нас смехотворно примитивными и поработят". Назовите точно этот проект или то, что мы все-таки сделали. Ответ: Большой Адронный Коллайдер. Зачет: Открытие бозона Хиггса. Источник: 1. http://www.scottaaronson.com/blog/?p=334 2. http://ru.wikipedia.org/wiki/Бозон_Хиггса Автор: Андрей Заболотский Тур: 24 тур. "Митридат и Тобаско" Редактор: Олег Холодов Вопрос 1: (pic: 20140065.jpg) Авторы вопроса видят ЕГО так. Назовите ЕГО одним словом. Ответ: Альфа-самец. Комментарий: Альфа - первая буква греческого алфавита. В системе греческой алфавитной записи чисел имеет числовое значение 1. Происходит от финикийской буквы алеф, которая в переводе означает "бык" и своим начертанием изображает голову быка. Автор: Александр Надеев, Олег Холодов Вопрос 2: Певец Марк Алмонд в результате пережитой аварии полностью оглох на левое ухо. Поэтому, по мнению автора вопроса, его парные татуировки не соответствуют изначально вкладываемому смыслу, и поступки Алмонда должны быть исключительно положительными. Ответьте точно, что это за татуировки и где они расположены. Ответ: Ангел на правом плече, дьявол - на левом. Комментарий: К увещеваниям дьявола певец глух в прямом смысле. Источник: 1. http://fuzz-magazine.ru/magazine/2008/10-2008/4053-2010-06-08-09-21-45 2. http://ru.wikipedia.org/wiki/Алмонд,_Марк Автор: Никита Поздняков Вопрос 3: Внимание, в вопросе есть замена. По мнению героев Лукьяненко, человеческое сознание, чтобы не скатываться в крайность и не разбрасываться, предпочитает иметь дело с АЛЬФОЙ. В шутку к гоминидным АЛЬФАМ причисляют и известное определение: "Двуногое без перьев с плоскими ногтями". Какое слово мы заменили словом "АЛЬФА"? Ответ: Триада. Комментарий: Гоминидная триада - совокупность трех признаков, присущих только человеку. "- Не знаю, - дед пожал плечами. - Два - это крайности. Четыре и более - слишком много вариантов для нашего сознания. Три - в самый раз. Ты имеешь в виду, что люди способны упускать информацию, не попадающую в "триаду"? - Да. - Не думаю. Мы просто сводим многообразие ситуаций к трем основным группам. Это достаточно эффективно и удобно для нас". Источник: 1. Сергей Лукьяненко. Звезды - холодные игрушки. 2. http://ru.wikipedia.org/wiki/Гоминидные_триады Автор: Татьяна Верясова, Олег Холодов Вопрос 4: [Раздаточный материал сложен вчетверо.] <раздатка> для социалистических стран марксизм-ленинизм стал главным учением рабочего класса. Впитав основные идеи марксистской школы, марксизм- ленинизм значительно упростил ее, сделав философией доступной массам Ответьте как можно точнее, на чем автор вопроса увидел данный текст. Ответ: Школьная тетрадь. Источник: (pic: 20140066.jpg) Автор: Александр Надеев Вопрос 5: В своем твиттере не лишенный самоиронии активный пользователь Захар Май написал двустишие: "Пороков изведал я множество, но главный из них - [ПРОПУСК]". Заполните пропуск неологизмом из 14 букв. Ответ: Микробложество. Источник: https://twitter.com/ZaxarBorisych/status/420817371754799107/ Автор: Никита Поздняков Вопрос 6: Внимание, в вопросе есть замены. ИКСОВАЯ АЛЬФА имеет длину несколько сантиметров. АЛЬФА ИКСА имела длину 12,5 метров. Некоторые источники утверждают, что на лбу ИКСА можно увидеть китайский иероглиф "Король". Что мы заменили на АЛЬФУ ИКСА? Ответ: Гусеница тигра. Комментарий: Тигровая гусеница - коричнево-черная. Источник: 1. http://ru.wikipedia.org/wiki/Pyrrharctia_isabella 2. http://ru.wikipedia.org/wiki/Тигр_(танк) 3. http://www.factroom.ru/facts/24304 Автор: Александр Надеев, Олег Холодов Вопрос 7: Первый в СССР был открыт в Ленинграде. ЕГО посещение с непосредственным участием в "действе" даже входило в программу пребывания финских туристов в северной столице в 60-е годы прошлого века. "Витрина" одного из НИХ в настоящее время предлагает большой выбор всевозможных предметов и услуг, от теплохода до аксессуаров. Назовите ЕГО. Ответ: Дворец бракосочетания. Комментарий: Витрина на сайте дворца. Источник: 1. Л. Парфёнов. Намедни. Наша эра. 1961-1970. - М.: Колибри, 2011. 2. http://www.stena.ee/blog/lyubopytnye-fakty-o-brakosochetaniyah-v-sssr 3. http://www.porapoparam.ru/showcase Автор: Варвара Гарнюк, Олег Холодов Вопрос 8: (pic: 20140067.jpg) И на правой, и на левой картинке изображена АЛЬФА ИКСА. Назовите АЛЬФУ ИКСА. Ответ: Дельта Нила. Комментарий: На правой картинке - дельтовидная мышца Нила Арсмтронга. Автор: Александр Надеев Вопрос 9: Цитата из интервью экс-кандидата в президенты США Митта Ромни: "Что касается, например, Олимпиады в Мюнхене, на которой присутствовал ОН, - там, конечно, символическое значение Игр было этим самым присутствием перечеркнуто". Мы не просим назвать ЕГО. Скажите, в чем ошибся Ромни. Ответ: Перепутал игры в Берлине и Мюнхене. Комментарий: ОН - Гитлер. Источник: http://inotv.rt.com/2014-01-06/Mitt-Romni-Rossii-udastsya-obezopasit Автор: Варвара Гарнюк Вопрос 10: Фазиль Искандер назвал ЕЕ "южной сосулькой". Прозвище "южная сосулька" получило и изделие, представленное американской компанией на спецвыставке в Тбилиси. Хотя на взгляд авторов вопроса "батон, он и в Африке батон". Что Фазиль Искандер назвал "южной сосулькой"? Ответ: Чурчхелу. Комментарий: Дубинка полицейского по-английски - "baton". Источник: 1. http://ru.wikipedia.org/wiki/Полицейская_дубинка 2. http://izumizum.spb.ru/opus/?article/21ru Автор: Александр Надеев, Олег Холодов Вопрос 11: Термин "ТАКОЕ ОНО" возник из-за оговорки в эфире и созвучности фамилии. Название этому термину дал ТАКОЙ ОН, находящийся в ТАКОМ районе Москвы. Тимур Шаов упоминает его в связи с незыблемостью соблюдения законов физики. Восстановите прилагательное, которое мы заменили на слово "ТАКОЙ". Ответ: Басманный. Комментарий: "Басманное правосудие" - устойчивое выражение, которые различные авторы используют для характеристики судебной системы, <...> отличающейся, по их мнению, низкой степенью независимости судебных органов в принятии решений; постановлениями, удобными для властей, но идущими вразрез с законностью. <...> Получило название от наименования Басманного районного суда города Москвы, известного проведением громких и спорных судебных процессов, вызвавших разностороннюю критику в адрес российской судебной системы. Источник: 1. http://ru.wikipedia.org/wiki/Басманное_правосудие 2. Тимур Шаов, "Элементарная частица". Автор: Никита Поздняков Вопрос 12: Сына Николая Валуева, посещающего школу, похожую на многие американские, и статус в британском кабинете министров Лорда - хранителя малой печати объединяет название рассказа Аркадия Аверченко. Воспроизведите это название. Ответ: "Без портфеля". Комментарий: В школе, где учится сын Валуева, стоят шкафчики, поэтому всё, что носит с собой ученик - это домашние задания. Положение Лорда-хранителя Малой печати соответствует должности министра без портфеля. Источник: 1. http://www.mk.ru/social/highlife/article/2013/08/29/907295-syin-valueva-hodit-v-shkolu-bez-portfelya.html 2. http://ru.wikipedia.org/wiki/Лорд-хранитель_Малой_печати 3. http://www.rulit.net/books/ministr-bez-portfelya-read-189829-1.html Автор: Никита Поздняков Вопрос 13: На эмблемах Олимпийских игр ОН обычно обозначается двумя или четырьмя знаками. Но один раз ОН был обозначен пятью знаками. А какой город тогда принимал Олимпиаду? Ответ: Рим. Комментарий: 1960 год был записан римскими цифрами - MCMLX. Автор: ??? Вопрос 14: Немцы считают, что ЖЕМЧУЖИНА является "меркой любви". Англичане ласково называют ЖЕМЧУЖИНУ "око дня". Поэт Монтгомери писал: "... ЖЕМЧУЖИНА никогда не умирает". Какое слово мы заменили на "ЖЕМЧУЖИНА"? Ответ: Маргаритка. Комментарий: Маргарита - восходит к др.-греч. μαργαρίτης [маргаритис] - жемчужина, жемчуг. Источник: http://www.florets.ru/legendy-o-tsvetah/margaritki.html Автор: ??? Вопрос 15: В честь АФРОДИТЫ были названы мост через Тибр и высший из пиков горы Стэнли. Но нам АФРОДИТА в первую очередь известна благодаря своему экстравагантному желанию отведать нечто поострее. Кого мы заменили на АФРОДИТУ? Ответ: Маргариту [Савойскую]. Комментарий: В древнегреческой мифологии "Маргаритос" - один из эпитетов богини Афродиты. Пицца (итал. pizza) - от итал. pizzicare - "быть острым". Источник: 1. http://ru.wikipedia.org/wiki/Маргарита_Савойская_(королева) 2. http://www.pizza-margherita.ru/history.html Автор: Татьяна Верясова, Олег Холодов Вопрос 16: Внимание, в вопросе есть замена. В преддверии Сочинской олимпиады в статье "Германия на олимпиаде: Ныряющее падение локтя" читаем: "Германия может смело рассчитывать на медали в этом виде спорта". В серии компьютерных игр "Assassin's Creed" ныряющее падение локтя - прием, часто используемый в ордене Ассассинов. О каком виде спорта идет речь? Ответ: Прыжки на лыжах с трамплина. Комментарий: Рестлер Шейн Макмэн проводит версию "Ныряющего падения локтя", в которой соперник лежит на комментаторском столе, а он находится на верхней стойке и прыгает на противника своим локтем. Он называет этот прием Прыжок Веры. Источник: 1. http://www.germania-online.ru/gesellschaft/obshchestvo-detal/datum/2014/01/23/germanija-na-olimpiade-pryzhok-very.html 2. http://ru.wikipedia.org/wiki/Воздушные_атаки_(реслинг) Автор: ??? Вопрос 17: Если верить мифам коми, создатель морозов, бог Омоль, подобно известному персонажу, когда-то запятнал себя поистине темным делом - кражей. Добрый бог Ен - кстати, отец небесного Медведя - заставил вора вернуть краденое. Но доказательства вины Омоля можно увидеть до сих пор. Что именно считают такими доказательствами мифы? Ответ: Пятна на солнце. Комментарий: Эти холодные области считаются следами пальцев Омоля. Этот злой бог когда-то украл с неба солнце, но Ен заставил его вернуть похищенное. Так же, как Медведь у Чуковского заставил вернуть краденое солнце Крокодила. Источник: В. Петрухин. Мифы финно-угров. - М.: АСТ, Астрель, 2005. - С. 198, 206, 218 (http://lib.rus.ec/b/370923/read). Автор: ??? Вопрос 18: Познакомившись в 1812 году с башкирскими всадниками, французы стали называть их Жнецами, а как их называли более романтично настроенные француженки? Ответ: Амурами. Источник: Е. Тарле. Наполеон Бонапарт. Автор: ??? Вопрос 19: Внимание, в вопросе есть замена. Гай Светоний пишет: "Но, проиграв в одном, Цезарь выиграл в другом, что ценилось в Риме гораздо более высоко: он показал себя безупречным, образцовым ИКСОМ". Статья в женском журнала "Krasa", посвященная искусству завоевания мужчины, называется "До последнего ИКСА". Какое слово мы заменили на ИКС? Ответ: Патрон. Комментарий: Патрон - покровитель, заступник, меценат. Источник: 1. http://www.peoples.ru/state/king/rome/caesar/history.html 2. http://www.krasa.uz/zdorovye/do-poslednego-patrona/ Автор: ??? Вопрос 20: Вот уже сорок лет во всех документах ФРГ это слово - табу. Сегодня за него можно получить пощечину или даже угодить в полицию. Этим словом в разговорном немецком могут называть проституток. Феминистки считают искоренение этого слова своей самой большой победой в Германии. Напишите это слово. Ответ: Фройляйн. Зачет: Fräulein. Комментарий: Тут у феминисток аргументов хватает с лихвой. Незамужней может быть и женщина в летах, и за то, что она одинока, негоже обращаться к ней, как к девочке. К тому же суффикс -lein придает уменьшительно-ласкательный оттенок. Так что получается, что "фройляйн" - это не полноценная женщина, а так, молодо-зелено, при этом к холостым мужчинам никаких уменьшительных частиц не приставляют. Источник: http://www.germania-online.ru/wissenschaft-bildung/nauka-i-obrazovanie-detal/datum/2013/08/28/jazykovoi-seksizm-nemcev-bespokojat-polovye-voprosy.html Автор: ??? Вопрос 21: Туземцы малайского архипелага используют для этого кокосовый орех с проколотой в нем маленькой дырочкой. А что для этого используем все мы? Ответ: Часы. Комментарий: Туземцы кладут орех в какой-нибудь сосуд с водой. Когда вода наполнит орех, он опустится на дно. Своеобразный аналог песочных часов. Источник: "Вокруг света", 1974, N 10. Автор: ??? Вопрос 22: Прослушайте две строки из стихотворения А. Блока "Сытые": "Шипят пергаментные речи, С трудом шевелятся мозги". По мнению литературоведа А. Туркова, эти две строчки могли бы стать подписью к одной картине И. Репина. Какой? Ответ: "Заседание Государственного Совета". Источник: А. Турков. Александр Блок (Серия "ЖЗЛ"). - М.: Молодая гвардия, 1969. - С. 84. Автор: ??? Вопрос 23: Святослав Логинов утверждает, что слово "ОН" первоначально означало "наездник на драконе". На сайте любителей игры "Казаки" лучшим средством от НИХ считается русский витязь. Какое слово мы заменили на "ОН"? Ответ: Драгун. Источник: 1. http://www.rusf.ru/loginov/books/story03.htm 2. http://clannwo.ucoz.ru/publ/interesnye_fakty_iz_igry/1-1-0-8 Автор: ??? Вопрос 24: Один из пользователей Живого Журнала задается вопросом: "Имеет ли ОН отношение к такси? Или к Хорватии?". А другой отвечает, что, скорее всего, ЕГО первым использовал шахматист. Назовите ЕГО двумя словами. Ответ: Финишный флаг. Зачет: Клетчатый флаг. Источник: http://useless-faq.livejournal.com/7347502.html Автор: ??? Тур: 25 тур. Тур оргкомитета N 1 Редактор: Александр Губин (Челябинск) Вопрос 1: В ЕГО рассказе "Убийство" герой, входя в комнату к брату в начале рассказа, видит прислоненный к стене утюг. Назовите этого писателя. Ответ: Чехов. Комментарий: А через некоторое время этим утюгом он и убивает несчастного брата. Такое вот своеобразное "ружье на стене". Источник: 1. А.П. Чехов. Убийство. - М.: Эксмо, 2011. - С. 153. 2. http://ru.wikipedia.org/wiki/Ружьё_Чехова Автор: Александр Губин (Челябинск) Вопрос 2: В горе, у подножия которой расположен китайский городок Фумин, два десятилетия добывали гранит, а теперь разработки прекращены. Недавно горсовет решил закрасить "рану" в горе зеленой краской. Напишите несклоняемое слово, которое встречается в статье об этом. Ответ: Фэншуй. Комментарий: Чтобы восстановить благоприятные потоки энергии ци. Источник: 1. http://www.nkj.ru/archive/articles/11836/ 2. http://ru.wikipedia.org/wiki/Фэншуй Автор: Александр Губин (Челябинск) Вопрос 3: Внимание, в вопросе есть замена. В столице Таиланда Бангкоке ежемесячно случается около двадцати дорожно-транспортных происшествий с участием ШУТОВ. В Банкоке официально запрещается угощать ШУТОВ, штраф составляет 10 тысяч бат, или 240 евро. Какое слово мы заменили на "ШУТ"? Ответ: Слон. Комментарий: Шахматная фигура слон у французов называется шутом. Источник: 1. http://www.live-cs.ru/topics/151737/ 2. http://ru.wikipedia.org/wiki/Слон_(шахматы) Автор: Александр Губин (Челябинск), Олег Холодов (Серпухов) Вопрос 4: "Аллюзии придумали не символисты, их придумала жизнь", - писал Олег Стожар. А одного из героев произведения Тибора Фишера "Коллекционная вещь" постоянно видели в компании его любимых кошки, собаки и свиньи. Назовите клички упомянутых кошки, собаки и свиньи. Ответ: Кошка, Собака и Свинья. Источник: Т. Фишер. Коллекционная вещь. - М.: Иностранка, БСГ-Пресс, 2003. - С. 203. Автор: ??? Вопрос 5: Согласно одной недостоверной версии, руководитель компании Рей Крок изначально использовал для привлечения Деревянного идола, но тот быстро изнашивался. Будучи большим поклонником "Мастера и Маргариты", Рей решил поставить своего сотрудника вместо идола. Напишите имя этого сотрудника. Ответ: Рональд. Комментарий: Якобы искал имя похожее на Воланд. Источник: 1. http://wap.forumkavkaza.forum24.ru/?1-1-20-00000114-000-0-0 2. http://ru.wikipedia.org/wiki/McDonald%E2%80%99s Автор: Александр Губин (Челябинск) Вопрос 6: В одном комиксе одно из НИХ произносит: "Смотри, вон та толпа людей похожа на пони". Назовите ИХ. Ответ: Облака. Комментарий: Люди тоже пытаются увидеть в облаках различные предметы и животных. Упоминание комикса тоже не зря. Источник: (pic: 20140068.jpg) Автор: ??? Вопрос 7: Несмотря на то, что при отосклерозе не поражаются наковальня и молоточек, многие больные сравнивают шум в ушах при обострении заболевания со звуками, воспроизводимыми ИМИ. Представьте себе, но большинство из НИХ, на самом деле, хищники. Назовите ИХ. Ответ: Кузнечики. Комментарий: А тот, что попался Носову, был, судя по всему, редкостью. Источник: 1. http://www.ordodeus.ru/Ordo_Deus12_Otoskleroz.html 2. http://ru.wikipedia.org/wiki/Кузнечиковые Автор: Александр Губин (Челябинск) Вопрос 8: В октябре 2002 года произошло событие, которое вызвало широкий отклик среди химиков всего мира. Королевское химическое общество Великобритании удостоило ЕГО звания своего почетного члена. В решении указывается, что он успешно применял знание химии для решения жизненно важных вопросов. Назовите ЕГО имя и фамилию. Ответ: Шерлок Холмс. Источник: http://www.icensor.ru/block_lenta/index.php?ELEMENT_ID=201 Автор: Александр Губин (Челябинск) Вопрос 9: Согласно Википедии, пролежав полгода в больнице на грани жизни и смерти, Караваджо написал свой первый портрет "Больной Вакх". Напишите четыре буквы, которые мы пропустили в тексте этого вопроса. Ответ: Авто. Комментарий: А просто портреты он писал и задолго до того. Источник: http://ru.wikipedia.org/wiki/Караваджо Автор: Александр Губин (Челябинск) Вопрос 10: В 2005 году в Европейском Союзе ввели новые паспорта. Они содержат фото, отпечаток пальца, адрес проживания и данные о прививках. Документ и позволит без проблем пересекать внутриевропейские границы. Какое слово в тексте этого вопроса мы заменили? Ответ: Носа. Комментарий: Это паспорта для собак, а отпечаток носа у каждой собаки индивидуален. Источник: 1. http://eur-lex.europa.eu/LexUriServ/LexUriServ.do?uri=CELEX:32003D0803:EN:HTML 2. http://ru.wikipedia.org/wiki/Ветеринарный_паспорт Автор: Александр Губин (Челябинск) Вопрос 11: Герои фильма Стейси Татла приглашают к себе людей, с религиозными и политическими взглядами которых они не согласны, и подсыпают им цианистый калий в еду. Эта картина называется так же, как и другая, гораздо более известная работа. Напишите это название. Ответ: "Last supper". Зачет: "Тайная вечеря", "Последний ужин". Комментарий: Имеется в виду фреска Леонардо. Источник: 1. http://ru.wikipedia.org/wiki/Последний_ужин_(фильм,_1995) 2. http://ru.wikipedia.org/wiki/Тайная_вечеря_(Леонардо_да_Винчи) Автор: Александр Губин (Челябинск) Вопрос 12: Программист из сериала "Интерны", по его утверждению, помогал "бабушкам" попасть ТУДА. Судя по всему, дело было в 2012 года. Куда ТУДА? Ответ: На Евровидение. Комментарий: Бабушки, естественно, бурановские. а программист занимался накрутками. Источник: Т/с "Интерны", s03e10. Автор: Александр Губин (Челябинск) Вопрос 13: В 2010 году американская компания, строившая забор между США и Мексикой для предотвращения нелегальной иммиграции, была оштрафована почти на пять миллионов долларов. А за что? Ответ: За использование труда иммигрантов. Зачет: По смыслу. Источник: http://prikol.i.ua/view/501842/ Автор: Александр Губин (Челябинск) Вопрос 14: Недавно Министерство сельского хозяйства США решило стандартизировать ЭТОТ ПРОДУКТ. Отныне ОН будет делиться на три категории: крупнокалиберный, разнокалиберный и мелкокалиберный. В России в допетровскую эпоху ЭТОТ продукт получали естественным способом, без тепловой обработки, от чего он и получил свое название. Назовите ЭТОТ продукт коротким словом. Ответ: Сыр. Комментарий: Таким образом будут стандартизироваться дырки в швейцарском сыре. А слово "сыр" этимологически связано со словом "сырой". Источник: 1. http://ru.wikipedia.org/wiki/Сыр 2. "Наука и жизнь", 2003, N 8. Автор: Александр Губин (Челябинск) Вопрос 15: Если верить Андрею Водянику - создателю русской версии олимпийского факела, в его основе лежит уникальная система множественного горения: внутри факела горит огонь, от которого загорается огонь внешний. Если внешний вдруг потухнет от порыва ветра или дождя, то он тут же загорится снова от внутреннего пламени. По словам Андрея Водяника, при разработке факела он использовал принцип ЕЕ. Назовите ЕЕ. Ответ: Матрешка. Комментарий: Внутри внешнего огня есть еще и внутренний. Но ведь не помогло - олимпийский факел неожиданно погас уже несколько раз. Источник: http://www.sports.ru/sochi2014/153439504.html Автор: Александр Губин (Челябинск) Вопрос 16: Несмотря на то что олимпийский факел наполняют сжиженным газом, можно образно сказать, что внутри горит ОН. Считается, что впервые ЕГО прототип был применен еще в 424 году до н.э. Назовите ЕГО двумя словами. Ответ: Греческий огонь. Комментарий: Как известно, олимпийский огонь зажигают в Олимпии (Греция) за несколько месяцев до открытия игр с помощью параболического зеркала, фокусирующего лучи Солнца. Источник: 1. http://ru.wikipedia.org/wiki/Олимпийский_огонь 2. http://ru.wikipedia.org/wiki/Греческий_огонь Автор: Александр Губин (Челябинск) Вопрос 17: В одной из серий противник черепашек-ниндзя похищал ИХ. Один из НИХ заработал на банковских процентах несколько миллиардов долларов. Назовите ИХ двумя словами. Ответ: Разносчики пиццы. Зачет: По смыслу. Комментарий: Любовь черепашек к пицце широко известна. Филипп Дж. Фрай из м/с "Футурама" спустя 1000 лет криогенной заморозки получил на свои изначальные 93 цента 4,3 млрд. долларов. Источник: 1. М/с "Черепашки-ниндзя". 2. http://ru.wikipedia.org/wiki/Филип_Дж._Фрай Автор: Александр Губин (Челябинск) Вопрос 18: Авторы одного медицинского исследования отмечают, что пациенту, перенесшему операцию, следует в обязательном порядке подвергнуться ЭТОЙ процедуре, чтобы избежать осложнений. Назовите фамилию человека, в честь которого названа ЭТА процедура. Ответ: [Вильгельм Конрад] Рентген. Комментарий: Статья посвящена забывчивости хирургов, оставляющих инструменты внутри зашитого пациента. Источник: 1. http://www.fiziolog.isu.ru/page_5(D).htm 2. http://ru.wikipedia.org/wiki/Рентгенография Автор: Александр Губин (Челябинск) Вопрос 19: Согласно статистике Министерства здравоохранения, в США ежегодно 7000 человек умирают из-за НЕГО. ОН, как считается, является своеобразным шифром, нужным для того, чтобы заранее не напугать. Напишите же, как можно четче, причину гибели людей. Ответ: Непонятный почерк у врачей. Зачет: По смыслу. Источник: http://otvet.mail.ru/question/19618117 Автор: Александр Губин (Челябинск) Вопрос 20: Побывав на национальном татарском празднике "Сабантуй", Андрей Бедняков отметил, что конь для татарина - это друг, помощник и ... Закончите его замечание коротким словом. Ответ: Еда. Зачет: По смыслу с соблюдением формы. Источник: Телепередача "Орел и решка", s03e15. Автор: Александр Губин (Челябинск) Вопрос 21: Долгое время считалось, что ИХ всего два и они сплошные. А современные исследователи сравнивают ИХ с дорожками грампластинок. Назовите ИХ двумя словами. Ответ: Кольца Сатурна. Источник: 1. http://www.nkj.ru/archive/articles/710/ 2. http://ru.wikipedia.org/wiki/Сатурн#.D0.9A.D0.BE.D0.BB.D1.8C.D1.86.D0.B0 Автор: Александр Губин (Челябинск) Вопрос 22: На Урале в начале XX века многие крестьяне-работяги поднимали хозяйства с нуля, работали круглосуточно, а вместо подушки пользовались ЭТИМ. Назовите ЭТО. Ответ: Кулак. Комментарий: Так, по одной версии, произошло слово "кулак" в значении крепкого крестьянина. Источник: 1. http://www.rg.ru/2013/02/20/golodomor.html 2. http://ru.wikipedia.org/wiki/Кулак_(крестьянин) Автор: Александр Губин (Челябинск) Вопрос 23: Флорида испытывает сильное культурное и в какой-то степени политическое влияние со стороны эмигрантов из соседнего государства. Говоря об этом, Жанна Бадоева называет Флориду двумя словами. Напишите их. Ответ: Полуостров свободы. Комментарий: А соседняя страна - конечно, Куба - остров свободы. Источник: 1. Телепередача "Орел и решка", s03e04. 2. http://ru.wikipedia.org/wiki/Куба Автор: ??? Вопрос 24: Надменный герой фильма "Игрушка", сделав неожиданное предложение многодетной семье, дал им ЕЕ. Не торопитесь и назовите ЕЕ тремя словами. Ответ: Минута на размышление. Зачет: По смыслу с соблюдением формы. Источник: Х/ф "Игрушка", 57-я минута. Автор: Александр Губин (Челябинск) Тур: 26 тур. Тур оргкомитета N 2 Вопрос 1: <раздатка> "Архангел Михаил изыдя на Восток и виде звезду и взял слово от нее. Архангел Гавриил изыде на Запад и виде звезду и взял слово от нее. Рафаиил изыде от полудня и виде звезду и взял, слово от нее. Оурил изыде на полноч и виде звезду и взял, слово от нее. Принесли они слова перед Господом И рече Господь: чти Оруил. И рече Оруил ПЕРВЫЙ". Перед вами цитата из славянского апокрифа. Какое слово мы заменили на "ПЕРВЫЙ"? Ответ: Адам. Комментарий: По-гречески восток - Aratore, запад - Dysis, север - Akrtos, юг - Misebria. Источник: Энциклопедия символов, знаков и эмблем. - М.: Эксмо, 2005. - С. 20. Автор: Олег Холодов Вопрос 2: Вспоминая о ночевке в лесу, Николай Гумилёв пишет, что видел, как ОНА, опустив морду, принюхивается к чьему-то следу. ОНА хорошо видна на картине Винсента ван Гога "Звездная ночь над Роной". Назовите ЕЕ двумя словами, которые не начинаются на одну и ту же букву. Ответ: Большая медведица. Источник: http://ru.wikipedia.org/wiki/Большая_Медведица Автор: Ника Холодова Вопрос 3: Рассказ Рассела об одиноком смотрителе космического маяка в классическом переводе называется "Ниточка к сердцу". Однако переводчик М. Фроленок дал этой истории другое, более прозаическое, но не менее точное название, состоящее всего из одного слова. Как называется рассказ в этом переводе? Ответ: "Пуповина". Источник: В вопросе. Автор: ??? Вопрос 4: Этого человека, которому 1 мая 2013 года исполнилось бы 95 лет, каждый видит по-своему. Так, Эдуард Назаров считает, что он очень похож на Винни-Пуха, а Александр Татарский считает, что больше всего на него похож Бонифаций. А вот Владимир Зуйков уверен, что режиссер из мультфильма "Фильм! Фильм! Фильм!" - это вылитый ОН. Назовите его. Ответ: [Федор Савельевич] Хитрук. Комментарий: Мультипликатор. Источник: "Антенна", 2007, N 18. - С. 20. Автор: Олег Холодов Вопрос 5: В 1977 году в ФРГ появился новый знак, обозначающий экологически чистый продукт, который за короткое время получили около трех тысяч производителей. Создатель этого знака в первоначальном варианте хотел поместить на нем лицо великой актрисы. Как называется этот знак? Ответ: Голубой Ангел. Комментарий: Фотография Марлен Дитрих. "Голубой Ангел" - экранизация романа Генриха Манна "Профессор Унрат". Источник: http://ru.wikipedia.org/wiki/Голубой_ангел Автор: Олег Холодов Вопрос 6: Менее чем за год до ЭТОГО события соединенные силы христианских держав, разгромив при Лепанто турецкий флот, ознаменовали конец турецкой экспансии в Европу. Но папа Григорий XIII заявил, что ЭТО событие стоит пятидесяти таких побед. А испанский монарх Филипп II, узнав об ЭТОМ, впервые за много лет рассмеялся. Иван Грозный написал письмо Максимилиану II, в котором осудил ЭТО событие. Назовите ЭТО событие, если ему предшествовал "союз Марса и Венеры". Ответ: Варфоломеевская ночь. Комментарий: 10 августа 1572 года принц Генрих Конде женился на католичке Марии Клевской. А 24 августа 1572 года была Варфоломеевская ночь. Автор: Станислав Лещинский Вопрос 7: 12 сентября 2012 года менеджер по региональному развитию одной из российских компаний выставила свою ДОЧЬ на интернет-аукционе, требуя за столь необычный лот 500 тысяч рублей. В описании лота говорится, что РАБЫНЯ "находится в отличном состоянии, слегка поношена, без пробоин, ухожена, выглядит как новая". В 1901 году американский врач Дункан Макдугалл после серии экспериментов пришел к выводу, что вес ЦАРИЦЫ колеблется от 15 до 35 граммов. Какое слово мы заменили на "ДОЧЬ", "РАБЫНЯ" и "ЦАРИЦА"? Ответ: Душа. Комментарий: Замены взяты из знаменитого стихотворения Николая Заболоцкого "Не позволяй душе лениться": "Она рабыня и царица, Она работница и дочь, Она обязана трудиться И день и ночь, и день и ночь!". Источник: 1. http://www.audit-it.ru/news/soft/490254.html 2. http://ru.wikipedia.org/wiki/Душа 3. http://rupoem.ru/zabolockij/ne-pozvolyaj-dushe.aspx Автор: ??? Вопрос 8: Широко распространено заблуждение, что это произведение написал Шопен по просьбе Жорж Санд. На самом деле опус 64 ничего общего с этим произведением не имеет. Автор этого всемирно известного произведения доподлинно не известен, но существует мистификация, что его написал в XIX веке пятилетний немецкий мальчик Фердинанд Фло. А как это произведение называется в Германии и по сей день? Ответ: "Блошиный марш". Комментарий: Фамилия композитора образована от немецкого слова "блоха" (нем. Floh). Эта мелодия по-разному называется в разных странах. Разноязычные названия собачьего вальса свела воедино японский музыковед Хироми Окетани; по ее сведениям, в Германии он называется "блошиным вальсом" (нем. Flohwalzer), в Нидерландах - "блошиным маршем" (нидерл. Vlooien-Mars), в Венгрии - "ослиным маршем" (венг. Szamár-Induló), в Болгарии - "кошачьим маршем" (болг. Котешки марш), в Финляндии - "кошачьей полькой" (финск. Kissanpolkka) и т.д. Автор: Олег Холодов Вопрос 9: Гумилев считал ИХ такими же амулетами старых усадеб, как произведения барона Брамбеуса и Жан-Жака Руссо. А некий Иван Петрович утверждал, что может показать свое мастерство при условии, что ОНИ ему будут знакомы, и при этом упомянул число и предмет. Назовите этот предмет. Ответ: Карта. Комментарий: ОНИ - это пистолеты. "И не расстаться с амулетами, Фортуна катит колесо, На полке, рядом с пистолетами, Барон Брамбеус и Руссо". "- В тридцати шагах промаху в карту не дам, разумеется из знакомых пистолетов" ("Выстрел"). Источник: 1. http://alexnest.ru/russkaya-klassika/v-g/gumilev-n-s/nikolaj-gumilyov-starye-usadby/ 2. http://www.rvb.ru/pushkin/01text/06prose/01prose/0858.htm Автор: Станислав Лещинский Вопрос 10: ИХ часто сравнивают с представителями одного обширного семейства, у НИХ тяжелая судьба, "товарищи часто поворачиваются к ним спиной". Несмотря на расхожую фразу, ИХ недооценивают, а ИХ "врагов" превозносят. Но всё же один из НИХ смог достичь вершины. Назовите его. Ответ: Лев Яшин. Комментарий: ОНИ - это футбольные вратари. "Вратарь - половина команды". Большинство футбольных наград вручают нападающим или атакующим полузащитникам. Лев Яшин - единственный вратарь, который выиграл "Золотой мяч", случилось это в 1963 году. Источник: 1. ЛОАВ. 2. http://ru.wikipedia.org/wiki/Золотой_мяч_(France_Football) Автор: Олег Холодов Вопрос 11: Уважаемые знатоки! Вы хотя бы однажды читали "Камасутру"? Или, может быть, знакомились с древнеиндийским трактатом по искусству любви "Анангаранга"? А вот в Советском энциклопедическом словаре в статье "Порнография" упоминается только одно печатное издание. Назовите его. Ответ: Уголовный кодекс [СССР]. Источник: Советский энциклопедический словарь. - М.: Советская энциклопедия, 1982. - С. 1191. Автор: Олег Холодов Вопрос 12: Прозвище острова Сейбл, у берегов которого встречаются теплый Гольфстрим и холодное Лабрадорское течение, совпадает с названием предмета, подаренного адмиралу Нельсону по случаю победы над французским флотом у Абукира близким другом Нельсона капитаном корабля "Свифтмор" Веном Галлоуэллом. Надеемся, вам не составит труда воспроизвести это прозвище. Ответ: Гроб. Комментарий: Гроб был изготовлен из куска главной мачты захваченного французского корабля "Ориента". Источник: 1. http://ru.wikipedia.org/wiki/Сейбл 2. http://www.hrono.ru/statii/adm_nelson.html 3. http://full-control.narod.ru/grob/grob_mus.htm Автор: Олег Холодов Вопрос 13: В некоторых кругах "Мастера и Маргариту" Булгакова считают евангелием от ПЕРВОГО. Памятник ПЕРВОМУ чуть было не поставили в Свияжске - как мятежнику и революционеру, но потом остановились на фигуре ВТОРОГО, рангом пониже. По мнению Роберта Хайнлайна, подвиг ВТОРОГО намного выше, чем подвиг его соратников. Назовите и ПЕРВОГО, и ВТОРОГО. Ответ: Люцифер и Иуда Искариот. Источник: 1. "Итоги", 20.11.2006 г. - С. 116,. 2. Роберт Хайнлайн. Ересь. Автор: Олег Холодов Вопрос 14: Во время очередной эпидемии холеры в Петербурге губернатор Глазунов послал председателю санитарной комиссии телеграмму следующего содержания: "Холера идет на убыль. Буду рад, если петербуржцы [слово пропущено]". На что получил ответ: "Телеграмма неясна. На какой гласной в последнем слове вы ставите ударение?". Воспроизведите пропущенное слово. Ответ: Передохнут. Источник: http://www.piter.nev.ru/cgi-bin/gb/showbook.cgi?act=show&page=8&id=3 Автор: Станислав Лещинский Вопрос 15: В мире идет война, наибольшие потери несет Франция, отступает Италия, их теснят Австралия, Чили, ЮАР, США. К решительному наступлению готовится Аргентина. Льется кровь. Во всех городах и весях перекраиваются карты. Если вы вникли в суть проблемы, то легко ответите, о каких войнах идет речь. Ответ: Винные войны. Комментарий: Имелись в виду винные карты. Источник: "Эхо планеты", 2005, N 9. Автор: Станислав Лещинский Вопрос 16: В романе Роберта Шекли "Охотник-жертва" инструктор Симмсон говорит охотнику за людьми Блэквеллу: "Назначаю вам курс "Маскировка и методы убийства в тропиках". Там обучают кое-каким приемам, которые вам могут пригодиться. А заодно курс обеспечит вас тем, что иметь совершенно необходимо, если не желаете выглядеть белой вороной". Чем? Ответ: Загаром. Источник: Р. Шекли. Охотник-жертва. Автор: Оксана Вдовенко Вопрос 17: Среди простых американцев популярна горькая шутка о том, что некое слово является аббревиатурой. Знакомый автора шутит, что отправляется на это слово "в поисках более интересных задач". Напишите это слово абсолютно точно. Ответ: Job. Комментарий: Job - "just over broke" ("немногим лучше банкротства") Job.ru - система поиска работы и сотрудников. Источник: 1. http://www.dev.by/blogs/main/sekrety-nematerialnoy-motivatsii-istoriya-rona-avitsura 2. http://dic.academic.ru/dic.nsf/eng_rus_apresyan/52084/ Автор: Олег Холодов Вопрос 18: Генерал Костенецкий почитал русский язык родоначальником всех европейских языков, особенно французского. А какое слово он объяснял так: "Человек заперся в комнате своей, и кто ни пришел бы, хозяина как бы нет дома". Ответ: Кабинет. Источник: "Идеи вашего дома", 2005, N 1. - С. 177. Автор: Ника Холодова Вопрос 19: Внимание, в вопросе есть замена. Обладатель "Золотого глобуса" за саундтрек к фильму "Гладиатор" Дживан Гаспарян, исполнитель на дудуке, говорил: "Первый и последний этап подготовки к работе - это "небольшая медитация на гору". Настроение поднимается, пальцы бегают быстро, в мыслях ясность и чистота". Какие два слова мы заменили словосочетанием "медитация на гору"? Ответ: Доза "Арарата". Зачет: Доза коньяка и т.п. по смыслу. Источник: "Итоги", 25.12.2006 г. Автор: Олег Холодов Вопрос 20: Алекс Экслер предложил рецепт коктейля: 20 г водки (символ тирании), 30 г пургена (символ борьбы за свободу), 50 г лимонада "Буратино" (символ сына) и маленький райский фрукт. Как называется такой коктейль? Ответ: "Вильгельм Телль". Комментарий: Райское - маленькое - яблочко. Источник: http://popurri.narod.ru/top10/100/select118.html Автор: Станислав Лещинский Вопрос 21: Когда ПЕРВЫЙ услышал стихи ВТОРОГО, он совершенно искренне сказал: "Эти стихи гораздо лучше, чем мои", на что ВТОРОЙ скромно заметил: "Так и было задумано". Я не прошу вас назвать ни ПЕРВОГО, ни ВТОРОГО, хоть они вам должны быть хорошо известны. Назовите то, что ПЕРВЫЙ подарил ВТОРОМУ. Ответ: Горшок. Комментарий: Речь идет о Винни-Пухе и ослике Иа-Иа. Источник: А.А. Милн. Винни-Пух и все-все-все. Автор: Олег Холодов Вопрос 22: Время от времени в одном из штатов США проходит красочный фестиваль, воссоздающий жизнь, протекавшую здесь когда-то. Действие, разыгрываемое на фестивале, бывает щедро насыщено пушечной стрельбой, бряцанием холодного оружия и дымом пожаров, а участие в нем принимает множество актеров. Называется фестиваль "Крик дикого барана". А в каком штате он проходит? Ответ: Аляска. Комментарий: Праздник посвящен временам Русской Америки времен правления Александра Андреевича Баранова. Автор: Олег Холодов Вопрос 23: "Самый известный человек на планете, о котором никто ничего не знает". Какую историческую личность так назвал автор рецензии на один из фильмов с участием Джонни Деппа? Ответ: Джек Потрошитель. Комментарий: В фильме "Из ада" герой Деппа как раз охотится за Потрошителем. Источник: http://www.filmz.ru/pub/7/480_1.html Автор: Станислав Лещинский Вопрос 24: Местоположение одного из НИХ подскажет тиамин. На хрестоматийной картине русского художника, начатой в 1881 году и завершенной через 18 лет, ОН стоит рядом со своей противоположностью, изображенной по центру. Назовите имена, которые получил с легкой руки Дербенёва каждый из НИХ в известной песне. Ответ: Декабрь, Январь, Февраль. Комментарий: B1 - тиамин, а также координаты одного из белых шахматных коней. В 1881 Васнецов начал работу над картиной "Богатыри", но завершил ее лишь через 18 лет, т.к. получил заказ расписать строящийся в Киеве кафедральный Владимирский собор. Белый конь принадлежит Добрыне, черный - Илье. Леонид Дербенёв - автор слов песни "Три белых коня". Источник: 1. http://www.med2000.ru/farm/lb5/168.htm 2. http://www.hrono.ru/biograf/vasnecovv.html 3. http://www.detki-konfetki.com/music/19.html Автор: Сергей Антоновский Тур: 27 тур. Тур оргкомитета N 3 Редактор: Александр Берелехис Вопрос 1: В вопросе есть замена. "Горра была одной из самых старых человеческих колоний. Здесь еще иногда называли Терру Землей, в двух университетах открыли отделения археологии, а правящая фамилия возводила свое происхождение едва ли не к Адаму". Известно, что Адам был награжден медалью Колумба. Какую фамилию мы заменили на "Адам"? Ответ: Гагарин. Комментарий: Оба первые люди. Один - на Земле, второй - в космосе. Источник: 1. http://www.loveread.ec/read_book.php?id=533&p=62 2. http://zemljaki.mybb.ru/viewtopic.php?id=544&p=6 Автор: Олег Холодов Вопрос 2: Вот часть описания Довлатовым своего посещения зоопарка: "Верблюд был похож на моего учителя химии. Уссурийский тигр был приукрашенной копией Сталина. Орангутанг выглядел стареющим актером, за плечами которого бурная жизнь". После этого Довлатов делает заключение о гениальности одного человека. Назовите этого человека, если также известно, что, когда ему предложили стать мэром, тот ответил: "Зачем? Я и так уже король". Ответ: [Уолт] Дисней. Комментарий: Известно, что Диснея прозывали "мышиным королем". Источник: 1. Сергей Довлатов. Филиал. - СПб.: Азбука-Классика, 2007. 2. http://www.peoples.ru/undertake/cinema/disney/history2.html Автор: ??? Вопрос 3: Внимание, в вопросе есть замена. Во время матча в Новосибирске с местной "Сибирью" комментатор Сергей Гимаев назвал игроков "Адмирала" СУВОРОВЦАМИ. Его соведущий на это отметил, что СУВОРОВЦАМИ они станут только после игры в Омске. Какое слово мы заменили в тексте вопроса на СУВОРОВЦЕВ? Ответ: Колчаковцы. Комментарий: Аллюзии здесь, безусловно, на Колчака, который был адмиралом и в Омске был выбран Верховным правителем. Источник: Телетрансляция матча "Сибирь" - "Адмирал" на телеканале "Россия-2", 09.09.2013 г. Автор: ??? Вопрос 4: По ночам Альф из одноименного сериала смотрел на НЕЕ, которую предварительно клал в стиральную машину. Назовите ЕЕ. Ответ: [Игрушечная] овца. Комментарий: Скорее всего, для того чтобы уснуть, но это не доказано. Источник: Т/с "Альф", s01e04. Автор: ??? Вопрос 5: По версии Владимира Свержина, аббат Гвидо ответил ЕМУ: "Несомненно, друг мой. Но одному только Богу известно, как это ей до сих пор удается". Напишите ЕГО имя или фамилию, а можете имя и фамилию. Ответ: Галилео Галилей. Зачет: Галилео; Галилей. Комментарий: - И все-таки она вертится! (Галилео Галилей, ученый) - Несомненно, друг мой. Но одному только Богу известно, как это ей до сих пор удается. (Аббат Гвидо Маншич, инквизитор) Источник: http://www.aldebaran.ru/rufan/sverg/sverg1/ Автор: ??? Вопрос 6: В Англии принято аплодировать во время НЕЕ. Назовите ЕЕ двумя словами, начинающимися на одну и ту же букву. Ответ: Минута молчания. Источник: http://www.liverpoolfc.ru/forum/viewtopic.php?p=306423 Автор: ??? Вопрос 7: Внимание, в вопросе есть замены. Одна из шуток утверждает, что "постоянная Ньютона" расположена в конце ДОПИНГА. "ДОПИНГ" - название фильма о молодых хулиганах, которые в погоне за острыми ощущениями угоняли дорогие автомобили и разбивали их о витрины магазинов. Что мы заменили на "постоянная Ньютона"? Ответ: Точка G. Комментарий: Допинг - Shopping. Постоянная Ньютона - Гравитационная постоянная, обозначается обычно G. Буква П - седьмая в английском алфавите. Источник: 1. http://atkritka.com/120679/ 2. http://ru.wikipedia.org/wiki/Шоппинг_(фильм) Автор: ??? Вопрос 8: Хоккейный клуб "Эдмонтон Ойлерс" обладает огромными запасами талантов и, тем не менее, уже который год не показывает достойного результата. Комментируя эту ситуацию, журналист sports.ru пишет, что талантов в Эдмонтоне, как гейзеров в Исландии. Напишите слова, которые заменены на "гейзер" и "Исландию". Ответ: Нефть, Россия. Комментарий: Столь же не рачительное отношение они демонстрируют по отношению к имеющимся ресурсам. Источник: http://www.sports.ru/tribuna/blogs/895_mn/519582.html Автор: ??? Вопрос 9: На прогулках маленького Гумберта Гумберта частенько привечали пожилые дамы, которых Набоков сравнивает с НЕЙ. Если буквально перевести ЕЕ название с итальянского, то получится, что ОНА находится в ожидании. Назовите ЕЕ. Ответ: Пизанская башня. Комментарий: Пожилые американки склонялись над ним. Как Пизанская башня, которая как бы в ожидании своего падения на землю. z-checkdb: На самом деле "pendente" означает "в ожидании" на португальском языке (http://translate.google.com/#pt/ru/pendente), а с итальянского "torre pendente" переводится так, как и следует ожидать, - "наклонная башня" (http://slovari.yandex.ru/pendente/it-ru/#lingvo/) (Евгений Рубашкин). Источник: 1. http://lib.ru/NABOKOW/lolita.txt 2. http://ru.wikipedia.org/wiki/Пизанская_башня 3. http://translate.google.ru/#auto/ru/Torre%20pendente Автор: ??? Вопрос 10: Героине фильма "Крутая Джорджия" предлагают показать крышку, чтобы дело пошло скорее. Она отказывается, говоря, что для нее "ЭТО как жизнь - непонятно, что будет в конце". Назовите ЭТО. Ответ: Пазл. Комментарий: Собрать пазл по картинке гораздо проще, чем без нее. Источник: http://rutracker.org/forum/viewtopic.php?t=368778 Автор: ??? Вопрос 11: Пользователь форума сайта газеты "Спорт-экспресс" Битл61 после матча "Зенит" - ЦСКА, в котором Ахмед Муса не только не забил сам, но и помешал забить своему сокоманднику, обозвал Мусу ЦЫГАНОМ. Какое сложное слово мы заменили на ЦЫГАНА? Ответ: Конокрад. Комментарий: Надеемся, вашу победу никто украсть не сможет. Источник: http://football.sport-express.ru/reviews/36967/ Автор: ??? Вопрос 12: Журнал "Вокруг света" сообщает, что Ноэл ди Медейрус Роза, умерший в 1937 году, был известен как композитор-песенник, певец, виртуоз гитары, мандолины и банджо, а также как Великий марабу. Русская Википедия считает, что Великим марабу можно назвать Владимира Путина или Уго Чавеса. Какое слово мы заменили на "марабу"? Ответ: Самбист. Комментарий: В отличие от Путина, который является известным самбистом, и Уго Чавеса, который является самбистом по рождению (потомок от смешанных браков индейцев и негров), Ноэл ди Медейрус Роза достиг своих высот в песне и танце самба, а не в борьбе самбо. В некоторых странах Латинской Америки используются другие слова для обозначения потомков негро-индейских браков: в Бразилии - "кафУзу" (порт. cafuzo), в Мексике - "лОбо" (исп. Lobo), в Гаити - "марабУ" (фр. marabou), в Гондурасе, Белизе, Гватемале - "гарИфуна" (исп. garífuna). Источник: 1. "Вокруг света", 2009, N 5. - С. 46. 2. http://ru.wikipedia.org/wiki/Самбо 3. http://ru.wikipedia.org/wiki/Самбо_(этнографический_термин) Автор: Иван Морозов, Олег Холодов Вопрос 13: Немного озорную эпитафию сборной Канады по хоккею, проигравшей американцам в финале кубка мира, журнал "Коммерсанта" озаглавил словами "Канада СДЕЛАЛА ЭТО". Напишите два слова, которые мы заменили на "СДЕЛАЛА ЭТО". Ответ: Отбросила коньки. Источник: 1. http://www.kommersant.ru/doc/2159081 2. http://ru.wiktionary.org/wiki/отбросить_коньки Автор: ??? Вопрос 14: Ежегодно в середине лета тысячи туристов и местных жителей штурмуют небольшую скалу в штате Колорадо. Назовите ее словом французского происхождения. Ответ: Бастилия. Комментарий: 14 июля - День взятия Бастилии. Источник: http://otvet.mail.ru/question/7525049 Автор: ??? Вопрос 15: Ведущая телепередачи "Орел и решка", отстояв в очереди, заметила, что тысячи туристов ежедневно штурмуют ЕГО ворота. Напишите ЕГО название. Ответ: Эрмитаж. Зачет: Зимний дворец. Источник: 1. Телепередача "Орел и решка", s05e12. 2. http://ru.wikipedia.org/wiki/Штурм_Зимнего_дворца Автор: ??? Вопрос 16: Прослушайте цитату из повести Чехова "Ариадна": "Она мечтала о титуле, о блеске, но в то же время ей не хотелось упустить и меня. Когда меня любят, то я чувствую это даже на расстоянии, ..., тут же веяло на меня холодом, и когда она говорила мне о любви, то мне казалось, что я слышу ЕГО". Назовите ЕГО двумя словами. Ответ: Механический соловей. Зачет: Металлический соловей. Комментарий: Как и у Андерсена, механический соловей здесь противопоставляется истинным чувствам. Источник: http://www.feb-web.ru/feb/chekhov/texts/sp0/sp9/sp9-107-.htm Автор: ??? Вопрос 17: В Англии увеличилось число детей, приходящих в детские сады со слабым развитием речи. Специалисты утверждают, что причина в конструкции современных ИХ, ориентированных больше на внешний мир, а не на общение с родителями. Назовите ИХ двумя словами. Ответ: Детские коляски. Комментарий: В современных складных колясках малыш сидит лицом к движению и спиной к маме, поэтому она меньше с ним разговаривает. В старомодных больших детских колясках ребенок обращен лицом к везущей его матери, и потому контакт с ней, в том числе словесный, гораздо теснее, даже если он не понимает, что ему говорят. Источник: http://www.nkj.ru/archive/articles/6361/ Автор: ??? Вопрос 18: Внимание, в вопросе есть замена. В статье в газете "Ока-инфо", посвященной гастролям Евгения Григорьева в Серпухове, последний делится с читателями сокровенным: "Я продал Григорию Викторовичу "КАПЛЮ КРОВИ..." за 300 долларов". Сам покупатель торгует КАПЛЕЙ КРОВИ по цене от 7 долларов и выше. Известно, что КАПЛЯ КРОВИ часто фигурировала в обрядах лечебной магии. Какие слова мы заменили на КАПЛЮ КРОВИ? Ответ: Рюмка водки. Комментарий: Речь идет о песне "Рюмка водки на столе". Жека и Лепс. Источник: 1. http://oka-info.ru/news/article/19403/ 2. http://ru.tsn.ua/glamur/grigoriy-leps-otkryvaet-v-kieve-restoran-gde-ryumka-vodki-budet-stoit-60-grn-334046.html 3. http://forum.webgolos.com Автор: Олег Холодов Вопрос 19: Промоутер темнокожего боксера из фильма "Гладиатор" говорит про своего подопечного, что встреча с ним опасна для здоровья, но до НЕЕ ему все-таки далеко. Назовите либо ЕЕ одним словом, либо прозвище боксера двумя. Ответ: Чума. Зачет: Черная смерть. Источник: http://ru.wikipedia.org/wiki/Гладиатор_(фильм,_1992) Автор: ??? Вопрос 20: Дуплет. 1. Саша Гитри однажды заметил, что "читать полезно - глаза не так устают". Разгадав смысл этого афоризма, ответьте, какие два слова мы в нем пропустили. 2. Саша Гитри однажды заметил, что "если о женщине хотят сказать что-то хорошее, используют ЕГО, если плохое - ИХ". Что мы заменили на "ЕГО" и "ИХ"? Ответ: 1. "... между строк...". 2. Единственное число, множественное число. Источник: http://citaty.su/aforizmy-i-citaty-sashi-gitri Автор: Олег Холодов Вопрос 21: Внимание, в вопросе есть замены. Играть в ГОРОДКИ умели еще в Древнем Египте. В жаргоне рекламистов есть фраза "играть в ГОРОДКИ". Томас Хьюз, опираясь на поговорку, утверждал, что "ОНО и ГОРОДКИ или нечто лучшее в этом же роде должны составлять существенную часть образования каждого англичанина". Что мы заменили на "ОНО"? Ответ: Пиво. Комментарий: Жизнь - это не только пиво и кегли. Источник: 1. http://www.fizkultura-vsem.ru/кегли/ 2. http://www.designer31.ru/budni-dizajnera/reklamnyj-zhargon.htm 3. http://www.liveinternet.ru/users/3109898/post193291810/ Автор: Олег Холодов Вопрос 22: Внимание, в вопросе есть замена. Однажды Эдуард Багрицкий сказал: "Я лечу свою ДУШУ, когда читаю Мандельштама". Юстина Ковальчик назвала Олимпийские игры в Ванкувере "играми ДУШЕВНЫМИ". Какое слово мы заменили на ДУШУ? Ответ: Астма. Комментарий: "После этих соревнований я должна, пожалуй, сказать своему врачу, чтобы он и у меня нашел какую-нибудь астму. Из первой шестерки только я оказалась здоровой". Принцип замены: астма - удушение. Источник: 1. http://www.astmatik.net/Ivelikieneiskluchenie 2. http://www.vesti.ru/doc.html?id=343445 Автор: Олег Холодов Вопрос 23: Внимание, в вопросе есть замена. Французский философ и антрополог Леви-Брюль называл ЕГО "богом дикарей". ОЛДИ писали: "Этот день от НЕГО отделяли два века", а потом уточняли: "ОН подобен сонате". Что мы заменили на "ОН"? Ответ: Сон. Комментарий: Первая часть сонаты всегда быстрая, стремительная. Источник: http://www.gumer.info/bogoslov_Buks/Philos/kurg/03.php Автор: Олег Холодов Вопрос 24: (pic: 20140069.jpg) Последняя раздача! Число 648 является числом харшад и число Смита. По последним данным можно сказать, что число 648 является еще и ТАКИМ числом. Каким ТАКИМ? Ответ: Последним. Комментарий: Фотография называется "Последний". Всего за марафон было сыграно 24 x 27 = 648 вопросов. Число харшад - такое число, которое делится на сумму его цифр. Число Смита - такое составное число, сумма цифр которого (в некоторой системе счисления, обычно в десятичной) равняется сумме цифр всех его простых сомножителей с учетом кратности. Источник: http://ru.wikipedia.org/wiki/648_(число) Автор: Олег Холодов